Sei sulla pagina 1di 251

http://nurseslabs.

com/nclex-exam-gastrointestinal-disorders-180-items/

1. Topics
Included topics in this practice quiz are:

Liver Failure

Liver Disorders

Ostomy Care

Guidelines
Follow the guidelines below to make the most out of this exam:

Read each question carefully and choose the best answer.

You are given one minute per question. Spend your time wisely!

Answers and rationales are given below. Be sure to read them.

If you need more clarifications, please direct them to the comments section.

Question 1
Donald is a 61 y.o. man with diverticulitis. Diverticulitis is characterized by:
A Periodic rectal hemorrhage
B Hypertension and tachycardia
C Vomiting and elevated temperature
D Crampy and lower left quadrant pain and low-grade fever
Question 2
Youre advising a 21 y.o. with a colostomy who reports problems with flatus. What food
should you recommend?
A
Peas
B
Cabbage
C
Broccoli
D
Yogurt
Question 3

Youre caring for Beth who underwent a Billroth II procedure (surgical removal of the
pylorus and duodenum) for treatment of a peptic ulcer. Which findings suggest that the
patient is developing dumping syndrome, a complication associated with this procedure?
A
Flushed, dry skin.
B
Headache and bradycardia
C
Dizziness and sweating.
D
Dyspnea and chest pain
Question 4
Youre assessing the stoma of a patient with a healthy, well-healed colostomy. You expect
the stoma to appear:
A
Pale, pink and moist
B
Red and moist
C
Dark or purple colored
D
Dry and black
Question 5
After an abdominal resection for colon cancer, Madeline returns to her room with a
Jackson-Pratt drain in place. The purpose of the drain is to:
A Irrigate the incision with a saline solution
B Prevent bacterial infection of the incision
C Measure the amount of fluid lost after surgery
D Prevent accumulation of drainage in the wound
Question 6
Sharon has cirrhosis of the liver and develops ascites. What intervention is necessary to
decrease the excessive accumulation of serous fluid in her peritoneal cavity?
A
Restrict fluids
B
Encourage ambulation
C
Increase sodium in the diet
D
Give antacids as prescribed
Question 7
Youre performing an abdominal assessment on Brent who is 52 y.o. In which order do
you proceed?
A Observation, percussion, palpation, auscultation
B Observation, auscultation, percussion, palpation
C Percussion, palpation, auscultation, observation
D Palpation, percussion, observation, auscultation
Question 8
A 53 y.o. patient has undergone a partial gastrectomy for adenocarcinoma of the stomach.
An NG tube is in place and is connected to low continuous suction. During the immediate
postoperative period, you expect the gastric secretions to be which color?
A
Brown
B
Clear
C
Red
D
Yellow

Question 9
Youre caring for Betty with liver cirrhosis. Which of the following assessment findings
leads you to suspect hepatic encephalopathy in her?
A
Asterixis
B
Chvosteks sign
C
Trousseaus sign
D
Hepatojugular reflex
Question 10
Findings during an endoscopic exam include a cobblestone appearance of the colon in
your patient. The findings are characteristic of which disorder?
A
Ulcer
B
Crohns disease
C
Chronic gastritis
D
Ulcerative colitis
Question 11
Your patient with peritonitis is NPO and complaining of thirst. What is your priority?
A
Increase the I.V. infusion rate
B
Use diversion activities
C
Provide frequent mouth care
D
Give ice chips every 15 minutes
Question 12
Brenda, a 36 y.o. patient is on your floor with acute pancreatitis. Treatment for her
includes:
A
Continuous peritoneal lavage
B
Regular diet with increased fat
C
Nutritional support with TPN
D
Insertion of a T tube to drain the pancreas
Question 13
Leigh Ann is receiving pancrelipase (Viokase) for chronic pancreatitis. Which
observation best indicates the treatment is effective?
A There is no skin breakdown
B Her appetite improves
C She loses more than 10 lbs
D Stools are less fatty and decreased in frequency
Question 14
Your patient recently had abdominal surgery and tells you that he feels a popping
sensation in his incision during a coughing spell, followed by severe pain. You anticipate
an evisceration. Which supplies should you take to his room?
A A suture kit.
B
Sterile water and a suture kit.
C
Sterile water and sterile dressings.
D Sterile saline solution and sterile dressings.

Question 15
Dark, tarry stools indicate bleeding in which location of the GI tract?
A
Upper colon
B
Lower colon
C
Upper GI tract
D
Small intestine
Question 16
Annabelle is being discharged with a colostomy, and youre teaching her about colostomy
care. Which statement correctly describes a healthy stoma?
A At first, the stoma may bleed slightly when touched.
B The stoma should appear dark and have a bluish hue.
C A burning sensation under the stoma faceplate is normal.
D The stoma should remain swollen away from the abdomen.
Question 17
Britney, a 20 y.o. student is admitted with acute pancreatitis. Which laboratory findings
do you expect to be abnormal for this patient?
A Serum creatinine and BUN
B Alanine aminotransferase (ALT) and aspartate aminotransferase (AST)
C Serum amylase and lipase
D Cardiac enzymes
Question 18
Matt is a 49 y.o. with a hiatal hernia that you are about to counsel. Health care counseling
for Matt should include which of the following instructions?
A Restrict intake of high-carbohydrate foods
B
Increase fluid intake with meals
C
Increase fat intake
D Eat three regular meals a day
Question 19
A patient who underwent abdominal surgery now has a gaping incision due to delayed
wound healing. Which method is correct when you irrigate a gaping abdominal incision
with sterile normal saline solution, using a piston syringe?
A Rapidly instill a stream of irrigating solution into the wound.
B Apply a wet-to-dry dressing to the wound after the irrigation.
C Moisten the area around the wound with normal saline solution after the irrigation.
D Irrigate continuously until the solution becomes clear or all of the solution is used.
Question 20
Youre caring for a 28 y.o. woman with hepatitis B. Shes concerned about the duration of
her recovery. Which response isnt appropriate?
A Encourage her to not worry about the future.
B Encourage her to express her feelings about the illness.
C Discuss the effects of hepatitis B on future health problems.
D Provide avenues for financial counseling if she expresses the need.

Question 21
The student nurse is preparing a teaching care plan to help improve nutrition in a patient
with achalasia. You include which of the following:
A
Swallow foods while leaning forward
B
Omit fluids at mealtimes
C
Eat meals sitting upright
D
Avoid soft and semi soft foods
Question 22
Janice is waiting for discharge instructions after her herniorrhaphy. Which of the
following instructions do you include?
A Eat a low-fiber diet
B Resume heavy lifting in 2 weeks
C Lose weight, if obese
D Resume sexual activity once discomfort is gone
Question 23
Christina is receiving an enteral feeding that requires a concentration of 80 ml of
supplement mixed with 20 ml of water. How much water do you mix with an 8 oz
(240ml) can of feeding?
A
60 ml
B
70 ml
C
80 ml
D
90 ml
Question 24
A patient with Crohns disease is admitted after 4 days of diarrhea. Which of the
following urine specific gravity values do you expect to find in this patient?
A
1.005
B
1.011
C
1.020
D
1.030
Question 25
A patient has an acute upper GI hemorrhage. Your interventions include:
A Treating hypovolemia
B Treating hypervolemia
C Controlling the bleeding source
D Treating shock and diagnosing the bleeding source
Question 26
Your patient has a retractable gastric peptic ulcer and has had a gastric vagotomy. Which
factor increases as a result of vagotomy?
A
Peristalsis
B
Gastric acidity
C
Gastric motility
D
Gastric pH

Question 27
Eleanor, a 62 y.o. woman with diverticulosis is your patient. Which interventions would
you expect to include in her care?
A Low-fiber diet and fluid restrictions
B Total parenteral nutrition and bed rest
C High-fiber diet and administration of psyllium
D Administration of analgesics and antacids
Question 28
Youre patient is complaining of abdominal pain during assessment. What is your
priority?
A Auscultate to determine changes in bowel sounds.
B Observe the contour of the abdomen
C Palpate the abdomen for a mass.
D Percuss the abdomen to determine if fluid is present.
Question 29
Jason, a 22 y.o. accident victim, requires an NG tube for feeding. What should you
immediately do after inserting an NG tube for liquid enteral feedings?
A Aspirate for gastric secretions with a syringe
B Begin feeding slowly to prevent cramping
C Get an X-ray of the tip of the tube within 24 hours
D Clamp off the tube until the feedings begin
Question 30
You are developing a care plan on Sally, a 67 y.o. patient with hepatic encephalopathy.
Which of the following do you include?
A Administering a lactulose enema as ordered
B Encouraging a protein-rich diet
C Administering sedatives, as necessary
D Encouraging ambulation at least four times a day
Question 31
Youre discharging Nathaniel with hepatitis B. Which statement suggests understanding
by the patient?
A Now I can never get hepatitis again.
B I can safely give blood after 3 months.
C Ill never have a problem with my liver again, even if I drink alcohol.
D My family knows that if I get tired and start vomiting, I may be getting sick again.
Question 32
Stephanie, a 28 y.o. accident victim, requires TPN. The rationale for TPN is to provide:
A Necessary fluids and electrolytes to the body
B Complete nutrition by the I.V. route.
C Tube feedings for nutritional supplementation
D Dietary supplementation with liquid protein given between meals
Question 33

Your patient has a GI tract that is functioning, but has the inability to swallow foods.
Which is the preferred method of feeding for your patient?
A
TPN
B
PPN
C
NG feeding
D
Oral liquid supplements
Question 34
The student nurse is participating in colorectal cancer-screening program. Which patient
has the fewest risk factors for colon cancer?
A Janice, a 45 y.o. with a 25-year history of ulcerative colitis
B George, a 50 y.o. whose father died of colon cancer
C Herman, a 60 y.o. who follows a low-fat, high-fiber diet
D Sissy, a 72 y.o. with a history of breast cancer
Question 35
Before bowel surgery, Lee is to administer enemas until clear. During administration, he
complains of intestinal cramps. What do you do next?
A Discontinue the procedure
B Lower the height of the enema container
C Complete the procedure as quickly as possible
D Continue administration of the enema as ordered without making any adjustments
Question 36
Youre caring for a patient with a sigmoid colostomy. The stool from this colostomy is:
A
Formed
B
Semisolid
C
Semiliquid
D
Watery
Question 37
Katrina is diagnosed with lactose intolerance. To avoid complications with lack of
calcium in the diet, which food should be included in the diet?
A
Fruit
B
Whole grains
C
Milk and cheese products
D
Dark green, leafy vegetables
Question 38
Jerod is experiencing an acute episode of ulcerative colitis. Which is priority for this
patient?
A Replace lost fluid and sodium
B Monitor for increased serum glucose level from steroid therapy
C Restrict the dietary intake of foods high in potassium
D Note any change in the color and consistency of stools
Question 39

Michael, a 42 y.o. man is admitted to the med-surg floor with a diagnosis of acute
pancreatitis. His BP is 136/76, pulse 96, Resps 22 and temp 101. His past history includes
hyperlipidemia and alcohol abuse. The doctor prescribes an NG tube. Before inserting the
tube, you explain the purpose to patient. Which of the following is a most accurate
explanation?
A It empties the stomach of fluids and gas.
B It prevents spasms at the sphincter of Oddi.
C It prevents air from forming in the small intestine and large intestine.
D It removes bile from the gallbladder.
Question 40
Ralph has a history of alcohol abuse and has acute pancreatitis. Which lab value is most
likely to be elevated?
A
Calcium
B
Glucose
C
Magnesium
D
Potassium
Question 41
Glenda has cholelithiasis (gallstones). You expect her to complain of:
A Pain in the right upper quadrant, radiating to the shoulder
B Pain in the right lower quadrant, with rebound tenderness
C Pain in the left upper quadrant, with shortness of breath
D Pain in the left lower quadrant, with mild cramping
Question 42
A 29 y.o. patient has an acute episode of ulcerative colitis. What diagnostic test confirms
this diagnosis?
A
Barium Swallow.
B
Stool examination
C
Gastric analysis
D
Sigmoidoscopy
Question 43
Claire, a 33 y.o. is on your floor with a possible bowel obstruction. Which intervention is
priority for her?
A
Obtain daily weights
B
Measure abdominal girth
C
Keep strict intake and output
D
Encourage her to increase fluids
Question 44
Which stoma would you expect a malodorous, enzyme-rich, caustic liquid output that is
yellow, green, or brown?
A
Ileostomy
B
Ascending colostomy
C
Transverse colostomy
D
Descending colostomy

Question 45
Your patient, Christopher, has a diagnosis of ulcerative colitis and has severe abdominal
pain aggravated by movement, rebound tenderness, fever, nausea, and decreased urine
output. This may indicate which complication?
A
Fistula
B
Bowel perforation
C
Bowel obstruction
D
Abscess
Question 46
Hepatic encephalopathy develops when the blood level of which substance increases?
A
Ammonia
B
Amylase
C
Calcium
D
Potassium
Question 47
A patient with chronic alcohol abuse is admitted with liver failure. You closely monitor
the patients blood pressure because of which change that is associated with the liver
failure?
A
Hypoalbuminemia
B
Increased capillary permeability
C
Abnormal peripheral vasodilation
D
Excess rennin release from the kidneys
Question 48
Elmer is scheduled for a proctoscopy and has an I.V. The doctor wrote an order for 5mg
of I.V. diazepam (Valium). Which order is correct regarding diazepam?
A Give diazepam in the I.V. port closest to the vein.
B Mix diazepam with 50 ml of dextrose 5% in water and give over 15 minutes.
C Give diazepam rapidly I.V. to prevent the bloodstream from diluting the drug mixture.
D Question the order because I.V. administration of diazepam is contraindicated.
Question 49
Youre patient, post-op drainage of a pelvic abscess secondary to diverticulitis, begins to
cough violently after drinking water. His wound has ruptured and a small segment of the
bowel is protruding. Whats your priority?
Ask the patient what happened, call the doctor, and cover the area with a water-soaked
A
bedsheet.
B Obtain vital signs, call the doctor, and obtain emergency orders.
Have a CAN hold the wound together while you obtain vital signs, call the doctor and flex the
C
patients knees.
Have the doctor called while you remain with the patient, flex the patients knees, and cover
D
the wound with sterile towels soaked in sterile saline solution.
Question 50

An intubated patient is receiving continuous enteral feedings through a Salem sump tube
at a rate of 60ml/hr. Gastric residuals have been 30-40ml when monitored Q4H. You
check the gastric residual and aspirate 220ml. What is your first response to this finding?
A Notify the doctor immediately
B Stop the feeding, and clamp the NG tube
C Discard the 220ml, and clamp the NG tube
D Give a prescribed GI stimulant such as metoclopramide (Reglan)
Question 51
Youre caring for Jane, a 57 y.o. patient with liver cirrhosis who developed ascites and
requires paracentesis. Before her paracentesis, you instruct her to:
A
Empty her bladder
B
Lie supine in bed
C
Remain NPO for 4 hours
D
Clean her bowels with an enema
Question 52
Stephen is a 62 y.o. patient that has had a liver biopsy. Which of the following groups of
signs alert you to a possible pneumothorax?
A Dyspnea and reduced or absent breath sounds over the right lung
B Tachycardia, hypotension, and cool, clammy skin
C Fever, rebound tenderness, and abdominal rigidity
D Redness, warmth, and drainage at the biopsy site
Question 53
The student nurse is teaching the family of a patient with liver failure. You instruct them
to limit which foods in the patients diet?
A
Meats and beans
B
Butter and gravies
C
Potatoes and pastas
D
Cakes and pastries
Question 54
Youre preparing a patient with a malignant tumor for colorectal surgery and subsequent
colostomy. The patient tells you hes anxious. What should your initial step be in working
with this patient?
A Determine what the patient already knows about colostomies
B Show the patient some pictures of colostomies
C Arrange for someone who has a colostomy to visit the patient
D Provide the patient with written material about colostomy care
Question 55
Rob is a 46 y.o. admitted to the hospital with a suspected diagnosis of Hepatitis B. Hes
jaundiced and reports weakness. Which intervention will you include in his care?
A
Regular exercise
B
A low-protein diet.
C
Allow patient to select his meals.
D
Rest period after small, frequent meals.

Question 56
Kevin has a history of peptic ulcer disease and vomits coffee-ground emesis. What does
this indicate?
A He has fresh, active upper GI bleeding.
B He needs immediate saline gastric lavage.
C His gastric bleeding occurred 2 hours earlier.
D He needs a transfusion of packed RBCs.
Question 57
You have a patient with achalasia (incomplete muscle relaxtion of the GI tract, especially
sphincter muscles). Which medications do you anticipate to administer?
A
Isosorbide dinitrate (Isordil)
B
Digoxin (Lanoxin)
C
Captopril (Capoten)
D
Propranolol (Inderal)
Question 58
Youre caring for Lewis, a 67 y.o. patient with liver cirrhosis who developed ascites and
requires paracentesis. Relief of which symptom indicated that the paracentesis was
effective?
A
Pruritus
B
Dyspnea
C
Jaundice
D
Peripheral Neuropathy
Question 59
Arthur has a family history of colon cancer and is scheduled to have a sigmoidoscopy. He
is crying as he tells you, I know that I have colon cancer, too. Which response is most
therapeutic?
A I know just how you feel.
B You seem upset.
C Oh, dont worry about it, everything will be just fine.
D Why do you think you have cancer?
Question 60
Your teaching Anthony how to use his new colostomy. How much skin should remain
exposed between the stoma and the ring of the appliance?
A
1/16
B
1/4
C
1/2
D
1
Question 61
Anthony, a 60 y.o. patient, has just undergone a bowel resection with a colostomy. During
the first 24 hours, which of the following observations about the stoma should you report
to the doctor?
A
Pink color
B
Light edema

C
Small amount of oozing
D
Trickles of bright red blood
Question 62
Youre doing preoperative teaching with Gertrude who has ulcerative colitis who needs
surgery to create an ileoanal reservoir. Which information do you include?
A A reservoir is created that exits through the abdominal wall.
B A second surgery is required 12 months after the first surgery.
C A permanent ileostomy is created.
D The surgery occurs in two stages.
Question 63
Your patient Maria takes NSAIDS for her degenerative joint disease, has developed
peptic ulcer disease. Which drug is useful in preventing NSAID-induced peptic ulcer
disease?
A
Calcium carbonate (Tums)
B
Famotidine (Pepcid)
C
Misoprostol (Cytotec)
D
Sucralfate (Carafate)
Question 64
You have to teach ostomy self care to a patient with a colostomy. You tell the patient to
measure and cut the wafer:
A
To the exact size of the stoma
B
About 1/16 larger than the stoma
C
About 1/8 larger than the stoma
D
About 1/4 larger than the stoma
Question 65
Your goal is to minimize Davids risk of complications after a herniorrhaphy. You instruct
the patient to:
A Avoid the use of pain medication.
B Cough and deep breathe Q2H.
C Splint the incision if he cant avoid sneezing or coughing.
D Apply heat to scrotal swelling.
Question 66
Youre caring for Carin who has just had ileostomy surgery. During the first 24 hours
post-op, how much drainage can you expect from the ileostomy?
A
100 ml
B
500 ml
C
1500 ml
D
5000 ml
Question 67
Youre preparing a teaching plan for a 27 y.o. named Jeff who underwent surgery to close
a temporary ileostomy. Which nutritional guideline do you include in this plan?
A
There is no need to change eating habits

B
Eat six small meals a day
C
Eat the largest meal in the evening
D
Restrict fluid intake
Question 68
Anna is 45 y.o. and has a bleeding ulcer. Despite multiple blood transfusions, her HGB is
7.5g/dl and HCT is 27%. Her doctor determines that surgical intervention is necessary
and she undergoes partial gastrectomy. Postoperative nursing care includes:
A Giving pain medication Q6H
B Flushing the NG tube with sterile water
C Positioning her in high Fowlers position
D Keeping her NPO until the return of peristalsis
Question 69
Type A chronic gastritis can be distinguished from type B by its ability to:
A
Cause atrophy of the parietal cells
B
Affect only the antrum of the stomach
C
Thin the lining of the stomach walls
D
Decrease gastric secretions
Question 70
A patient has a severe exacerbation of ulcerative colitis. Long-term medications will
probably include:
A
Antacids
B
Antibiotics.
C
Corticosteroids
D
Histamine2-receptor blockers
Question 71
Gail is scheduled for a cholecystectomy. After completion of preoperative teaching, Gail
states,If I lie still and avoid turning after the operation, Ill avoid pain. Do you think this
is a good idea? What is the best response?
A Youll need to turn from side to side every 2 hours.
B Its always a good idea to rest quietly after surgery.
C The doctor will probably order you to lie flat for 24 hours.
D Why dont you decide about activity after you return from the recovery room?
Question 72
What information is correct about stomach cancer?
A Stomach pain is often a late symptom
B Surgery is often a successful treatment
C Chemotherapy and radiation are often successful treatments
D The patient can survive for an extended time with TPN
Question 73
Develop a teaching care plan for Angie who is about to undergo a liver biopsy. Which of
the following points do you include?
A Youll need to lie on your stomach during the test.

B Youll need to lie on your right side after the test.


C During the biopsy youll be asked to exhale deeply and hold it.
D The biopsy is performed under general anesthesia.
Question 74
George has a T tube in place after gallbladder surgery. Before discharge, what
information or instructions should be given regarding the T tube drainage?
A If there is any drainage, notify the surgeon immediately.
B The drainage will decrease daily until the bile duct heals.
C First, the drainage is dark green; then it becomes dark yellow.
D If the drainage stops, milk the tube toward the puncture wound.
Question 75
You promote hemodynamic stability in a patient with upper GI bleeding by:
A Encouraging oral fluid intake
B Monitoring central venous pressure
C Monitoring laboratory test results and vital signs
D Giving blood, electrolyte and fluid replacement
Question 76
Sitty, a 66 y.o. patient underwent a colostomy for ruptured diverticulum. She did well
during the surgery and returned to your med-surg floor in stable condition. You assess her
colostomy 2 days after surgery. Which finding do you report to the doctor?
A
Blanched stoma
B
Edematous stoma
C
Reddish-pink stoma
D
Brownish-black stoma
Question 77
Youre developing the plan of care for a patient experiencing dumping syndrome after a
Billroth II procedure. Which dietary instructions do you include?
A
Omit fluids with meals
B
Increase carbohydrate intake
C
Decrease protein intake
D
Decrease fat intake
Question 78
After abdominal surgery, your patient has a severe coughing episode that causes wound
evisceration. In addition to calling the doctor, which intervention is most appropriate?
A Irrigate the wound & organs with Betadine
B Cover the wound with a saline soaked sterile dressing
C Apply a dry sterile dressing & binder
D Push the organs back & cover with moist sterile dressings
Question 79
Nathaniel has severe pruritus due to having hepatitis B. What is the best intervention for
his comfort?
A
Give tepid baths

B
Avoid lotions and creams
C
Use hot water to increase vasodilation
D
Use cold water to decrease the itching
Question 80
Regina is a 46 y.o. woman with ulcerative colitis. You expect her stools to look like:
A
Watery and frothy
B
Bloody and mucous
C
Firm and well-formed
D
Alternating constipation and diarrhea
Once you are finished, click the button below. Any items you have not completed will be
marked incorrect.
Practice Mode: This is an interactive version of the Text Mode. All questions are given in
a single page and correct answers, rationales or explanations (if any) are immediately
shown after you have selected an answer.

NCLEX Exam: Gastrointestinal Disorders 1 (80 Items)

Question 1

A patient with chronic alcohol abuse is admitted with liver failure. You closely
monitor the patients blood pressure because of which change that is
associated with the liver failure?

Hypoalbuminemia

Increased capillary permeability

Abnormal peripheral vasodilation

Excess rennin release from the kidneys

Question 2

Youre assessing the stoma of a patient with a healthy, well-healed colostomy.


You expect the stoma to appear:

Pale, pink and moist

Red and moist

Dark or purple colored

Dry and black

Question 3

Youre caring for a patient with a sigmoid colostomy. The stool from this
colostomy is:

Formed

Semisolid

Semiliquid

Watery

Question 4

Youre advising a 21 y.o. with a colostomy who reports problems with flatus.
What food should you recommend?

Peas

Cabbage

Broccoli

Yogurt

Question 5

You have to teach ostomy self care to a patient with a colostomy. You tell the
patient to measure and cut the wafer:

To the exact size of the stoma

About 1/16 larger than the stoma

About 1/8 larger than the stoma

About 1/4 larger than the stoma

Question 6

Youre performing an abdominal assessment on Brent who is 52 y.o. In which


order do you proceed?

Observation, percussion, palpation, auscultation

Observation, auscultation, percussion, palpation

Percussion, palpation, auscultation, observation

Palpation, percussion, observation, auscultation

Question 7

Youre doing preoperative teaching with Gertrude who has ulcerative colitis
who needs surgery to create an ileoanal reservoir. Which information do you
include?

A A reservoir is created that exits through the abdominal wall.


B A second surgery is required 12 months after the first surgery.
C A permanent ileostomy is created.
D The surgery occurs in two stages.
Question 8

Youre caring for Carin who has just had ileostomy surgery. During the first 24
hours post-op, how much drainage can you expect from the ileostomy?

100 ml

500 ml

1500 ml

5000 ml

Question 9

Youre preparing a teaching plan for a 27 y.o. named Jeff who underwent
surgery to close a temporary ileostomy. Which nutritional guideline do you
include in this plan?

There is no need to change eating habits

Eat six small meals a day

Eat the largest meal in the evening

Restrict fluid intake

Question 10

Arthur has a family history of colon cancer and is scheduled to have a


sigmoidoscopy. He is crying as he tells you, I know that I have colon cancer,
too. Which response is most therapeutic?

I know just how you feel.

You seem upset.

Oh, dont worry about it, everything will be just fine.

D Why do you think you have cancer?


Question 11

Youre caring for Beth who underwent a Billroth II procedure (surgical removal
of the pylorus and duodenum) for treatment of a peptic ulcer. Which findings
suggest that the patient is developing dumping syndrome, a complication
associated with this procedure?

Flushed, dry skin.

Headache and bradycardia

Dizziness and sweating.

Dyspnea and chest pain

Question 12

Youre developing the plan of care for a patient experiencing dumping


syndrome after a Billroth II procedure. Which dietary instructions do you
include?

Omit fluids with meals

Increase carbohydrate intake

Decrease protein intake

Decrease fat intake

Question 13

Youre caring for Lewis, a 67 y.o. patient with liver cirrhosis who developed
ascites and requires paracentesis. Relief of which symptom indicated that the
paracentesis was effective?

Pruritus

Dyspnea

Jaundice

Peripheral Neuropathy

Question 14

Youre caring for Jane, a 57 y.o. patient with liver cirrhosis who developed
ascites and requires paracentesis. Before her paracentesis, you instruct her
to:

Empty her bladder

Lie supine in bed

Remain NPO for 4 hours

Clean her bowels with an enema

Question 15

After abdominal surgery, your patient has a severe coughing episode that
causes wound evisceration. In addition to calling the doctor, which
intervention is most appropriate?

A Irrigate the wound & organs with Betadine


B Cover the wound with a saline soaked sterile dressing
C Apply a dry sterile dressing & binder
D Push the organs back & cover with moist sterile dressings
Question 16

Youre caring for Betty with liver cirrhosis. Which of the following assessment
findings leads you to suspect hepatic encephalopathy in her?

Asterixis

Chvosteks sign

Trousseaus sign

Hepatojugular reflex

Question 17

You are developing a care plan on Sally, a 67 y.o. patient with hepatic
encephalopathy. Which of the following do you include?

Administering a lactulose enema as ordered

Encouraging a protein-rich diet

Administering sedatives, as necessary

Encouraging ambulation at least four times a day

Question 18

You have a patient with achalasia (incomplete muscle relaxtion of the GI


tract, especially sphincter muscles). Which medications do you anticipate to
administer?
Isosorbide dinitrate (Isordil)

Digoxin (Lanoxin)

Captopril (Capoten)

Propranolol (Inderal)

Question 19

The student nurse is preparing a teaching care plan to help improve nutrition
in a patient with achalasia. You include which of the following:

Swallow foods while leaning forward

Omit fluids at mealtimes

Eat meals sitting upright

Avoid soft and semi soft foods

Question 20

Britney, a 20 y.o. student is admitted with acute pancreatitis. Which


laboratory findings do you expect to be abnormal for this patient?

A Serum creatinine and BUN


B Alanine aminotransferase (ALT) and aspartate aminotransferase (AST)
C Serum amylase and lipase
D Cardiac enzymes
Question 21

A patient with Crohns disease is admitted after 4 days of diarrhea. Which of


the following urine specific gravity values do you expect to find in this
patient?

1.005

1.011

1.020

1.030

Question 22

Your goal is to minimize Davids risk of complications after a herniorrhaphy.


You instruct the patient to:

A Avoid the use of pain medication.


B Cough and deep breathe Q2H.
C Splint the incision if he cant avoid sneezing or coughing.
D Apply heat to scrotal swelling.
Question 23

Janice is waiting for discharge instructions after her herniorrhaphy. Which of


the following instructions do you include?

Eat a low-fiber diet

Resume heavy lifting in 2 weeks

Lose weight, if obese

Resume sexual activity once discomfort is gone

Question 24

Develop a teaching care plan for Angie who is about to undergo a liver biopsy.
Which of the following points do you include?

A Youll need to lie on your stomach during the test.


B Youll need to lie on your right side after the test.
C During the biopsy youll be asked to exhale deeply and hold it.
D The biopsy is performed under general anesthesia.
Question 25

Stephen is a 62 y.o. patient that has had a liver biopsy. Which of the following
groups of signs alert you to a possible pneumothorax?

A Dyspnea and reduced or absent breath sounds over the right lung
B Tachycardia, hypotension, and cool, clammy skin
C Fever, rebound tenderness, and abdominal rigidity
D Redness, warmth, and drainage at the biopsy site
Question 26

Michael, a 42 y.o. man is admitted to the med-surg floor with a diagnosis of


acute pancreatitis. His BP is 136/76, pulse 96, Resps 22 and temp 101. His
past history includes hyperlipidemia and alcohol abuse. The doctor prescribes

an NG tube. Before inserting the tube, you explain the purpose to patient.
Which of the following is a most accurate explanation?
A It empties the stomach of fluids and gas.
B It prevents spasms at the sphincter of Oddi.
C It prevents air from forming in the small intestine and large intestine.
D It removes bile from the gallbladder.
Question 27

Jason, a 22 y.o. accident victim, requires an NG tube for feeding. What should
you immediately do after inserting an NG tube for liquid enteral feedings?

Aspirate for gastric secretions with a syringe

Begin feeding slowly to prevent cramping

Get an X-ray of the tip of the tube within 24 hours

Clamp off the tube until the feedings begin

Question 28

Stephanie, a 28 y.o. accident victim, requires TPN. The rationale for TPN is to
provide:

A Necessary fluids and electrolytes to the body


B Complete nutrition by the I.V. route.
C Tube feedings for nutritional supplementation
D Dietary supplementation with liquid protein given between meals
Question 29

Type A chronic gastritis can be distinguished from type B by its ability to:

Cause atrophy of the parietal cells

Affect only the antrum of the stomach

Thin the lining of the stomach walls

Decrease gastric secretions

Question 30

Matt is a 49 y.o. with a hiatal hernia that you are about to counsel. Health
care counseling for Matt should include which of the following instructions?

Restrict intake of high-carbohydrate foods

Increase fluid intake with meals

Increase fat intake

Eat three regular meals a day

Question 31

Jerod is experiencing an acute episode of ulcerative colitis. Which is priority


for this patient?

A Replace lost fluid and sodium


B Monitor for increased serum glucose level from steroid therapy
C Restrict the dietary intake of foods high in potassium
D Note any change in the color and consistency of stools
Question 32

A 29 y.o. patient has an acute episode of ulcerative colitis. What diagnostic


test confirms this diagnosis?

Barium Swallow.

Stool examination

Gastric analysis

Sigmoidoscopy

Question 33

Eleanor, a 62 y.o. woman with diverticulosis is your patient. Which


interventions would you expect to include in her care?

Low-fiber diet and fluid restrictions

Total parenteral nutrition and bed rest

High-fiber diet and administration of psyllium

Administration of analgesics and antacids

Question 34

Regina is a 46 y.o. woman with ulcerative colitis. You expect her stools to look
like:

Watery and frothy

Bloody and mucous

Firm and well-formed

Alternating constipation and diarrhea

Question 35

Donald is a 61 y.o. man with diverticulitis. Diverticulitis is characterized by:

A Periodic rectal hemorrhage


B Hypertension and tachycardia
C Vomiting and elevated temperature
D Crampy and lower left quadrant pain and low-grade fever
Question 36

Brenda, a 36 y.o. patient is on your floor with acute pancreatitis. Treatment


for her includes:

Continuous peritoneal lavage

Regular diet with increased fat

Nutritional support with TPN

Insertion of a T tube to drain the pancreas

Question 37

Glenda has cholelithiasis (gallstones). You expect her to complain of:

A Pain in the right upper quadrant, radiating to the shoulder


B Pain in the right lower quadrant, with rebound tenderness
C Pain in the left upper quadrant, with shortness of breath
D Pain in the left lower quadrant, with mild cramping
Question 38

After an abdominal resection for colon cancer, Madeline returns to her room
with a Jackson-Pratt drain in place. The purpose of the drain is to:

Irrigate the incision with a saline solution

Prevent bacterial infection of the incision

Measure the amount of fluid lost after surgery

Prevent accumulation of drainage in the wound

Question 39

Anthony, a 60 y.o. patient, has just undergone a bowel resection with a


colostomy. During the first 24 hours, which of the following observations
about the stoma should you report to the doctor?

Pink color

Light edema

Small amount of oozing

Trickles of bright red blood

Question 40

Your teaching Anthony how to use his new colostomy. How much skin should
remain exposed between the stoma and the ring of the appliance?

1/16

1/4

1/2

Question 41

Claire, a 33 y.o. is on your floor with a possible bowel obstruction. Which


intervention is priority for her?

Obtain daily weights

Measure abdominal girth

Keep strict intake and output

Encourage her to increase fluids

Question 42

Your patient has a GI tract that is functioning, but has the inability to swallow
foods. Which is the preferred method of feeding for your patient?

TPN

PPN

NG feeding

Oral liquid supplements

Question 43

Youre patient is complaining of abdominal pain during assessment. What is


your priority?

Auscultate to determine changes in bowel sounds.

Observe the contour of the abdomen

Palpate the abdomen for a mass.

Percuss the abdomen to determine if fluid is present.

Question 44

Before bowel surgery, Lee is to administer enemas until clear. During


administration, he complains of intestinal cramps. What do you do next?

A Discontinue the procedure


B Lower the height of the enema container
C Complete the procedure as quickly as possible
D

Continue administration of the enema as ordered without making any


adjustments

Question 45

Leigh Ann is receiving pancrelipase (Viokase) for chronic pancreatitis. Which


observation best indicates the treatment is effective?

There is no skin breakdown

Her appetite improves

She loses more than 10 lbs

Stools are less fatty and decreased in frequency

Question 46

Ralph has a history of alcohol abuse and has acute pancreatitis. Which lab
value is most likely to be elevated?

Calcium

Glucose

Magnesium

Potassium

Question 47

Anna is 45 y.o. and has a bleeding ulcer. Despite multiple blood transfusions,
her HGB is 7.5g/dl and HCT is 27%. Her doctor determines that surgical
intervention is necessary and she undergoes partial gastrectomy.
Postoperative nursing care includes:

Giving pain medication Q6H

Flushing the NG tube with sterile water

Positioning her in high Fowlers position

Keeping her NPO until the return of peristalsis

Question 48

Sitty, a 66 y.o. patient underwent a colostomy for ruptured diverticulum. She


did well during the surgery and returned to your med-surg floor in stable
condition. You assess her colostomy 2 days after surgery. Which finding do
you report to the doctor?

Blanched stoma

Edematous stoma

Reddish-pink stoma

Brownish-black stoma

Question 49

Sharon has cirrhosis of the liver and develops ascites. What intervention is
necessary to decrease the excessive accumulation of serous fluid in her
peritoneal cavity?

Restrict fluids

Encourage ambulation

Increase sodium in the diet

Give antacids as prescribed

Question 50

Katrina is diagnosed with lactose intolerance. To avoid complications with lack


of calcium in the diet, which food should be included in the diet?

Fruit

Whole grains

Milk and cheese products

Dark green, leafy vegetables

Question 51

Nathaniel has severe pruritus due to having hepatitis B. What is the best
intervention for his comfort?

Give tepid baths

Avoid lotions and creams

Use hot water to increase vasodilation

Use cold water to decrease the itching

Question 52

Rob is a 46 y.o. admitted to the hospital with a suspected diagnosis of


Hepatitis B. Hes jaundiced and reports weakness. Which intervention will you
include in his care?

Regular exercise

A low-protein diet.

Allow patient to select his meals.

Rest period after small, frequent meals.

Question 53

Youre discharging Nathaniel with hepatitis B. Which statement suggests


understanding by the patient?

A Now I can never get hepatitis again.


B I can safely give blood after 3 months.

C Ill never have a problem with my liver again, even if I drink alcohol.
D

My family knows that if I get tired and start vomiting, I may be getting sick
again.

Question 54

Gail is scheduled for a cholecystectomy. After completion of preoperative


teaching, Gail states,If I lie still and avoid turning after the operation, Ill
avoid pain. Do you think this is a good idea? What is the best response?

A Youll need to turn from side to side every 2 hours.


B Its always a good idea to rest quietly after surgery.
C The doctor will probably order you to lie flat for 24 hours.
D Why dont you decide about activity after you return from the recovery room?
Question 55

Youre caring for a 28 y.o. woman with hepatitis B. Shes concerned about the
duration of her recovery. Which response isnt appropriate?

A Encourage her to not worry about the future.


B Encourage her to express her feelings about the illness.
C Discuss the effects of hepatitis B on future health problems.
D Provide avenues for financial counseling if she expresses the need.
Question 56

Elmer is scheduled for a proctoscopy and has an I.V. The doctor wrote an
order for 5mg of I.V. diazepam (Valium). Which order is correct regarding
diazepam?

A Give diazepam in the I.V. port closest to the vein.


B Mix diazepam with 50 ml of dextrose 5% in water and give over 15 minutes.
C

Give diazepam rapidly I.V. to prevent the bloodstream from diluting the drug
mixture.

D Question the order because I.V. administration of diazepam is contraindicated.


Question 57

Annabelle is being discharged with a colostomy, and youre teaching her


about colostomy care. Which statement correctly describes a healthy stoma?

A At first, the stoma may bleed slightly when touched.


B The stoma should appear dark and have a bluish hue.
C A burning sensation under the stoma faceplate is normal.
D The stoma should remain swollen away from the abdomen.
Question 58

A patient who underwent abdominal surgery now has a gaping incision due to
delayed wound healing. Which method is correct when you irrigate a gaping
abdominal incision with sterile normal saline solution, using a piston syringe?

A Rapidly instill a stream of irrigating solution into the wound.


B Apply a wet-to-dry dressing to the wound after the irrigation.
C

Moisten the area around the wound with normal saline solution after the
irrigation.

D Irrigate continuously until the solution becomes clear or all of the solution is used.
Question 59

Hepatic encephalopathy develops when the blood level of which substance


increases?

Ammonia

Amylase

Calcium

Potassium

Question 60

Your patient recently had abdominal surgery and tells you that he feels a
popping sensation in his incision during a coughing spell, followed by severe
pain. You anticipate an evisceration. Which supplies should you take to his
room?

A suture kit.

Sterile water and a suture kit.

Sterile water and sterile dressings.

Sterile saline solution and sterile dressings.

Question 61

Findings during an endoscopic exam include a cobblestone appearance of the


colon in your patient. The findings are characteristic of which disorder?

Ulcer

Crohns disease

Chronic gastritis

Ulcerative colitis

Question 62

What information is correct about stomach cancer?

A Stomach pain is often a late symptom


B Surgery is often a successful treatment
C Chemotherapy and radiation are often successful treatments
D The patient can survive for an extended time with TPN
Question 63

Dark, tarry stools indicate bleeding in which location of the GI tract?

Upper colon

Lower colon

Upper GI tract

Small intestine

Question 64

A patient has an acute upper GI hemorrhage. Your interventions include:

Treating hypovolemia

Treating hypervolemia

Controlling the bleeding source

Treating shock and diagnosing the bleeding source

Question 65

You promote hemodynamic stability in a patient with upper GI bleeding by:

Encouraging oral fluid intake

Monitoring central venous pressure

Monitoring laboratory test results and vital signs

Giving blood, electrolyte and fluid replacement

Question 66

Youre preparing a patient with a malignant tumor for colorectal surgery and
subsequent colostomy. The patient tells you hes anxious. What should your
initial step be in working with this patient?

A Determine what the patient already knows about colostomies


B Show the patient some pictures of colostomies
C Arrange for someone who has a colostomy to visit the patient
D Provide the patient with written material about colostomy care
Question 67

Your patient, Christopher, has a diagnosis of ulcerative colitis and has severe
abdominal pain aggravated by movement, rebound tenderness, fever,
nausea, and decreased urine output. This may indicate which complication?

Fistula

Bowel perforation

Bowel obstruction

Abscess

Question 68

A patient has a severe exacerbation of ulcerative colitis. Long-term


medications will probably include:

Antacids

Antibiotics.

Corticosteroids

Histamine2-receptor blockers

Question 69

The student nurse is teaching the family of a patient with liver failure. You
instruct them to limit which foods in the patients diet?

Meats and beans

Butter and gravies

Potatoes and pastas

Cakes and pastries

Question 70

An intubated patient is receiving continuous enteral feedings through a


Salem sump tube at a rate of 60ml/hr. Gastric residuals have been 30-40ml
when monitored Q4H. You check the gastric residual and aspirate 220ml.
What is your first response to this finding?

A Notify the doctor immediately


B Stop the feeding, and clamp the NG tube
C Discard the 220ml, and clamp the NG tube
D Give a prescribed GI stimulant such as metoclopramide (Reglan)
Question 71

Your patient with peritonitis is NPO and complaining of thirst. What is your
priority?

Increase the I.V. infusion rate

Use diversion activities

Provide frequent mouth care

Give ice chips every 15 minutes

Question 72

Kevin has a history of peptic ulcer disease and vomits coffee-ground emesis.
What does this indicate?

He has fresh, active upper GI bleeding.

He needs immediate saline gastric lavage.

His gastric bleeding occurred 2 hours earlier.

He needs a transfusion of packed RBCs.

Question 73

A 53 y.o. patient has undergone a partial gastrectomy for adenocarcinoma of


the stomach. An NG tube is in place and is connected to low continuous
suction. During the immediate postoperative period, you expect the gastric
secretions to be which color?

Brown

Clear

Red

Yellow

Question 74

Your patient has a retractable gastric peptic ulcer and has had a gastric
vagotomy. Which factor increases as a result of vagotomy?

Peristalsis

Gastric acidity

Gastric motility

Gastric pH

Question 75

Christina is receiving an enteral feeding that requires a concentration of 80


ml of supplement mixed with 20 ml of water. How much water do you mix
with an 8 oz (240ml) can of feeding?

60 ml

70 ml

80 ml

90 ml

Question 76

Which stoma would you expect a malodorous, enzyme-rich, caustic liquid


output that is yellow, green, or brown?
Ileostomy

Ascending colostomy

Transverse colostomy

Descending colostomy

Question 77

George has a T tube in place after gallbladder surgery. Before discharge, what
information or instructions should be given regarding the T tube drainage?

A If there is any drainage, notify the surgeon immediately.


B The drainage will decrease daily until the bile duct heals.
C First, the drainage is dark green; then it becomes dark yellow.
D If the drainage stops, milk the tube toward the puncture wound.
Question 78

Your patient Maria takes NSAIDS for her degenerative joint disease, has
developed peptic ulcer disease. Which drug is useful in preventing NSAIDinduced peptic ulcer disease?

Calcium carbonate (Tums)

Famotidine (Pepcid)

Misoprostol (Cytotec)

Sucralfate (Carafate)

Question 79

The student nurse is participating in colorectal cancer-screening program.


Which patient has the fewest risk factors for colon cancer?

A Janice, a 45 y.o. with a 25-year history of ulcerative colitis


B George, a 50 y.o. whose father died of colon cancer
C Herman, a 60 y.o. who follows a low-fat, high-fiber diet
D Sissy, a 72 y.o. with a history of breast cancer
Question 80

Youre patient, post-op drainage of a pelvic abscess secondary to


diverticulitis, begins to cough violently after drinking water. His wound has

ruptured and a small segment of the bowel is protruding. Whats your


priority?
A

Ask the patient what happened, call the doctor, and cover the area with a watersoaked bedsheet.

B Obtain vital signs, call the doctor, and obtain emergency orders.
C

Have a CAN hold the wound together while you obtain vital signs, call the doctor
and flex the patients knees.

Have the doctor called while you remain with the patient, flex the patients knees,
and cover the wound with sterile towels soaked in sterile saline solution.

Once you are finished, click the button below. Any items you have not
completed will be marked incorrect.

In Text Mode: All questions and answers are given for reading and answering at your
own pace. You can also copy this exam and make a print out.

1. A patient with chronic alcohol abuse is admitted with liver failure. You closely
monitor the patients blood pressure because of which change that is associated with
the liver failure?

1. Hypoalbuminemia
2. Increased capillary permeability
3. Abnormal peripheral vasodilation
4. Excess rennin release from the kidneys

2. Youre assessing the stoma of a patient with a healthy, well-healed colostomy. You
expect the stoma to appear:

1. Pale, pink and moist


2. Red and moist
3. Dark or purple colored
4. Dry and black

3. Youre caring for a patient with a sigmoid colostomy. The stool from this
colostomy is:

1. Formed
2. Semisolid
3. Semiliquid
4. Watery

4. Youre advising a 21 y.o. with a colostomy who reports problems with flatus.
What food should you recommend?

1. Peas
2. Cabbage
3. Broccoli
4. Yogurt

5. You have to teach ostomy self care to a patient with a colostomy. You tell the
patient to measure and cut the wafer:

1. To the exact size of the stoma.


2. About 1/16 larger than the stoma.
3. About 1/8 larger than the stoma.
4. About 1/4 larger than the stoma.

6. Youre performing an abdominal assessment on Brent who is 52 y.o. In which


order do you proceed?

1. Observation, percussion, palpation, auscultation


2. Observation, auscultation, percussion, palpation
3. Percussion, palpation, auscultation, observation
4. Palpation, percussion, observation, auscultation

7. Youre doing preoperative teaching with Gertrude who has ulcerative colitis who
needs surgery to create an ileoanal reservoir. Which information do you include?

1. A reservoir is created that exits through the abdominal wall.


2. A second surgery is required 12 months after the first surgery.
3. A permanent ileostomy is created.
4. The surgery occurs in two stages.

8. Youre caring for Carin who has just had ileostomy surgery. During the first 24
hours post-op, how much drainage can you expect from the ileostomy?

1. 100 ml
2. 500 ml
3. 1500 ml
4. 5000 ml

9. Youre preparing a teaching plan for a 27 y.o. named Jeff who underwent surgery
to close a temporary ileostomy. Which nutritional guideline do you include in this
plan?

1. There is no need to change eating habits.


2. Eat six small meals a day.
3. Eat the largest meal in the evening.
4. Restrict fluid intake.

10. Arthur has a family history of colon cancer and is scheduled to have a
sigmoidoscopy. He is crying as he tells you, I know that I have colon cancer, too.
Which response is most therapeutic?

1. I know just how you feel.


2. You seem upset.
3. Oh, dont worry about it, everything will be just fine.
4. Why do you think you have cancer?

11. Youre caring for Beth who underwent a Billroth II procedure (surgical removal
of the pylorus and duodenum) for treatment of a peptic ulcer. Which findings
suggest that the patient is developing dumping syndrome, a complication associated
with this procedure?

1. Flushed, dry skin.


2. Headache and bradycardia.
3. Dizziness and sweating.
4. Dyspnea and chest pain.

12. Youre developing the plan of care for a patient experiencing dumping syndrome
after a Billroth II procedure. Which dietary instructions do you include?

1. Omit fluids with meals.


2. Increase carbohydrate intake.
3. Decrease protein intake.
4. Decrease fat intake.

13. Youre caring for Lewis, a 67 y.o. patient with liver cirrhosis who developed
ascites and requires paracentesis. Relief of which symptom indicated that the
paracentesis was effective?

1. Pruritus
2. Dyspnea
3. Jaundice
4. Peripheral Neuropathy

14. Youre caring for Jane, a 57 y.o. patient with liver cirrhosis who developed
ascites and requires paracentesis. Before her paracentesis, you instruct her to:

1. Empty her bladder.


2. Lie supine in bed.
3. Remain NPO for 4 hours.
4. Clean her bowels with an enema.

15. After abdominal surgery, your patient has a severe coughing episode that causes
wound evisceration. In addition to calling the doctor, which intervention is most
appropriate?

1. Irrigate the wound & organs with Betadine.


2. Cover the wound with a saline soaked sterile dressing.
3. Apply a dry sterile dressing & binder.
4. Push the organs back & cover with moist sterile dressings.

16. Youre caring for Betty with liver cirrhosis. Which of the following assessment
findings leads you to suspect hepatic encephalopathy in her?

1. Asterixis
2. Chvosteks sign
3. Trousseaus sign
4. Hepatojugular reflex

17. You are developing a care plan on Sally, a 67 y.o. patient with hepatic
encephalopathy. Which of the following do you include?

1. Administering a lactulose enema as ordered.


2. Encouraging a protein-rich diet.
3. Administering sedatives, as necessary.
4. Encouraging ambulation at least four times a day.

18. You have a patient with achalasia (incomplete muscle relaxtion of the GI tract,
especially sphincter muscles). Which medications do you anticipate to administer?

1. Isosorbide dinitrate (Isordil)


2. Digoxin (Lanoxin)
3. Captopril (Capoten)
4. Propranolol (Inderal)

19. The student nurse is preparing a teaching care plan to help improve nutrition in
a patient with achalasia. You include which of the following:

1. Swallow foods while leaning forward.


2. Omit fluids at mealtimes.
3. Eat meals sitting upright.
4. Avoid soft and semi soft foods.

20. Britney, a 20 y.o. student is admitted with acute pancreatitis. Which laboratory
findings do you expect to be abnormal for this patient?

1. Serum creatinine and BUN


2. Alanine aminotransferase (ALT) and aspartate aminotransferase (AST)
3. Serum amylase and lipase
4. Cardiac enzymes

21. A patient with Crohns disease is admitted after 4 days of diarrhea. Which of the
following urine specific gravity values do you expect to find in this patient?

1. 1.005
2. 1.011
3. 1.020
4. 1.030

22. Your goal is to minimize Davids risk of complications after a herniorrhaphy.


You instruct the patient to:

1. Avoid the use of pain medication.


2. Cough and deep breathe Q2H.
3. Splint the incision if he cant avoid sneezing or coughing.
4. Apply heat to scrotal swelling.

23. Janice is waiting for discharge instructions after her herniorrhaphy. Which of
the following instructions do you include?

1. Eat a low-fiber diet.


2. Resume heavy lifting in 2 weeks.
3. Lose weight, if obese.
4. Resume sexual activity once discomfort is gone.

24. Develop a teaching care plan for Angie who is about to undergo a liver biopsy.
Which of the following points do you include?

1. Youll need to lie on your stomach during the test.


2. Youll need to lie on your right side after the test.
3. During the biopsy youll be asked to exhale deeply and hold it.
4. The biopsy is performed under general anesthesia.

25. Stephen is a 62 y.o. patient that has had a liver biopsy. Which of the following
groups of signs alert you to a possible pneumothorax?

1. Dyspnea and reduced or absent breath sounds over the right lung
2. Tachycardia, hypotension, and cool, clammy skin
3. Fever, rebound tenderness, and abdominal rigidity
4. Redness, warmth, and drainage at the biopsy site

26. Michael, a 42 y.o. man is admitted to the med-surg floor with a diagnosis of acute
pancreatitis. His BP is 136/76, pulse 96, Resps 22 and temp 101. His past history
includes hyperlipidemia and alcohol abuse. The doctor prescribes an NG tube.
Before inserting the tube, you explain the purpose to patient. Which of the following
is a most accurate explanation?

1. It empties the stomach of fluids and gas.


2. It prevents spasms at the sphincter of Oddi.
3. It prevents air from forming in the small intestine and large intestine.
4. It removes bile from the gallbladder.

27. Jason, a 22 y.o. accident victim, requires an NG tube for feeding. What should
you immediately do after inserting an NG tube for liquid enteral feedings?

1. Aspirate for gastric secretions with a syringe.


2. Begin feeding slowly to prevent cramping.
3. Get an X-ray of the tip of the tube within 24 hours.
4. Clamp off the tube until the feedings begin.

28. Stephanie, a 28 y.o. accident victim, requires TPN. The rationale for TPN is to
provide:

1. Necessary fluids and electrolytes to the body.


2. Complete nutrition by the I.V. route.
3. Tube feedings for nutritional supplementation.
4. Dietary supplementation with liquid protein given between meals.

29. Type A chronic gastritis can be distinguished from type B by its ability to:

1. Cause atrophy of the parietal cells.


2. Affect only the antrum of the stomach.
3. Thin the lining of the stomach walls.
4. Decrease gastric secretions.

30. Matt is a 49 y.o. with a hiatal hernia that you are about to counsel. Health care
counseling for Matt should include which of the following instructions?

1. Restrict intake of high-carbohydrate foods.


2. Increase fluid intake with meals.
3. Increase fat intake.
4. Eat three regular meals a day.

31. Jerod is experiencing an acute episode of ulcerative colitis. Which is priority for
this patient?

1. Replace lost fluid and sodium.


2. Monitor for increased serum glucose level from steroid therapy.
3. Restrict the dietary intake of foods high in potassium.
4. Note any change in the color and consistency of stools.

32. A 29 y.o. patient has an acute episode of ulcerative colitis. What diagnostic test
confirms this diagnosis?

1. Barium Swallow.
2. Stool examination.
3. Gastric analysis.
4. Sigmoidoscopy.

33. Eleanor, a 62 y.o. woman with diverticulosis is your patient. Which interventions
would you expect to include in her care?

1. Low-fiber diet and fluid restrictions.


2. Total parenteral nutrition and bed rest.
3. High-fiber diet and administration of psyllium.
4. Administration of analgesics and antacids.

34. Regina is a 46 y.o. woman with ulcerative colitis. You expect her stools to look
like:

1. Watery and frothy.


2. Bloody and mucous.
3. Firm and well-formed.
4. Alternating constipation and diarrhea.

35. Donald is a 61 y.o. man with diverticulitis. Diverticulitis is characterized by:

1. Periodic rectal hemorrhage.


2. Hypertension and tachycardia.
3. Vomiting and elevated temperature.
4. Crampy and lower left quadrant pain and low-grade fever.

36. Brenda, a 36 y.o. patient is on your floor with acute pancreatitis. Treatment for
her includes:

1. Continuous peritoneal lavage.


2. Regular diet with increased fat.
3. Nutritional support with TPN.
4. Insertion of a T tube to drain the pancreas.

37. Glenda has cholelithiasis (gallstones). You expect her to complain of:

1. Pain in the right upper quadrant, radiating to the shoulder.


2. Pain in the right lower quadrant, with rebound tenderness.
3. Pain in the left upper quadrant, with shortness of breath.
4. Pain in the left lower quadrant, with mild cramping.

38. After an abdominal resection for colon cancer, Madeline returns to her room
with a Jackson-Pratt drain in place. The purpose of the drain is to:

1. Irrigate the incision with a saline solution.


2. Prevent bacterial infection of the incision.

3. Measure the amount of fluid lost after surgery.


4. Prevent accumulation of drainage in the wound.

39. Anthony, a 60 y.o. patient, has just undergone a bowel resection with a
colostomy. During the first 24 hours, which of the following observations about the
stoma should you report to the doctor?

1. Pink color.
2. Light edema.
3. Small amount of oozing.
4. Trickles of bright red blood.

40. Your teaching Anthony how to use his new colostomy. How much skin should
remain exposed between the stoma and the ring of the appliance?

1. 1/16
2. 1/4
3. 1/2
4. 1

41. Claire, a 33 y.o. is on your floor with a possible bowel obstruction. Which
intervention is priority for her?

1. Obtain daily weights.


2. Measure abdominal girth.
3. Keep strict intake and output.
4. Encourage her to increase fluids.

42. Your patient has a GI tract that is functioning, but has the inability to swallow
foods. Which is the preferred method of feeding for your patient?

1. TPN
2. PPN
3. NG feeding
4. Oral liquid supplements

43. Youre patient is complaining of abdominal pain during assessment. What is


your priority?

1. Auscultate to determine changes in bowel sounds.


2. Observe the contour of the abdomen.

3. Palpate the abdomen for a mass.


4. Percuss the abdomen to determine if fluid is present.

44. Before bowel surgery, Lee is to administer enemas until clear. During
administration, he complains of intestinal cramps. What do you do next?

1. Discontinue the procedure.


2. Lower the height of the enema container.
3. Complete the procedure as quickly as possible.
4. Continue administration of the enema as ordered without making any adjustments.

45. Leigh Ann is receiving pancrelipase (Viokase) for chronic pancreatitis. Which
observation best indicates the treatment is effective?

1. There is no skin breakdown.


2. Her appetite improves.
3. She loses more than 10 lbs.
4. Stools are less fatty and decreased in frequency.

46. Ralph has a history of alcohol abuse and has acute pancreatitis. Which lab value
is most likely to be elevated?

1. Calcium
2. Glucose
3. Magnesium
4. Potassium

47. Anna is 45 y.o. and has a bleeding ulcer. Despite multiple blood transfusions, her
HGB is 7.5g/dl and HCT is 27%. Her doctor determines that surgical intervention is
necessary and she undergoes partial gastrectomy. Postoperative nursing care
includes:

1. Giving pain medication Q6H.


2. Flushing the NG tube with sterile water.
3. Positioning her in high Fowlers position.
4. Keeping her NPO until the return of peristalsis.

48. Sitty, a 66 y.o. patient underwent a colostomy for ruptured diverticulum. She did
well during the surgery and returned to your med-surg floor in stable condition. You
assess her colostomy 2 days after surgery. Which finding do you report to the
doctor?

1. Blanched stoma
2. Edematous stoma
3. Reddish-pink stoma
4. Brownish-black stoma

49. Sharon has cirrhosis of the liver and develops ascites. What intervention is
necessary to decrease the excessive accumulation of serous fluid in her peritoneal
cavity?

1. Restrict fluids
2. Encourage ambulation
3. Increase sodium in the diet
4. Give antacids as prescribed

50. Katrina is diagnosed with lactose intolerance. To avoid complications with lack
of calcium in the diet, which food should be included in the diet?

1. Fruit
2. Whole grains
3. Milk and cheese products
4. Dark green, leafy vegetables

51. Nathaniel has severe pruritus due to having hepatitis B. What is the best
intervention for his comfort?

1. Give tepid baths.


2. Avoid lotions and creams.
3. Use hot water to increase vasodilation.
4. Use cold water to decrease the itching.

52. Rob is a 46 y.o. admitted to the hospital with a suspected diagnosis of Hepatitis
B. Hes jaundiced and reports weakness. Which intervention will you include in his
care?

1. Regular exercise.
2. A low-protein diet.
3. Allow patient to select his meals.
4. Rest period after small, frequent meals.

53. Youre discharging Nathaniel with hepatitis B. Which statement suggests


understanding by the patient?

1. Now I can never get hepatitis again.


2. I can safely give blood after 3 months.
3. Ill never have a problem with my liver again, even if I drink alcohol.
4. My family knows that if I get tired and start vomiting, I may be getting sick again.

54. Gail is scheduled for a cholecystectomy. After completion of preoperative


teaching, Gail states,If I lie still and avoid turning after the operation, Ill avoid
pain. Do you think this is a good idea? What is the best response?

1. Youll need to turn from side to side every 2 hours.


2. Its always a good idea to rest quietly after surgery.
3. The doctor will probably order you to lie flat for 24 hours.
4. Why dont you decide about activity after you return from the recovery room?

55. Youre caring for a 28 y.o. woman with hepatitis B. Shes concerned about the
duration of her recovery. Which response isnt appropriate?

1. Encourage her to not worry about the future.


2. Encourage her to express her feelings about the illness.
3. Discuss the effects of hepatitis B on future health problems.
4. Provide avenues for financial counseling if she expresses the need.

56. Elmer is scheduled for a proctoscopy and has an I.V. The doctor wrote an order
for 5mg of I.V. diazepam(Valium). Which order is correct regarding diazepam?

1. Give diazepam in the I.V. port closest to the vein.


2. Mix diazepam with 50 ml of dextrose 5% in water and give over 15 minutes.
3. Give diazepam rapidly I.V. to prevent the bloodstream from diluting the drug mixture.
4. Question the order because I.V. administration of diazepam is contraindicated.

57. Annabelle is being discharged with a colostomy, and youre teaching her about
colostomy care. Which statement correctly describes a healthy stoma?

1. At first, the stoma may bleed slightly when touched.


2. The stoma should appear dark and have a bluish hue.
3. A burning sensation under the stoma faceplate is normal.
4. The stoma should remain swollen away from the abdomen.

58. A patient who underwent abdominal surgery now has a gaping incision due to
delayed wound healing. Which method is correct when you irrigate a gaping
abdominal incision with sterile normal saline solution, using a piston syringe?

1. Rapidly instill a stream of irrigating solution into the wound.


2. Apply a wet-to-dry dressing to the wound after the irrigation.
3. Moisten the area around the wound with normal saline solution after the irrigation.
4. Irrigate continuously until the solution becomes clear or all of the solution is used.

59. Hepatic encephalopathy develops when the blood level of which substance
increases?

1. Ammonia
2. Amylase
3. Calcium
4. Potassium

60. Your patient recently had abdominal surgery and tells you that he feels a
popping sensation in his incision during a coughing spell, followed by severe pain.
You anticipate an evisceration. Which supplies should you take to his room?

1. A suture kit.
2. Sterile water and a suture kit.
3. Sterile water and sterile dressings.
4. Sterile saline solution and sterile dressings.

61. Findings during an endoscopic exam include a cobblestone appearance of the


colon in your patient. The findings are characteristic of which disorder?

1. Ulcer
2. Crohns disease
3. Chronic gastritis
4. Ulcerative colitis

62. What information is correct about stomach cancer?

1. Stomach pain is often a late symptom.


2. Surgery is often a successful treatment.
3. Chemotherapy and radiation are often successful treatments.
4. The patient can survive for an extended time with TPN.

63. Dark, tarry stools indicate bleeding in which location of the GI tract?

1. Upper colon.
2. Lower colon.

3. Upper GI tract.
4. Small intestine.

64. A patient has an acute upper GI hemorrhage. Your interventions include:

1. Treating hypovolemia.
2. Treating hypervolemia.
3. Controlling the bleeding source.
4. Treating shock and diagnosing the bleeding source.

65. You promote hemodynamic stability in a patient with upper GI bleeding by:

1. Encouraging oral fluid intake.


2. Monitoring central venous pressure.
3. Monitoring laboratory test results and vital signs.
4. Giving blood, electrolyte and fluid replacement.

66. Youre preparing a patient with a malignant tumor for colorectal surgery and
subsequent colostomy. The patient tells you hes anxious. What should your initial
step be in working with this patient?

1. Determine what the patient already knows about colostomies.


2. Show the patient some pictures of colostomies.
3. Arrange for someone who has a colostomy to visit the patient.
4. Provide the patient with written material about colostomy care.

67. Your patient, Christopher, has a diagnosis of ulcerative colitis and has severe
abdominal pain aggravated by movement, rebound tenderness, fever, nausea, and
decreased urine output. This may indicate which complication?

1. Fistula.
2. Bowel perforation.
3. Bowel obstruction.
4. Abscess.

68. A patient has a severe exacerbation of ulcerative colitis. Long-term medications


will probably include:

1. Antacids.
2. Antibiotics.

3. Corticosteroids.
4. Histamine2-receptor blockers.

69. The student nurse is teaching the family of a patient with liver failure. You
instruct them to limit which foods in the patients diet?

1. Meats and beans.


2. Butter and gravies.
3. Potatoes and pastas.
4. Cakes and pastries.

70. An intubated patient is receiving continuous enteral feedings through a Salem


sump tube at a rate of 60ml/hr. Gastric residuals have been 30-40ml when
monitored Q4H. You check the gastric residual and aspirate 220ml. What is your
first response to this finding?

1. Notify the doctor immediately.


2. Stop the feeding, and clamp the NG tube.
3. Discard the 220ml, and clamp the NG tube.
4. Give a prescribed GI stimulant such as metoclopramide (Reglan).

71. Your patient with peritonitis is NPO and complaining of thirst. What is your
priority?

1. Increase the I.V. infusion rate.


2. Use diversion activities.
3. Provide frequent mouth care.
4. Give ice chips every 15 minutes.

72. Kevin has a history of peptic ulcer disease and vomits coffee-ground emesis.
What does this indicate?

1. He has fresh, active upper GI bleeding.


2. He needs immediate saline gastric lavage.
3. His gastric bleeding occurred 2 hours earlier.
4. He needs a transfusion of packed RBCs.

73. A 53 y.o. patient has undergone a partial gastrectomy for adenocarcinoma of the
stomach. An NG tube is in place and is connected to low continuous suction. During
the immediate postoperative period, you expect the gastric secretions to be which
color?

1. Brown.
2. Clear.
3. Red.
4. Yellow.

74. Your patient has a retractable gastric peptic ulcer and has had a gastric
vagotomy. Which factor increases as a result of vagotomy?

1. Peristalsis.
2. Gastric acidity.
3. Gastric motility.
4. Gastric pH.

75. Christina is receiving an enteral feeding that requires a concentration of 80 ml of


supplement mixed with 20 ml of water. How much water do you mix with an 8 oz
(240ml) can of feeding?

1. 60 ml.
2. 70 ml.
3. 80 ml.
4. 90 ml.

76. Which stoma would you expect a malodorous, enzyme-rich, caustic liquid output
that is yellow, green, or brown?

1. Ileostomy.
2. Ascending colostomy.
3. Transverse colostomy.
4. Descending colostomy.

77. George has a T tube in place after gallbladder surgery. Before discharge, what
information or instructions should be given regarding the T tube drainage?

1. If there is any drainage, notify the surgeon immediately.


2. The drainage will decrease daily until the bile duct heals.
3. First, the drainage is dark green; then it becomes dark yellow.
4. If the drainage stops, milk the tube toward the puncture wound.

78. Your patient Maria takes NSAIDS for her degenerative joint disease, has
developed peptic ulcer disease. Which drug is useful in preventing NSAID-induced
peptic ulcer disease?

1. Calcium carbonate (Tums)


2. Famotidine (Pepcid)
3. Misoprostol (Cytotec)
4. Sucralfate (Carafate)

79. The student nurse is participating in colorectal cancer-screening program.


Which patient has the fewest risk factors for colon cancer?

1. Janice, a 45 y.o. with a 25-year history of ulcerative colitis


2. George, a 50 y.o. whose father died of colon cancer
3. Herman, a 60 y.o. who follows a low-fat, high-fiber diet
4. Sissy, a 72 y.o. with a history of breast cancer

80. Youre patient, post-op drainage of a pelvic abscess secondary to diverticulitis,


begins to cough violently after drinking water. His wound has ruptured and a small
segment of the bowel is protruding. Whats your priority?

1. Ask the patient what happened, call the doctor, and cover the area with a water-soaked
bedsheet.
2. Obtain vital signs, call the doctor, and obtain emergency orders.
3. Have a CAN hold the wound together while you obtain vital signs, call the doctor and
flex the patients knees.
4. Have the doctor called while you remain with the patient, flex the patients knees, and
cover the wound with sterile towels soaked in sterile saline solution.

Answers and Rationale

1. Answer: 1. Hypoalbuminemia

Blood pressure decreases as the body is unable to maintain normal oncotic pressure with
liver failure, so patients with liver failure require close blood pressure monitoring.
Increased capillary permeability, abnormal peripheral vasodilation, and excess rennin
released from the kidneys arent direct ramifications of liver failure.

2. Answer: 2. Red and moist

Good circulation causes tissues to be moist and red, so a healthy, well-healed stoma
appears red and moist.

3. Answer: 1. Formed

A colostomy in the sigmoid colon produces a solid, formed stool.

4. Answer: 4. Yogurt

High-fiber foods stimulate peristalsis, and a result, flatus. Yogurt reduces gas formation.

5. Answer: 2. About 1/16 larger than the stoma.

A proper fit protects the skin, but doesnt impair circulation. A 1/16 should be cut.

6. Answer: 2. Observation, auscultation, percussion, palpation

Observation, auscultation, percussion, palpation

7. Answer: 4. The surgery occurs in two stages.

An ileoanal reservoir is created in two stages. The two surgeries are about 2 to 3 months
apart. First, diseased intestines are removed and a temporary loop ileostomy is created.
Second, the loop ileostomy is closed and stool goes to the reservoir and out through the
anus.

8. Answer: 3. 1500 ml

The large intestine absorbs large amounts of water so the initial output from the
ileostomy may be as much as 1500 to 2000 ml/24 hours. Gradually, the small intestine
absorbs more fluid and the output decreases.

9. Answer: 2. Eat six small meals a day

To avoid overloading the small intestine, encourage the patient to eat six small, regularly
spaced meals.

10. Answer: 2. You seem upset.

Making observations about what you see or hear is a useful therapeutic technique. This
way, you acknowledge that you are interested in what the patient is saying and feeling.

11. Answer: 3. Dizziness and sweating.

After a Billroth II procedure, a large amount of hypertonic fluid enters the intestine. This
causes extracellular fluid to move rapidly into the bowel, reducing circulating blood
volume and producing vasomotor symptoms. Vasomotor symptoms produced by
dumping syndrome include dizziness and sweating, tachycardia, syncope, pallor, and
palpitations.

12. Answer: 1. Omit fluids with meals.

Gastric emptying time can be delayed by omitting fluids from your patients meal. A diet
low in carbs and high in fat & protein is recommended to treat dumping syndrome.

13. Answer: 2. Dyspnea

Ascites puts pressure on the diaphragm. Paracentesis is done to remove fluid and
reducing pressure on the diaphragm. The goal is to improve the patients breathing. The
others are signs of cirrhosis that arent relieved by paracentesis.

14. Answer: 1. Empty her bladder.

A full bladder can interfere with paracentesis and be punctured inadvertently.

15. Answer: 2. Cover the wound with a saline soaked sterile dressing.

Cover the organs with a sterile, nonadherent dressing moistened with normal saline. Do
this to prevent infection and to keep the organs from drying out.

16. Anaswer: 1. Asterixis

Asterixis is an early neurologic sign of hepatic encephalopathy elicited by asking the


patient to hold her arms stretched out. Asterixis is present if the hands rapidly extend and
flex.

17. Answer: 1. Administering a lactulose enema as ordered.

You may administer the laxative lactulose to reduce ammonia levels in the colon.

18. Answer: 1. Isosorbide dinitrate (Isordil)

Achalasia is characterized by incomplete relaxation of the LES, dilation of the lower


esophagus, and a lack of esophageal peristalsis. Because nitrates relax the lower
esophageal sphincter, expect to give Isordil orally or sublingually.

19. Answer: 3. Eat meals sitting upright.

Eating in the upright position aids in emptying the esophagus. Doing the opposite of the
other three also may be helpful.

20. Answer: 3. Serum amylase and lipase

Pancreatitis involves activation of pancreatic enzymes, such as amylase and lipase. These
levels are elevated in a patient with acute pancreatitis.

21. Answer: 4. 1.030

The normal range of specific gravity of urine is 1.010 to 1.025; a value of 1.030 may be
seen with dehydration.

22. Answer: 3. Splint the incision if he cant avoid sneezing or coughing.

Teach the pt to avoid activities that increase intra-abdominal pressure such as coughing,
sneezing, or straining with a bowel movement.

23. Answer: 3. Lose weight, if obese.

Because obesity weakens the abdominal muscles, advise weight loss for the patient who
has had a hernia repair.

24. Answer: 2. Youll need to lie on your right side after the test.

After a liver biopsy, the patient is placed on the right side to compress the liver and to
reduce the risk of bleeding or bile leakage.

25. Answer: 1. Dyspnea and reduced or absent breath sounds over the right lung

Signs and Symptoms of pneumothorax include dyspnea and decreased or absent breath
sounds over the affected lung (right lung).

26. Answer: 1. It empties the stomach of fluids and gas.

An NG tube is inserted into the patients stomach to drain fluid and gas.

27. Answer: 1. Aspirate for gastric secretions with a syringe.

Aspirating the stomach contents confirms correct placement. If an X-ray is ordered, it


should be done immediately, not in 24 hours.

28. Answer: 2. Complete nutrition by the I.V. route.

TPN is given I.V. to provide all the nutrients your patient needs. TPN isnt a tube feeding
nor is it a liquid dietary supplement.

29. Answer: 1. Cause atrophy of the parietal cells.

Type A causes changes in parietal cells.

30. Answer: 2. Increase fluid intake with meals.

Increasing fluids helps empty the stomach. A high carb diet isnt restricted and fat intake
shouldnt be increased.

31. Answer: 1. Replace lost fluid and sodium.

Diarrhea d/t an acute episode of ulcerative colitis leads to fluid & electrolyte losses so
fluid replacement takes priority.

32. Answer: 4. Sigmoidoscopy.

Sigmoidoscopy allows direct observation of the colon mucosa for changes, and if needed,
biopsy.

33. Answer: 3. High-fiber diet and administration of psyllium.

She needs a high-fiber diet and a psyllium (bulk laxative) to promote normal soft stools.

34. Answer: 2. Bloody and mucous.

Stools from ulcerative colitis are often bloody and contain mucus.

35. Answer: 4. Crampy and lower left quadrant pain and low-grade fever.

One sign of acute diverticulitis is crampy lower left quadrant pain. A low-grade fever is
another common sign.

36. Answer: 3. Nutritional support with TPN.

With acute pancreatitis, you need to rest the GI tract by TPN as nutritional support.

37. Answer: 1. Pain in the right upper quadrant, radiating to the shoulder.

The gallbladder is located in the RUQ and a frequent sign of gallstones is pain radiating
to the shoulder.

38. Answer: 4. Prevent accumulation of drainage in the wound.

A Jackson-Pratt drain promotes wound healing by allowing fluid to escape from the
wound.

39. Answer: 4. Trickles of bright red blood.

After creation of a colostomy, expect to see a stoma that is pink, slightly edematous, with
some oozing. Bright red blood, regardless of amount, indicates bleeding and should be
reported to the doctor.

40. Answer: 1. 1/16

Only a small amount of skin should be exposed and more than 1/16 of skin allows the
excrement to irritate the skin.

41. Answer: 2. Measure abdominal girth.

Measuring abdominal girth provides quantitative information about increases or


decreases in the amount of distention.

42. Answer: 3. NG feeding

Because the GI tract is functioning, feeding methods involve the enteral route which
bypasses the mouth but allows for a major portion of the GI tract to be used.

43. Answer: 2. Observe the contour of the abdomen.

The first step in assessing the abdomen is to observe its shape and contour, then
auscultate, palpate, and then percuss.

44. Answer: 2. Lower the height of the enema container.

Lowering the height decreases the amount of flow, allowing him to tolerate more fluid.

45. Answer: 4. Stools are less fatty and decreased in frequency.

Pancrelipase provides the exocrine pancreatic enzyme necessary for proper protein, fat,
and carb digestion. With increased fat digestion and absorption, stools become less
frequent and normal in appearance.

46. Answer: 2. Glucose

Glucose level increases and diabetes mellitus may result d/t the pancreatic damage to the
islets of langerhans.

47. Answer: 4. Keeping her NPO until the return of peristalsis.

After surgery, she remains NPO until peristaltic activity returns. This decreases the risk
for abdominal distention and obstruction.

48. Answer: 4. Brownish-black stoma

A brownish-black color indicates lack of blood flow, and maybe necrosis.

49. Answer: 1. Restrict fluids

Restricting fluids decrease the amount of body fluid and the accumulation of fluid in the
peritoneal space.

50. Answer: 4. Dark green, leafy vegetables

Dark green, leafy vegetables are rich in calcium.

51. Answer: 1. Give tepid baths.

For pruritus, care should include tepid sponge baths and use of emollient creams and
lotions.

52. Answer: 4. Rest period after small, frequent meals.

Rest periods and small frequent meals is indicated during the acute phase of hepatitis B.

53. Answer: 4. My family knows that if I get tired and start vomiting, I may be
getting sick again.

Hepatitis B can recur. Patients who have had hepatitis are permanently barred from
donating blood. Alcohol is metabolized by the liver and should be avoided by those who
have or had hepatitis B.

54. Answer: 1. Youll need to turn from side to side every 2 hours.

To prevent venous stasis and improve muscle tone, circulation, and respiratory function,
encourage her to move after surgery.

55. Answer: 1. Encourage her to not worry about the future.

Telling her not to worry minimizes her feelings.

56. Answer: 1. Give diazepam in the I.V. port closest to the vein.

Diazepam is absorbed by the plastic I.V. tubing and should be given in the port closest to
the vein.

57. Answer: 1. At first, the stoma may bleed slightly when touched.

For the first few days to a week, slight bleeding normally occurs when the stoma is
touched because the surgical site is still new. She should report profuse bleeding
immediately.

58. Answer: 4. Irrigate continuously until the solution becomes clear or all of the
solution is used.

To wash away tissue debris and drainage effectively, irrigate the wound until the solution
becomes clear or all the solution is used.

59. Answer: 1. Ammonia

Ammonia levels increase d/t improper shunting of blood, causing ammonia to enter
systemic circulation, which carries it to the brain.

60. Answer: 4. Sterile saline solution and sterile dressings.

Saline solution is isotonic, or close to body fluids in content, and is used along with
sterile dressings to cover an eviscerated wound and keep it moist.

61. Answer: 2. Crohns disease

Crohns disease penetrates the mucosa of the colon through all layers and destroys the
colon in patches, which creates a cobblestone appearance.

62. Answer: 1. Stomach pain is often a late symptom.

Stomach pain is often a late sign of stomach cancer; outcomes are particularly poor when
the cancer reaches that point. Surgery, chemotherapy, and radiation have minimal positive
effects. TPN may enhance the growth of the cancer.

63. Answer: 3. Upper GI tract.

Melena is the passage of dark, tarry stools that contain a large amount of digested blood.
It occurs with bleeding from the upper GI tract.

64. Answer: 1. Treating hypovolemia.

A patient with an acute upper GI hemorrhage must be treated for hypovolemia and
hemorrhagic shock. You as a nurse cant diagnose the problem. Controlling the bleeding
may require surgery or intensive medical treatment.

65. Answer: 4. Giving blood, electrolyte and fluid replacement.

To stabilize a patient with acute bleeding, NS or LR solution is given I.V. until BP rises
and urine output returns to 30ml/hr.

66. Answer: 1. Determine what the patient already knows about colostomies.

Initially, you should assess the patients knowledge about colostomies and how it will
affect his lifestyle.

67. Answer: 2. Bowel perforation.

An inflammatory condition that affects the surface of the colon, ulcerative colitis causes
friability and erosions with bleeding. Patients with ulcerative colitis are at increased risk
for bowel perforation, toxic megacolon, hemorrhage, cancer, and other anorectal and
systemic complications.

68. Answers: 3. Corticosteroids.

Medications to control inflammation such as corticosteroids are used for long-term


treatment.

69. Answer: 1. Meats and beans.

Meats and beans are high-protein foods. In liver failure, the liver is unable to metabolize
protein adequately, causing protein by-products to build up in the body rather than be
excreted.

70. Answer: 2. Stop the feeding, and clamp the NG tube.

A gastric residual greater than 2 hours worth of feeding or 100-150ml is considered too
high. The feeding should be stopped; NG tube clamped, and then allow time for the
stomach to empty before additional feeding is added.

71. Answer: 3. Provide frequent mouth care.

Frequent mouth care helps relieve dry mouth.

72. Answer: 3. His gastric bleeding occurred 2 hours earlier.

Coffee-ground emesis occurs when there is upper GI bleeding that has undergone gastric
digestion. For blood to appear as coffee-ground emesis, it would have to be digested for
approximately 2 hours.

73. Answer: 3. Red.

Normally, drainage is bloody for the first 24 hours after a partial gastrectomy; then it
changes to brown-tinged and then to yellow or clear.

74. Answer: 4. Gastric pH.

If the vagus nerve is cut as it enters the stomach, gastric acid secretion is decreased, but
intestinal motility is also decreased and gastric emptying is delayed. Because gastric
acids are decreased, gastric pH increases.

75. Answer: 1. 60 ml.

Dosage problem. Its 80/20 = 240/X. X=60.

76. Answer: 1. Ileostomy.

The output from an Ileostomy is described.

77. Answer: 2. The drainage will decrease daily until the bile duct heals.

As healing occurs from the bile duct, bile drains from the tube; the amount of bile should
decrease. Teach the patient to expect dark green drainage and to notify the doctor if
drainage stops.

78. Answer: 3. Misoprostol (Cytotec)

Misoprostol restores prostaglandins that protect the stomach from NSAIDS, which
diminish the prostaglandins.

79. Answer: 3. Herman, a 60 y.o. who follows a low-fat, high-fiber diet

80. Answer: 4. Have the doctor called while you remain with the patient, flex the
patients knees, and cover the wound with sterile towels soaked in sterile saline
solution.

2. Topics
Included topics in this practice quiz are:

Appendicitis

Diverticulitis

Inflammatory Bowel Diseases

Guidelines
Follow the guidelines below to make the most out of this exam:

Read each question carefully and choose the best answer.

You are given one minute per question. Spend your time wisely!

Answers and rationales are given below. Be sure to read them.

If you need more clarifications, please direct them to the comments section.

In Exam Mode: All questions are shown in random and the results, answers and
rationales (if any) will only be given after youve finished the quiz. You are given 1
minute per question, a total of 100 minutes for this exam.

NCLEX Exam: Gastrointestinal Disorders 2 (100 Items)

99:14

Question 1

The nurse is monitoring a client for the early signs of dumping syndrome.
Which symptom indicates this occurrence?
Abdominal cramping and pain

Bradycardia and indigestion

Sweating and pallor

Double vision and chest pain

Question 2

Medical management of the client with diverticulitis should include which of


the following treatments?

Reduced fluid intake

Increased fiber in diet

Administration of antibiotics

Exercises to increase intra-abdominal pressure

Question 3

Which of the following factors is believed to be linked to Crohns disease?

Constipation

Diet

Hereditary

Lack of exercise

Question 4

Which goal of the clients care should take priority during the first days of
hospitalization for an exacerbation of ulcerative colitis?

Promoting self-care and independence

Managing diarrhea

Maintaining adequate nutrition

Promoting rest and comfort

Question 5

Which of the following diagnostic tests may be performed to determine if a


client has gastric cancer?
Barium enema

Colonoscopy

Gastroscopy

Serum chemistry levels

Question 6

When teaching a community group about measures to prevent colon cancer,


which instruction should the nurse include?

A Limit fat intake to 20% to 25% of your total daily calories.


B Include 15 to 20 grams of fiber into your daily diet.
C Get an annual rectal examination after age 35.
D Undergo sigmoidoscopy annually after age 50.
Question 7

Which of the following medications is most effective for treating the pain
associated with irritable bowel disease?

Acetaminophen

Opiates

Steroids

Stool softeners

Question 8

Which of the following associated disorders may the client with Crohns
disease exhibit?

Ankylosing spondylitis

Colon cancer

Malabsorption

Lactase deficiency

Question 9

During the assessment of a clients mouth, the nurse notes the absence of
saliva. The client is also complaining of pain near the area of the ear. The
client has been NPO for several days because of the insertion of a NG tube.

Based on these findings, the nurse suspects that the client is developing
which of the following mouth conditions?
A

Stomatitis

Oral candidiasis

Parotitis

Gingivitis

Question 10

Which of the following diagnostic tests should be performed annually over


age 50 to screen for colon cancer?

Abdominal CT scan

Abdominal x-ray

Colonoscopy

Fecal occult blood test

Question 11

Crohns disease can be described as a chronic relapsing disease. Which of the


following areas in the GI system may be involved with this disease?

A The entire length of the large colon


B Only the sigmoid area
C The entire large colon through the layers of mucosa and submucosa
D The small intestine and colon; affecting the entire thickness of the bowel
Question 12

The nurse evaluates the clients stoma during the initial post-op period.
Which of the following observations should be reported immediately to the
physician?

The stoma is slightly edematous

The stoma is dark red to purple

The stoma oozes a small amount of blood

The stoma does not expel stool

Question 13

Which of the following complications of gastric resection should the nurse


teach the client to watch for?

Constipation

Dumping syndrome

Gastric spasm

Intestinal spasms

Question 14

Surgical management of ulcerative colitis may be performed to treat which of


the following complications?

Gastritis

Bowel herniation

Bowel outpouching

Bowel perforation

Question 15

Which of the following associated disorders may a client with ulcerative colitis
exhibit?

Gallstones

Hydronephrosis

Nephrolithiasis

Toxic megacolon

Question 16

Proton pump inhibitors:

Gastric ulcer formation

GERD

Achlorhydria

Diverticulosis

Question 17

The client with a new colostomy is concerned about the odor from the stool in
the ostomy drainage bag. The nurse teaches the client to include which of the
following foods in the diet to reduce odor?

Yogurt

Broccoli

Cucumbers

Eggs

Question 18

The client with Crohns disease has a nursing diagnosis of acute pain. The
nurse would teach the client to avoid which of the following in managing this
problem?

Lying supine with the legs straight

Massaging the abdomen

Using antispasmodic medication

Using relaxation techniques

Question 19

The nurse would question an order for which type of antacid in patients with
chronic renal failure?

Aluminum-containing antacids

Calcium-containing antacids

Magnesium-containing antacids

All of the above.

Question 20

In a client with diarrhea, which outcome indicates that fluid resuscitation is


successful?

A The client passes formed stools at regular intervals


B The client reports a decrease in stool frequency and liquidity
C The client exhibits firm skin turgor

D The client no longer experiences perianal burning.


Question 21

Which of the following laboratory results would be expected in a client with


peritonitis?

Partial thromboplastin time above 100 seconds

Hemoglobin level below 10 mg/dL

Potassium level above 5.5 mEq/L

White blood cell count above 15,000

Question 22

Which of the following definitions best describes diverticulosis?

A An inflamed outpouching of the intestine


B A noninflamed outpouching of the intestine
C The partial impairment of the forward flow of intestinal contents
D An abnormal protrusion of an organ through the structure that usually holds it.
Question 23

Which of the following diets is most commonly associated with colon cancer?

Low-fiber, high fat

Low-fat, high-fiber

Low-protein, high-carbohydrate

Low carbohydrate, high protein

Question 24

The nurse instructs the ileostomy client to do which of the following as a part
of essential care of the stoma?

Cleanse the peristomal skin meticulously

Take in high-fiber foods such as nuts

Massage the area below the stoma

Limit fluid intake to prevent diarrhea.

Question 25

The nurse would monitor for which of the following adverse reactions to
aluminum-containing antacids such as aluminum hydroxide (Amphojel)?

Diarrhea

Constipation

GI upset

Fluid retention

Question 26

If a client had irritable bowel syndrome, which of the following diagnostic


tests would determine if the diagnosis is Crohns disease or ulcerative colitis?

Abdominal computed tomography (CT) scan

Abdominal x-ray

Barium swallow

Colonoscopy with biopsy

Question 27

Digoxin preparations and absorbents should not be given simultaneously. As


a nurse, you are aware that if these agents are given simultaneously, which
of the following will occur?

Increased absorption of digoxin

Decreased absorption of digoxin

Increased absorption of the absorbent

Decreased absorption of the absorbent

Question 28

Which of the following interventions should be included in the medical


management of Crohns disease?

Increasing oral intake of fiber

Administering laxatives

Using long-term steroid therapy

Increasing physical activity

Question 29

When used with hyperacidic disorders of the stomach, antacids are given to
elevate the gastric pH to:

2.0

4.0

6.0

8.0

Question 30

The nurse is preparing a discharge teaching plan for the client who had an
umbilical hernia repair. Which of the following would the nurse include in the
plan?

Restricting pain medication

Maintaining bedrest

Avoiding coughing

Irrigating the drain

Question 31

The nurse is teaching the client how to perform a colostomy irrigation. To


enhance the effectiveness of the irrigation and fecal returns, what measure
should the nurse instruct the client to do?

Increase fluid intake

Reduce the amount of irrigation solution

Perform the irrigation in the evening

Place heat on the abdomen

Question 32

Which of the following nursing diagnoses is appropriate for a patient receiving


famotidine (Pepcid)?

Increased risk for infection due to immunosuppression

Potential risk for bleeding related to thrombocytopenia

Alteration in urinary elimination related to retention

D Alteration in tissue perfusion related to hypertension


Question 33

The client with a colostomy has an order for irrigation of the colostomy. The
nurse used which solution for irrigation?

Distilled water

Tap water

Sterile water

Lactated Ringers

Question 34

The nurse is doing pre-op teaching with the client who is about to undergo
creation of a Kock pouch. The nurse interprets that the client has the best
understanding of the nature of the surgery if the client makes which of the
following statements?

A I will need to drain the pouch regularly with a catheter.


B I will need to wear a drainage bag for the rest of my life.
C The drainage from this type of ostomy will be formed.
D I will be able to pass stool from my rectum eventually.
Question 35

Which of the following symptoms is a client with colon cancer most likely to
exhibit?

A change in appetite

A change in bowel habit

An increase in body weight

An increase in body temperature

Question 36

A client with gastric cancer can expect to have surgery for resection. Which of
the following should be the nursing management priority for the preoperative
client with gastric cancer?

Discharge planning

Correction of nutritional deficits

Prevention of DVT

Instruction regarding radiation treatment

Question 37

The mechanism of action of diphenoxylate (Lotomil) is:

An increase in intestinal excretion of water

An increase in intestinal motility

A decrease in peristalsis in the intestinal wall

A decrease in the reabsorption of water in the bowel

Question 38

Which of the following tests should be administered to a client suspected of


having diverticulosis?

Abdominal ultrasound

Barium enema

Barium swallow

Gastroscopy

Question 39

When teaching an elderly client how to prevent constipation, which of the


following instructions should the nurse include?

A Drink 6 glasses of fluid each day.


B Avoid grain products and nuts.
C Add at least 4 grams of bran to your cereal each morning.
D Be sure to get regular exercise.
Question 40

The nurse would monitor a patient using sodium bicarbonate to treat gastric
hyperacidity for signs and symptoms of:

Metabolic alkalosis

Metabolic acidosis

Hyperkalemia

Hypercalcemia

Question 41

Histamine2-receptor antagonists:

A Compete with histamine for binding sites on the parietal cells


B Irreversibly bind to H+/K+ATPase
C Cause a decrease in stomach pH
D Decrease signs and symptoms of allergies related to histamine release
Question 42

Which of the following types of diets is implicated in the development of


diverticulosis?

Low-fiber diet

High-fiber diet

High-protein diet

Low-carbohydrate diet

Question 43

Which of the following mechanisms can facilitate the development of


diverticulosis into diverticulitis?

Treating constipation with chronic laxative use, leading to dependence on


laxatives

Chronic constipation causing an obstruction, reducing forward flow of intestinal


contents

C Herniation of the intestinal mucosa, rupturing the wall of the intestine


D Undigested food blocking the diverticulum, predisposing the area to bacterial

invasion.
Question 44

The client has been admitted with a diagnosis of acute pancreatitis. The
nurse would assess this client for pain that is:

A Severe and unrelenting, located in the epigastric area and radiating to the back.
B

Severe and unrelenting, located in the left lower quadrant and radiating to the
groin.

C Burning and aching, located in the epigastric area and radiating to the umbilicus.
D Burning and aching, located in the left lower quadrant and radiating to the hip.
Question 45

To avoid fecal impaction, psyllium (Metamucil) should be administered with at


least how many ounces of fluid?

10

Question 46

Which of the following definitions best describes gastritis?

Erosion of the gastric mucosa

Inflammation of a diverticulum

Inflammation of the gastric mucosa

Reflux of stomach acid into the esophagus

Question 47

Which area of the alimentary canal is the most common location for Crohns
disease?

Ascending colon

Descending colon

Sigmoid colon

Terminal ileum

Question 48

The nurse is reviewing the record of a client with Crohns disease. Which of
the following stool characteristics would the nurse expect to note
documented on the clients record?

Chronic constipation

Diarrhea

Constipation alternating with diarrhea

Stool constantly oozing from the rectum

Question 49

The nurse is caring for a hospitalized client with a diagnosis of ulcerative


colitis. Which finding, if noted on assessment of the client, would the nurse
report to the physician?

Bloody diarrhea

Hypotension

A hemoglobin of 12 mg/dL

Rebound tenderness

Question 50

The client who has undergone creation of a colostomy has a nursing


diagnosis of Disturbed body image. The nurse would evaluate that the client
is making the most significant progress toward identified goals if the client:

Watches the nurse empty the colostomy bag

Looks at the ostomy site

Reads the ostomy product literature

Practices cutting the ostomy appliance

Question 51

When planning care for a client with ulcerative colitis who is experiencing
symptoms, which client care activities can the nurse appropriately delegate
to a unlicensed assistant? Select all that apply.

A Assessing the clients bowel sounds

B Providing skin care following bowel movements


C Evaluating the clients response to antidiarrheal medications
D Maintaining intake and output records
E Obtaining the clients weight
Question 52

Sucralfate (Carafate) achieves a therapeutic effect by:

Neutralizing gastric acid

Enhancing gastric absorption

Forming a protective barrier around gastric mucosa

Inhibiting gastric acid secretion

Question 53

Which of the following therapies is not included in the medical management


of a client with peritonitis?

Broad-spectrum antibiotics

Electrolyte replacement

I.V. fluids

Regular diet

Question 54

A 30-year old client experiences weight loss, abdominal distention, crampy


abdominal pain, and intermittent diarrhea after birth of her 2nd child.
Diagnostic tests reveal gluten-induced enteropathy. Which foods must she
eliminate from her diet permanently?

Milk and dairy products

Protein-containing foods

Cereal grains (except rice and corn)

Carbohydrates

Question 55

A client has a percutaneous endoscopic gastrostomy tube inserted for tube


feedings. Before starting a continuous feeding, the nurse should place the
client in which position?

Semi-Fowlers

Supine

Reverse Trendelenburg

High Fowlers

Question 56

A client has just had surgery for colon cancer. Which of the following
disorders might the client develop?

Peritonitis

Diverticulosis

Partial bowel obstruction

Complete bowel obstruction

Question 57

In a client with Crohns disease, which of the following symptoms should not
be a direct result from antibiotic therapy?

Decrease in bleeding

Decrease in temperature

Decrease in body weight

Decrease in the number of stools

Question 58

Care for the postoperative client after gastric resection should focus on which
of the following problems?

Body image

Nutritional needs

Skin care

Spiritual needs

Question 59

A patient unable to tolerate oral medications may be prescribed which of the


following proton pump inhibitors to be administered intravenously?

lansoprazole (Prevacid)

omeprazole (Prilosec)

pantoprazole (Protonix)

esomeprazole (Nexium)

Question 60

After a right hemicolectomy for treatment of colon cancer, a 57-year old


client is reluctant to turn while on bed rest. Which action by the nurse would
be appropriate?

A Asking a coworker to help turn the client


B Explaining to the client why turning is important
C Allowing the client to turn when hes ready to do so
D Telling the client that the physicians order states he must turn every 2 hours
Question 61

A client with which of the following conditions may be likely to develop rectal
cancer?

Adenomatous polyps

Diverticulitis

Hemorrhoids

Peptic ulcer disease

Question 62

The nurse would teach patients that antacids are effective in treatment of
hyperacidity because they:

Neutralize gastric acid

Decrease stomach motility

Decrease gastric pH

Decrease duodenal pH

Question 63

When administering sucralfate (Carafate) to a patient with a nasogastric


tube, it is important to:

A Crush the tablet into a fine powder before mixing with water
B Administer with a bolus tube feeding
C Allow the tablet to dissolve in water before administering
D Administer with an antacid for maximum benefit
Question 64

A client presents to the emergency room, reporting that he has been


vomiting every 30 to 40 minutes for the past 8 hours. Frequent vomiting puts
him at risk for which of the following?

Metabolic acidosis with hyperkalemia

Metabolic acidosis with hypokalemia

Metabolic alkalosis with hyperkalemia

Metabolic alkalosis with hypokalemia

Question 65

A nurse is monitoring a client admitted to the hospital with a diagnosis of


appendicitis. The client is scheduled for surgery in 2 hours. The client begins
to complain of increased abdominal pain and begins to vomit. On assessment
the nurse notes that the abdomen is distended and the bowel sounds are
diminished. Which of the following is the most appropriate nursing
intervention?

A Administer dilaudid
B Notify the physician
C Call and ask the operating room team to perform the surgery as soon as possible
D

Reposition the client and apply a heating pad on a warm setting to the clients
abdomen

Question 66

Colon cancer is most closely associated with which of the following


conditions?

Appendicitis

Hemorrhoids

Hiatal hernia

Ulcerative colitis

Question 67

A client who has ulcerative colitis has persistent diarrhea. He is thin and has
lost 12 pounds since the exacerbation of his ulcerative colitis. The nurse
should anticipate that the physician will order which of the following
treatment approaches to help the client meet his nutritional needs?

Initiate continuous enteral feedings

Encourage a high protein, high-calorie diet

Implement total parenteral nutrition

Provide six small meals a day

Question 68

Radiation therapy is used to treat colon cancer before surgery for which of
the following reasons?

Reducing the size of the tumor

Eliminating the malignant cells

Curing the cancer

Helping the bowel heal after surgery

Question 69

Which of the following symptoms is associated with ulcerative colitis?

Dumping syndrome

Rectal bleeding

Soft stools

Fistulas

Question 70

One of your patients is receiving digitalis orally and is also to receive an


antacid at the same time. Your most appropriate action, based on the
pharmacokinetics of antacids, is to:

A Delay the digitalis for 1 to 2 hours until the antacid is absorbed


B Give the antacid at least 2 to 4 hours before administering the digitalis
C Administer both medications as ordered and document in nurses notes
D

Contact the physician regarding the drug interaction and request a change in the
time of dosing of the drugs.

Question 71

The nurse is assessing for stoma prolapse in a client with a colostomy. The
nurse would observe which of the following if stoma prolapse occurred?

Sunken and hidden stoma

Dark- and bluish-colored stoma

Narrowed and flattened stoma

Protruding stoma

Question 72

A client with ulcerative colitis has an order to begin salicylate medication to


reduce inflammation. The nurse instructs the client to take the medication:

30 minutes before meals

On an empty stomach

After meals

On arising

Question 73

Which of the following conditions is most likely to directly cause peritonitis?

Cholelithiasis

Gastritis

Perforated ulcer

Incarcerated hernia

Question 74

Side effects of loperamide (Imodium) include all of the following except?

Diarrhea

Epigastric pain

Dry mouth

Anorexia

Question 75

A clients ulcerative colitis symptoms have been present for longer than 1
week. The nurse recognizes that the client should be assessed carefully for
signs of which of the following complications?

Heart failure

DVT

Hypokalemia

Hypocalcemia

Question 76

The nurse is reviewing the physicians orders written for a client admitted
with acute pancreatitis. Which physician order would the nurse question if
noted on the clients chart?

NPO status

Insert a nasogastric tube

An anticholinergic medication

Morphine for pain

Question 77

During the first few days of recovery from ostomy surgery for ulcerative
colitis, which of the following aspects should be the first priority of client
care?

Body image

Ostomy care

Sexual concerns

Skin care

Question 78

Which of the following symptoms may be exhibited by a client with Crohns


disease?

Bloody diarrhea

Narrow stools

N/V

Steatorrhea

Question 79

Which of the following factors is believed to cause ulcerative colitis?

Acidic diet

Altered immunity

Chronic constipation

Emotional stress

Question 80

The client has just had surgery to create an ileostomy. The nurse assesses the
client in the immediate post-op period for which of the following most
frequent complications of this type of surgery?

Intestinal obstruction

Fluid and electrolyte imbalance

Malabsorption of fat

Folate deficiency

Question 81

Which of the following nursing interventions should be implemented to


manage a client with appendicitis?
Assessing for pain

Encouraging oral intake of clear fluids

Providing discharge teaching

Assessing for symptoms of peritonitis

Question 82

A client with rectal cancer may exhibit which of the following symptoms?

Abdominal fullness

Gastric fullness

Rectal bleeding

Right upper quadrant pain

Question 83

Bismuth subsalicylate (Pepto-Bismol), as an absorbent, has which of the


following mechanisms of action?

Decreased GI motility

Decreased gastric secretions

Increased fluid absorption

Binding to diarrhea-causing bacteria for excretion

Question 84

Which of the following complications is thought to be the most common


cause of appendicitis?

A fecalith

Bowel kinking

Internal bowel occlusion

Abdominal bowel swelling

Question 85

A client with irritable bowel syndrome is being prepared for discharge. Which
of the following meal plans should the nurse give the client?

Low fiber, low-fat

High fiber, low-fat

Low fiber, high-fat

High-fiber, high-fat

Question 86

Which of the following terms best describes the pain associated with
appendicitis?

Aching

Fleeting

Intermittent

Steady

Question 87

Fistulas are most common with which of the following bowel disorders?

Crohns disease

Diverticulitis

Diverticulosis

Ulcerative colitis

Question 88

Which of the following aspects is the priority focus of nursing management


for a client with peritonitis?

Fluid and electrolyte balance

Gastric irrigation

Pain management

Psychosocial issues

Question 89

The nurse has given instructions to the client with an ileostomy about foods
to eat to thicken the stool. The nurse determines that the client needs further
instructions if the client stated to eat which of the following foods to make the
stools less watery?

Pasta

Boiled rice

Bran

Low-fat cheese

Question 90

Five days after undergoing surgery, a client develops a small-bowel


obstruction. A Miller-Abbott tube is inserted for bowel decompression. Which
nursing diagnosis takes priority?

Imbalanced nutrition: Less than body requirements

Acute pain

Deficient fluid volume

Excess fluid volume

Question 91

Which of the following symptoms would a client in the early stages of


peritonitis exhibit?

Abdominal distention

Abdominal pain and rigidity

Hyperactive bowel sounds

Right upper quadrant pain

Question 92

The nurse is doing an admission assessment on a client with a history of


duodenal ulcer. To determine whether the problem is currently active, the
nurse would assess the client for which of the following most frequent
symptom(s) of duodenal ulcer?

Pain that is relieved by food intake

Pain that radiated down the right arm

N/V

Weight loss

Question 93

Which of the following substances is most likely to cause gastritis?

Milk

Bicarbonate of soda, or baking soda

Enteric coated aspirin

Nonsteroidal anti-inflammatory drugs

Question 94

An enema is prescribed for a client with suspected appendicitis. Which of the


following actions should the nurse take?

Prepare 750 ml of irrigating solution warmed to 100*F

Question the physician about the order

Provide privacy and explain the procedure to the client

D Assist the client to left lateral Sims position


Question 95

Which of the following symptoms indicated diverticulosis?

No symptoms exist

Change in bowel habits

Anorexia with low-grade fever

Episodic, dull, or steady midabdominal pain

Question 96

The nurse is performing a colostomy irrigation on a client. During the


irrigation, a client begins to complain of abdominal cramps. Which of the
following is the most appropriate nursing action?

Notify the physician

Increase the height of the irrigation

Stop the irrigation temporarily

Medicate with dilaudid and resume the irrigation

Question 97

Which of the following areas is the most common site of fistulas in clients
with Crohns disease?

Anorectal

Ileum

Rectovaginal

Transverse colon

Question 98

Which of the following treatments is used for rectal cancer but not for colon
cancer?

Chemotherapy

Colonoscopy

Radiation

Surgical resection

Question 99

The client being seen in a physicians office has just been scheduled for a
barium swallow the next day. The nurse writes down which of the following
instructions for the client to follow before the test?

Fast for 8 hours before the test

Eat a regular supper and breakfast

Continue to take all oral medications as scheduled

D Monitor own bowel movement pattern for constipatio


Question 100

A client with gastric cancer may exhibit which of the following symptoms?

Abdominal cramping

Constant hunger

Feeling of fullness

Weight gain

Once you are finished, click the button below. Any items you have not
completed will be marked incorrect.

Practice Mode: This is an interactive version of the Text Mode. All questions are given in
a single page and correct answers, rationales or explanations (if any) are immediately
shown after you have selected an answer.

NCLEX Exam: Gastrointestinal Disorders 2 (100 Items)

Question 1

Which of the following complications is thought to be the most common


cause of appendicitis?

A fecalith

Bowel kinking

Internal bowel occlusion

Abdominal bowel swelling

Question 2

Which of the following terms best describes the pain associated with
appendicitis?

Aching

Fleeting

Intermittent

Steady

Question 3

Which of the following nursing interventions should be implemented to


manage a client with appendicitis?

Assessing for pain

Encouraging oral intake of clear fluids

Providing discharge teaching

Assessing for symptoms of peritonitis

Question 4

Which of the following definitions best describes gastritis?

Erosion of the gastric mucosa

Inflammation of a diverticulum

Inflammation of the gastric mucosa

Reflux of stomach acid into the esophagus

Question 5

Which of the following substances is most likely to cause gastritis?

Milk

Bicarbonate of soda, or baking soda

Enteric coated aspirin

Nonsteroidal anti-inflammatory drugs

Question 6

Which of the following definitions best describes diverticulosis?

A An inflamed outpouching of the intestine


B A noninflamed outpouching of the intestine
C The partial impairment of the forward flow of intestinal contents
D An abnormal protrusion of an organ through the structure that usually holds it.
Question 7

Which of the following types of diets is implicated in the development of


diverticulosis?

Low-fiber diet

High-fiber diet

High-protein diet

Low-carbohydrate diet

Question 8

Which of the following mechanisms can facilitate the development of


diverticulosis into diverticulitis?

A Treating constipation with chronic laxative use, leading to dependence on

laxatives
B

Chronic constipation causing an obstruction, reducing forward flow of intestinal


contents

C Herniation of the intestinal mucosa, rupturing the wall of the intestine


D

Undigested food blocking the diverticulum, predisposing the area to bacterial


invasion.

Question 9

Which of the following symptoms indicated diverticulosis?

No symptoms exist

Change in bowel habits

Anorexia with low-grade fever

Episodic, dull, or steady midabdominal pain

Question 10

Which of the following tests should be administered to a client suspected of


having diverticulosis?

Abdominal ultrasound

Barium enema

Barium swallow

Gastroscopy

Question 11

Medical management of the client with diverticulitis should include which of


the following treatments?

Reduced fluid intake

Increased fiber in diet

Administration of antibiotics

Exercises to increase intra-abdominal pressure

Question 12

Crohns disease can be described as a chronic relapsing disease. Which of the


following areas in the GI system may be involved with this disease?

A The entire length of the large colon


B Only the sigmoid area
C The entire large colon through the layers of mucosa and submucosa
D The small intestine and colon; affecting the entire thickness of the bowel
Question 13

Which area of the alimentary canal is the most common location for Crohns
disease?

Ascending colon

Descending colon

Sigmoid colon

Terminal ileum

Question 14

Which of the following factors is believed to be linked to Crohns disease?

Constipation

Diet

Hereditary

Lack of exercise

Question 15

Which of the following factors is believed to cause ulcerative colitis?

Acidic diet

Altered immunity

Chronic constipation

Emotional stress

Question 16

Fistulas are most common with which of the following bowel disorders?

Crohns disease

Diverticulitis

Diverticulosis

Ulcerative colitis

Question 17

Which of the following areas is the most common site of fistulas in clients
with Crohns disease?

Anorectal

Ileum

Rectovaginal

Transverse colon

Question 18

Which of the following associated disorders may a client with ulcerative colitis
exhibit?

Gallstones

Hydronephrosis

Nephrolithiasis

Toxic megacolon

Question 19

Which of the following associated disorders may the client with Crohns
disease exhibit?

Ankylosing spondylitis

Colon cancer

Malabsorption

Lactase deficiency

Question 20

Which of the following symptoms may be exhibited by a client with Crohns


disease?
Bloody diarrhea

Narrow stools

N/V

Steatorrhea

Question 21

Which of the following symptoms is associated with ulcerative colitis?

Dumping syndrome

Rectal bleeding

Soft stools

Fistulas

Question 22

If a client had irritable bowel syndrome, which of the following diagnostic


tests would determine if the diagnosis is Crohns disease or ulcerative colitis?

Abdominal computed tomography (CT) scan

Abdominal x-ray

Barium swallow

Colonoscopy with biopsy

Question 23

Which of the following interventions should be included in the medical


management of Crohns disease?

Increasing oral intake of fiber

Administering laxatives

Using long-term steroid therapy

Increasing physical activity

Question 24

In a client with Crohns disease, which of the following symptoms should not
be a direct result from antibiotic therapy?

Decrease in bleeding

Decrease in temperature

Decrease in body weight

Decrease in the number of stools

Question 25

Surgical management of ulcerative colitis may be performed to treat which of


the following complications?

Gastritis

Bowel herniation

Bowel outpouching

Bowel perforation

Question 26

Which of the following medications is most effective for treating the pain
associated with irritable bowel disease?

Acetaminophen

Opiates

Steroids

Stool softeners

Question 27

During the first few days of recovery from ostomy surgery for ulcerative
colitis, which of the following aspects should be the first priority of client
care?

Body image

Ostomy care

Sexual concerns

Skin care

Question 28

Colon cancer is most closely associated with which of the following


conditions?

Appendicitis

Hemorrhoids

Hiatal hernia

Ulcerative colitis

Question 29

Which of the following diets is most commonly associated with colon cancer?

Low-fiber, high fat

Low-fat, high-fiber

Low-protein, high-carbohydrate

Low carbohydrate, high protein

Question 30

Which of the following diagnostic tests should be performed annually over


age 50 to screen for colon cancer?

Abdominal CT scan

Abdominal x-ray

Colonoscopy

Fecal occult blood test

Question 31

Radiation therapy is used to treat colon cancer before surgery for which of
the following reasons?

Reducing the size of the tumor

Eliminating the malignant cells

Curing the cancer

Helping the bowel heal after surgery

Question 32

Which of the following symptoms is a client with colon cancer most likely to
exhibit?
A change in appetite

A change in bowel habit

An increase in body weight

An increase in body temperature

Question 33

A client has just had surgery for colon cancer. Which of the following
disorders might the client develop?

Peritonitis

Diverticulosis

Partial bowel obstruction

Complete bowel obstruction

Question 34

A client with gastric cancer may exhibit which of the following symptoms?

Abdominal cramping

Constant hunger

Feeling of fullness

Weight gain

Question 35

Which of the following diagnostic tests may be performed to determine if a


client has gastric cancer?

Barium enema

Colonoscopy

Gastroscopy

Serum chemistry levels

Question 36

A client with gastric cancer can expect to have surgery for resection. Which of
the following should be the nursing management priority for the preoperative
client with gastric cancer?

Discharge planning

Correction of nutritional deficits

Prevention of DVT

Instruction regarding radiation treatment

Question 37

Care for the postoperative client after gastric resection should focus on which
of the following problems?

Body image

Nutritional needs

Skin care

Spiritual needs

Question 38

Which of the following complications of gastric resection should the nurse


teach the client to watch for?

Constipation

Dumping syndrome

Gastric spasm

Intestinal spasms

Question 39

A client with rectal cancer may exhibit which of the following symptoms?

Abdominal fullness

Gastric fullness

Rectal bleeding

Right upper quadrant pain

Question 40

A client with which of the following conditions may be likely to develop rectal
cancer?
Adenomatous polyps

Diverticulitis

Hemorrhoids

Peptic ulcer disease

Question 41

Which of the following treatments is used for rectal cancer but not for colon
cancer?

Chemotherapy

Colonoscopy

Radiation

Surgical resection

Question 42

Which of the following conditions is most likely to directly cause peritonitis?

Cholelithiasis

Gastritis

Perforated ulcer

Incarcerated hernia

Question 43

Which of the following symptoms would a client in the early stages of


peritonitis exhibit?

Abdominal distention

Abdominal pain and rigidity

Hyperactive bowel sounds

Right upper quadrant pain

Question 44

Which of the following laboratory results would be expected in a client with


peritonitis?

Partial thromboplastin time above 100 seconds

Hemoglobin level below 10 mg/dL

Potassium level above 5.5 mEq/L

White blood cell count above 15,000

Question 45

Which of the following therapies is not included in the medical management


of a client with peritonitis?

Broad-spectrum antibiotics

Electrolyte replacement

I.V. fluids

Regular diet

Question 46

Which of the following aspects is the priority focus of nursing management


for a client with peritonitis?

Fluid and electrolyte balance

Gastric irrigation

Pain management

Psychosocial issues

Question 47

A client with irritable bowel syndrome is being prepared for discharge. Which
of the following meal plans should the nurse give the client?

Low fiber, low-fat

High fiber, low-fat

Low fiber, high-fat

High-fiber, high-fat

Question 48

A client presents to the emergency room, reporting that he has been


vomiting every 30 to 40 minutes for the past 8 hours. Frequent vomiting puts
him at risk for which of the following?

Metabolic acidosis with hyperkalemia

Metabolic acidosis with hypokalemia

Metabolic alkalosis with hyperkalemia

Metabolic alkalosis with hypokalemia

Question 49

Five days after undergoing surgery, a client develops a small-bowel


obstruction. A Miller-Abbott tube is inserted for bowel decompression. Which
nursing diagnosis takes priority?

Imbalanced nutrition: Less than body requirements

Acute pain

Deficient fluid volume

Excess fluid volume

Question 50

When teaching an elderly client how to prevent constipation, which of the


following instructions should the nurse include?

A Drink 6 glasses of fluid each day.


B Avoid grain products and nuts.
C Add at least 4 grams of bran to your cereal each morning.
D Be sure to get regular exercise.
Question 51

In a client with diarrhea, which outcome indicates that fluid resuscitation is


successful?

A The client passes formed stools at regular intervals


B The client reports a decrease in stool frequency and liquidity
C The client exhibits firm skin turgor
D The client no longer experiences perianal burning.
Question 52

When teaching a community group about measures to prevent colon cancer,


which instruction should the nurse include?

A Limit fat intake to 20% to 25% of your total daily calories.


B Include 15 to 20 grams of fiber into your daily diet.
C Get an annual rectal examination after age 35.
D Undergo sigmoidoscopy annually after age 50.
Question 53

A 30-year old client experiences weight loss, abdominal distention, crampy


abdominal pain, and intermittent diarrhea after birth of her 2nd child.
Diagnostic tests reveal gluten-induced enteropathy. Which foods must she
eliminate from her diet permanently?

Milk and dairy products

Protein-containing foods

Cereal grains (except rice and corn)

Carbohydrates

Question 54

After a right hemicolectomy for treatment of colon cancer, a 57-year old


client is reluctant to turn while on bed rest. Which action by the nurse would
be appropriate?

A Asking a coworker to help turn the client


B Explaining to the client why turning is important
C Allowing the client to turn when hes ready to do so
D Telling the client that the physicians order states he must turn every 2 hours
Question 55

A client has a percutaneous endoscopic gastrostomy tube inserted for tube


feedings. Before starting a continuous feeding, the nurse should place the
client in which position?

Semi-Fowlers

Supine

Reverse Trendelenburg

High Fowlers

Question 56

An enema is prescribed for a client with suspected appendicitis. Which of the


following actions should the nurse take?

Prepare 750 ml of irrigating solution warmed to 100*F

Question the physician about the order

Provide privacy and explain the procedure to the client

D Assist the client to left lateral Sims position


Question 57

The client being seen in a physicians office has just been scheduled for a
barium swallow the next day. The nurse writes down which of the following
instructions for the client to follow before the test?

Fast for 8 hours before the test

Eat a regular supper and breakfast

Continue to take all oral medications as scheduled

D Monitor own bowel movement pattern for constipatio


Question 58

The nurse is monitoring a client for the early signs of dumping syndrome.
Which symptom indicates this occurrence?

Abdominal cramping and pain

Bradycardia and indigestion

Sweating and pallor

Double vision and chest pain

Question 59

The nurse is preparing a discharge teaching plan for the client who had an
umbilical hernia repair. Which of the following would the nurse include in the
plan?

Restricting pain medication

Maintaining bedrest

Avoiding coughing

Irrigating the drain

Question 60

The nurse is caring for a hospitalized client with a diagnosis of ulcerative


colitis. Which finding, if noted on assessment of the client, would the nurse
report to the physician?

Bloody diarrhea

Hypotension

A hemoglobin of 12 mg/dL

Rebound tenderness

Question 61

The nurse is reviewing the record of a client with Crohns disease. Which of
the following stool characteristics would the nurse expect to note
documented on the clients record?

Chronic constipation

Diarrhea

Constipation alternating with diarrhea

Stool constantly oozing from the rectum

Question 62

The nurse is performing a colostomy irrigation on a client. During the


irrigation, a client begins to complain of abdominal cramps. Which of the
following is the most appropriate nursing action?

Notify the physician

Increase the height of the irrigation

Stop the irrigation temporarily

Medicate with dilaudid and resume the irrigation

Question 63

The nurse is teaching the client how to perform a colostomy irrigation. To


enhance the effectiveness of the irrigation and fecal returns, what measure
should the nurse instruct the client to do?

Increase fluid intake

Reduce the amount of irrigation solution

Perform the irrigation in the evening

Place heat on the abdomen

Question 64

The nurse is reviewing the physicians orders written for a client admitted
with acute pancreatitis. Which physician order would the nurse question if
noted on the clients chart?

NPO status

Insert a nasogastric tube

An anticholinergic medication

Morphine for pain

Question 65

The nurse is doing an admission assessment on a client with a history of


duodenal ulcer. To determine whether the problem is currently active, the
nurse would assess the client for which of the following most frequent
symptom(s) of duodenal ulcer?

Pain that is relieved by food intake

Pain that radiated down the right arm

N/V

Weight loss

Question 66

The nurse instructs the ileostomy client to do which of the following as a part
of essential care of the stoma?

Cleanse the peristomal skin meticulously

Take in high-fiber foods such as nuts

Massage the area below the stoma

Limit fluid intake to prevent diarrhea.

Question 67

The client who has undergone creation of a colostomy has a nursing


diagnosis of Disturbed body image. The nurse would evaluate that the client
is making the most significant progress toward identified goals if the client:

Watches the nurse empty the colostomy bag

Looks at the ostomy site

Reads the ostomy product literature

Practices cutting the ostomy appliance

Question 68

The nurse is assessing for stoma prolapse in a client with a colostomy. The
nurse would observe which of the following if stoma prolapse occurred?

Sunken and hidden stoma

Dark- and bluish-colored stoma

Narrowed and flattened stoma

Protruding stoma

Question 69

The client with a new colostomy is concerned about the odor from the stool in
the ostomy drainage bag. The nurse teaches the client to include which of the
following foods in the diet to reduce odor?

Yogurt

Broccoli

Cucumbers

Eggs

Question 70

The nurse has given instructions to the client with an ileostomy about foods
to eat to thicken the stool. The nurse determines that the client needs further
instructions if the client stated to eat which of the following foods to make the
stools less watery?

Pasta

Boiled rice

Bran

Low-fat cheese

Question 71

The client has just had surgery to create an ileostomy. The nurse assesses the
client in the immediate post-op period for which of the following most
frequent complications of this type of surgery?

Intestinal obstruction

Fluid and electrolyte imbalance

Malabsorption of fat

Folate deficiency

Question 72

The nurse is doing pre-op teaching with the client who is about to undergo
creation of a Kock pouch. The nurse interprets that the client has the best
understanding of the nature of the surgery if the client makes which of the
following statements?

A I will need to drain the pouch regularly with a catheter.


B I will need to wear a drainage bag for the rest of my life.
C The drainage from this type of ostomy will be formed.
D I will be able to pass stool from my rectum eventually.
Question 73

The client with a colostomy has an order for irrigation of the colostomy. The
nurse used which solution for irrigation?

Distilled water

Tap water

Sterile water

Lactated Ringers

Question 74

A nurse is monitoring a client admitted to the hospital with a diagnosis of


appendicitis. The client is scheduled for surgery in 2 hours. The client begins
to complain of increased abdominal pain and begins to vomit. On assessment
the nurse notes that the abdomen is distended and the bowel sounds are
diminished. Which of the following is the most appropriate nursing
intervention?

A Administer dilaudid
B Notify the physician
C Call and ask the operating room team to perform the surgery as soon as possible
D

Reposition the client and apply a heating pad on a warm setting to the clients
abdomen

Question 75

The client has been admitted with a diagnosis of acute pancreatitis. The
nurse would assess this client for pain that is:

A Severe and unrelenting, located in the epigastric area and radiating to the back.
B

Severe and unrelenting, located in the left lower quadrant and radiating to the
groin.

C Burning and aching, located in the epigastric area and radiating to the umbilicus.
D Burning and aching, located in the left lower quadrant and radiating to the hip.
Question 76

The client with Crohns disease has a nursing diagnosis of acute pain. The
nurse would teach the client to avoid which of the following in managing this
problem?

Lying supine with the legs straight

Massaging the abdomen

Using antispasmodic medication

Using relaxation techniques

Question 77

A client with ulcerative colitis has an order to begin salicylate medication to


reduce inflammation. The nurse instructs the client to take the medication:
30 minutes before meals

On an empty stomach

After meals

On arising

Question 78

During the assessment of a clients mouth, the nurse notes the absence of
saliva. The client is also complaining of pain near the area of the ear. The
client has been NPO for several days because of the insertion of a NG tube.
Based on these findings, the nurse suspects that the client is developing
which of the following mouth conditions?

Stomatitis

Oral candidiasis

Parotitis

Gingivitis

Question 79

The nurse evaluates the clients stoma during the initial post-op period.
Which of the following observations should be reported immediately to the
physician?

The stoma is slightly edematous

The stoma is dark red to purple

The stoma oozes a small amount of blood

The stoma does not expel stool

Question 80

When planning care for a client with ulcerative colitis who is experiencing
symptoms, which client care activities can the nurse appropriately delegate
to a unlicensed assistant? Select all that apply.

A Assessing the clients bowel sounds


B Providing skin care following bowel movements
C Evaluating the clients response to antidiarrheal medications
D Maintaining intake and output records

E Obtaining the clients weight


Question 81

Which goal of the clients care should take priority during the first days of
hospitalization for an exacerbation of ulcerative colitis?

Promoting self-care and independence

Managing diarrhea

Maintaining adequate nutrition

Promoting rest and comfort

Question 82

A clients ulcerative colitis symptoms have been present for longer than 1
week. The nurse recognizes that the client should be assessed carefully for
signs of which of the following complications?

Heart failure

DVT

Hypokalemia

Hypocalcemia

Question 83

A client who has ulcerative colitis has persistent diarrhea. He is thin and has
lost 12 pounds since the exacerbation of his ulcerative colitis. The nurse
should anticipate that the physician will order which of the following
treatment approaches to help the client meet his nutritional needs?

Initiate continuous enteral feedings

Encourage a high protein, high-calorie diet

Implement total parenteral nutrition

Provide six small meals a day

Question 84

Digoxin preparations and absorbents should not be given simultaneously. As


a nurse, you are aware that if these agents are given simultaneously, which
of the following will occur?

Increased absorption of digoxin

Decreased absorption of digoxin

Increased absorption of the absorbent

Decreased absorption of the absorbent

Question 85

When used with hyperacidic disorders of the stomach, antacids are given to
elevate the gastric pH to:

2.0

4.0

6.0

8.0

Question 86

One of your patients is receiving digitalis orally and is also to receive an


antacid at the same time. Your most appropriate action, based on the
pharmacokinetics of antacids, is to:

A Delay the digitalis for 1 to 2 hours until the antacid is absorbed


B Give the antacid at least 2 to 4 hours before administering the digitalis
C Administer both medications as ordered and document in nurses notes
D

Contact the physician regarding the drug interaction and request a change in the
time of dosing of the drugs.

Question 87

The nurse would teach patients that antacids are effective in treatment of
hyperacidity because they:

Neutralize gastric acid

Decrease stomach motility

Decrease gastric pH

Decrease duodenal pH

Question 88

The nurse would monitor for which of the following adverse reactions to
aluminum-containing antacids such as aluminum hydroxide (Amphojel)?

Diarrhea

Constipation

GI upset

Fluid retention

Question 89

The nurse would question an order for which type of antacid in patients with
chronic renal failure?

Aluminum-containing antacids

Calcium-containing antacids

Magnesium-containing antacids

All of the above.

Question 90

The nurse would monitor a patient using sodium bicarbonate to treat gastric
hyperacidity for signs and symptoms of:

Metabolic alkalosis

Metabolic acidosis

Hyperkalemia

Hypercalcemia

Question 91

Which of the following nursing diagnoses is appropriate for a patient receiving


famotidine (Pepcid)?

Increased risk for infection due to immunosuppression

Potential risk for bleeding related to thrombocytopenia

Alteration in urinary elimination related to retention

D Alteration in tissue perfusion related to hypertension


Question 92

Histamine2-receptor antagonists:

A Compete with histamine for binding sites on the parietal cells


B Irreversibly bind to H+/K+ATPase
C Cause a decrease in stomach pH
D Decrease signs and symptoms of allergies related to histamine release
Question 93

Proton pump inhibitors:

Gastric ulcer formation

GERD

Achlorhydria

Diverticulosis

Question 94

A patient unable to tolerate oral medications may be prescribed which of the


following proton pump inhibitors to be administered intravenously?

lansoprazole (Prevacid)

omeprazole (Prilosec)

pantoprazole (Protonix)

esomeprazole (Nexium)

Question 95

When administering sucralfate (Carafate) to a patient with a nasogastric


tube, it is important to:

A Crush the tablet into a fine powder before mixing with water
B Administer with a bolus tube feeding
C Allow the tablet to dissolve in water before administering
D Administer with an antacid for maximum benefit
Question 96

Sucralfate (Carafate) achieves a therapeutic effect by:

Neutralizing gastric acid

Enhancing gastric absorption

Forming a protective barrier around gastric mucosa

Inhibiting gastric acid secretion

Question 97

To avoid fecal impaction, psyllium (Metamucil) should be administered with at


least how many ounces of fluid?

10

Question 98

Bismuth subsalicylate (Pepto-Bismol), as an absorbent, has which of the


following mechanisms of action?

Decreased GI motility

Decreased gastric secretions

Increased fluid absorption

Binding to diarrhea-causing bacteria for excretion

Question 99

Side effects of loperamide (Imodium) include all of the following except?

Diarrhea

Epigastric pain

Dry mouth

Anorexia

Question 100

The mechanism of action of diphenoxylate (Lotomil) is:

An increase in intestinal excretion of water

An increase in intestinal motility

A decrease in peristalsis in the intestinal wall

A decrease in the reabsorption of water in the bowel

Once you are finished, click the button below. Any items you have not
completed will be marked incorrect.

In Text Mode: All questions and answers are given for reading and answering at your own pace.
You can also copy this exam and make a print out.
1. Which of the following complications is thought to be the most common cause of
appendicitis?
1. A fecalith
2. Bowel kinking
3. Internal bowel occlusion
4. Abdominal bowel swelling
2. Which of the following terms best describes the pain associated with appendicitis?
1. Aching
2. Fleeting
3. Intermittent
4. Steady
3. Which of the following nursing interventions should be implemented to manage a client
with appendicitis?
1. Assessing for pain
2. Encouraging oral intake of clear fluids
3. Providing discharge teaching
4. Assessing for symptoms of peritonitis
4. Which of the following definitions best describes gastritis?
1. Erosion of the gastric mucosa
2. Inflammation of a diverticulum
3. Inflammation of the gastric mucosa
4. Reflux of stomach acid into the esophagus

5. Which of the following substances is most likely to cause gastritis?


1. Milk
2. Bicarbonate of soda, or baking soda
3. Enteric coated aspirin
4. Nonsteriodal anti-imflammatory drugs
6. Which of the following definitions best describes diverticulosis?
1. An inflamed outpouching of the intestine
2. A noninflamed outpouching of the intestine
3. The partial impairment of the forward flow of intestinal contents
4. An abnormal protrusion of an organ through the structure that usually holds it.
7. Which of the following types of diets is implicated in the development of diverticulosis?
1. Low-fiber diet
2. High-fiber diet
3. High-protein diet
4. Low-carbohydrate diet
8. Which of the following mechanisms can facilitate the development of diverticulosis into
diverticulitis?
1. Treating constipation with chronic laxative use, leading to dependence on laxatives
2. Chronic constipation causing an obstruction, reducing forward flow of intestinal contents
3. Herniation of the intestinal mucosa, rupturing the wall of the intestine
4. Undigested food blocking the diverticulum, predisposing the area to bacterial invasion.
9. Which of the following symptoms indicated diverticulosis?
1. No symptoms exist
2. Change in bowel habits
3. Anorexia with low-grade fever
4. Episodic, dull, or steady midabdominal pain
10. Which of the following tests should be administered to a client suspected of having
diverticulosis?
1. Abdominal ultrasound
2. Barium enema

3. Barium swallow
4. Gastroscopy
11. Medical management of the client with diverticulitis should include which of the
following treatments?
1. Reduced fluid intake
2. Increased fiber in diet
3. Administration of antibiotics
4. Exercises to increase intra-abdominal pressure
12. Crohns disease can be described as a chronic relapsing disease. Which of the following
areas in the GI system may be involved with this disease?
1. The entire length of the large colon
2. Only the sigmoid area
3. The entire large colon through the layers of mucosa and submucosa
4. The small intestine and colon; affecting the entire thickness of the bowel
13. Which area of the alimentary canal is the most common location for Crohns disease?
1. Ascending colon
2. Descending colon
3. Sigmoid colon
4. Terminal ileum
14. Which of the following factors is believed to be linked to Crohns disease?
1. Constipation
2. Diet
3. Hereditary
4. Lack of exercise
15. Which of the following factors is believed to cause ulcerative colitis?
1. Acidic diet
2. Altered immunity
3. Chronic constipation
4. Emotional stress
16. Fistulas are most common with which of the following bowel disorders?

1. Crohns disease
2. Diverticulitis
3. Diverticulosis
4. Ulcerative colitis
17. Which of the following areas is the most common site of fistulas in clients with Crohns
disease?
1. Anorectal
2. Ileum
3. Rectovaginal
4. Transverse colon
18. Which of the following associated disorders may a client with ulcerative colitis exhibit?
1. Gallstones
2. Hydronephrosis
3. Nephrolithiasis
4. Toxic megacolon
19. Which of the following associated disorders may the client with Crohns disease
exhibit?
1. Ankylosing spondylitis
2. Colon cancer
3. Malabsorption
4. Lactase deficiency
20. Which of the following symptoms may be exhibited by a client with Crohns disease?
1. Bloody diarrhea
2. Narrow stools
3. N/V
4. Steatorrhea
21. Which of the following symptoms is associated with ulcerative colitis?
1. Dumping syndrome
2. Rectal bleeding
3. Soft stools
4. Fistulas

22. If a client had irritable bowel syndrome, which of the following diagnostic tests would
determine if the diagnosis is Crohns disease or ulcerative colitis?
1. Abdominal computed tomography (CT) scan
2. Abdominal x-ray
3. Barium swallow
4. Colonoscopy with biopsy
23. Which of the following interventions should be included in the medical management of
Crohns disease?
1. Increasing oral intake of fiber
2. Administering laxatives
3. Using long-term steroid therapy
4. Increasing physical activity
24. In a client with Crohns disease, which of the following symptoms should not be a direct
result from antibiotic therapy?
1. Decrease in bleeding
2. Decrease in temperature
3. Decrease in body weight
4. Decrease in the number of stools
25. Surgical management of ulcerative colitis may be performed to treat which of the
following complications?
1. Gastritis
2. Bowel herniation
3. Bowel outpouching
4. Bowel perforation
26. Which of the following medications is most effective for treating the pain associated
with irritable bowel disease?
1. Acetaminophen
2. Opiates
3. Steroids
4. Stool softeners
27. During the first few days of recovery from ostomy surgery for ulcerative colitis, which
of the following aspects should be the first priority of client care?

1. Body image
2. Ostomy care
3. Sexual concerns
4. Skin care
28. Colon cancer is most closely associated with which of the following conditions?
1. Appendicitis
2. Hemorrhoids
3. Hiatal hernia
4. Ulcerative colitis
29. Which of the following diets is most commonly associated with colon cancer?
1. Low-fiber, high fat
2. Low-fat, high-fiber
3. Low-protein, high-carbohydrate
4. Low carbohydrate, high protein
30. Which of the following diagnostic tests should be performed annually over age 50 to
screen for colon cancer?
1. Abdominal CT scan
2. Abdominal x-ray
3. Colonoscopy
4. Fecal occult blood test
31. Radiation therapy is used to treat colon cancer before surgery for which of the
following reasons?
1. Reducing the size of the tumor
2. Eliminating the malignant cells
3. Curing the cancer
4. Helping the bowel heal after surgery
32. Which of the following symptoms is a client with colon cancer most likely to exhibit?
1. A change in appetite
2. A change in bowel habits
3. An increase in body weight
4. An increase in body temperature

33. A client has just had surgery for colon cancer. Which of the following disorders might
the client develop?
1. Peritonitis
2. Diverticulosis
3. Partial bowel obstruction
4. Complete bowel obstruction
34. A client with gastric cancer may exhibit which of the following symptoms?
1. Abdominal cramping
2. Constant hunger
3. Feeling of fullness
4. Weight gain
35. Which of the following diagnostic tests may be performed to determine if a client has
gastric cancer?
1. Barium enema
2. Colonoscopy
3. Gastroscopy
4. Serum chemistry levels
36. A client with gastric cancer can expect to have surgery for resection. Which of the
following should be the nursing management priority for the preoperative client with
gastric cancer?
1. Discharge planning
2. Correction of nutritional deficits
3. Prevention of DVT
4. Instruction regarding radiation treatment
37. Care for the postoperative client after gastric resection should focus on which of the
following problems?
1. Body image
2. Nutritional needs
3. Skin care
4. Spiritual needs
38. Which of the following complications of gastric resection should the nurse teach the
client to watch for?

1. Constipation
2. Dumping syndrome
3. Gastric spasm
4. Intestinal spasms
39. A client with rectal cancer may exhibit which of the following symptoms?
1. Abdominal fullness
2. Gastric fullness
3. Rectal bleeding
4. Right upper quadrant pain
40. A client with which of the following conditions may be likely to develop rectal cancer?
1. Adenomatous polyps
2. Diverticulitis
3. Hemorrhoids
4. Peptic ulcer disease
41. Which of the following treatments is used for rectal cancer but not for colon cancer?
1. Chemotherapy
2. Colonoscopy
3. Radiation
4. Surgical resection
42. Which of the following conditions is most likely to directly cause peritonitis?
1. Cholelithiasis
2. Gastritis
3. Perforated ulcer
4. Incarcerated hernia
43. Which of the following symptoms would a client in the early stages of peritonitis
exhibit?
1. Abdominal distention
2. Abdominal pain and rigidity
3. Hyperactive bowel sounds
4. Right upper quadrant pain
44. Which of the following laboratory results would be expected in a client with peritonitis?

1. Partial thromboplastin time above 100 seconds


2. Hemoglobin level below 10 mg/dL
3. Potassium level above 5.5 mEq/L
4. White blood cell count above 15,000
45. Which of the following therapies is not included in the medical management of a client
with peritonitis?
1. Broad-spectrum antibiotics
2. Electrolyte replacement
3. I.V. fluids
4. Regular diet
46. Which of the following aspects is the priority focus of nursing management for a client
with peritonitis?
1. Fluid and electrolyte balance
2. Gastric irrigation
3. Pain management
4. Psychosocial issues
47. A client with irritable bowel syndrome is being prepared for discharge. Which of the
following meal plans should the nurse give the client?
1. Low fiber, low-fat
2. High fiber, low-fat
3. Low fiber, high-fat
4. High-fiber, high-fat
48. A client presents to the emergency room, reporting that he has been vomiting every 30
to 40 minutes for the past 8 hours. Frequent vomiting puts him at risk for which of the
following?
1. Metabolic acidosis with hyperkalemia
2. Metabolic acidosis with hypokalemia
3. Metabolic alkalosis with hyperkalemia
4. Metabolic alkalosis with hypokalemia
49. Five days after undergoing surgery, a client develops a small-bowel obstruction. A
Miller-Abbott tube is inserted for bowel decompression. Which nursing diagnosis takes
priority?

1. Imbalanced nutrition: Less than body requirements


2. Acute pain
3. Deficient fluid volume
4. Excess fluid volume
50. When teaching an elderly client how to prevent constipation, which of the following
instructions should the nurse include?
1. Drink 6 glasses of fluid each day.
2. Avoid grain products and nuts.
3. Add at least 4 grams of bran to your cereal each morning.
4. Be sure to get regular exercise.
51. In a client with diarrhea, which outcome indicates that fluid resuscitation is successful?
1. The client passes formed stools at regular intervals
2. The client reports a decrease in stool frequency and liquidity
3. The client exhibits firm skin turgor
4. The client no longer experiences perianal burning.
52. When teaching a community group about measures to prevent colon cancer, which
instruction should the nurse include?
1. Limit fat intake to 20% to 25% of your total daily calories.
2. Include 15 to 20 grams of fiber into your daily diet.
3. Get an annual rectal examination after age 35.
4. Undergo sigmoidoscopy annually after age 50.
53. A 30-year old client experiences weight loss, abdominal distention, crampy abdominal
pain, and intermittent diarrhea after birth of her 2nd child. Diagnostic tests reveal gluteninduced enteropathy. Which foods must she eliminate from her diet permanently?
1. Milk and dairy products
2. Protein-containing foods
3. Cereal grains (except rice and corn)
4. Carbohydrates
54. After a right hemicolectomy for treatment of colon cancer, a 57-year old client is
reluctant to turn while on bed rest. Which action by the nurse would be appropriate?
1. Asking a coworker to help turn the client
2. Explaining to the client why turning is important.

3. Allowing the client to turn when hes ready to do so


4. Telling the client that the physicians order states he must turn every 2 hours
55. A client has a percutaneous endoscopic gastrostomy tube inserted for tube feedings.
Before starting a continuous feeding, the nurse should place the client in which position?
1. Semi-Fowlers
2. Supine
3. Reverse Trendelenburg
4. High Fowlers
56. An enema is prescribed for a client with suspected appendicitis. Which of the following
actions should the nurse take?
1. Prepare 750 ml of irrigating solution warmed to 100*F
2. Question the physician about the order
3. Provide privacy and explain the procedure to the client
4. Assist the client to left lateral Sims position
57. The client being seen in a physicians office has just been scheduled for a barium
swallow the next day. The nurse writes down which of the following instructions for the
client to follow before the test?
1. Fast for 8 hours before the test
2. Eat a regular supper and breakfast
3. Continue to take all oral medications as scheduled.
4. Monitor own bowel movement pattern for constipation
58. The nurse is monitoring a client for the early signs of dumping syndrome. Which
symptom indicates this occurrence?
1. Abdominal cramping and pain
2. Bradycardia and indigestion
3. Sweating and pallor
4. Double vision and chest pain
59. The nurse is preparing a discharge teaching plan for the client who had an umbilical
hernia repair. Which of the following would the nurse include in the plan?
1. Restricting pain medication
2. Maintaining bedrest

3. Avoiding coughing
4. Irrigating the drain
60. The nurse is caring for a hospitalized client with a diagnosis of ulcerative colitis. Which
finding, if noted on assessment of the client, would the nurse report to the physician?
1. Bloody diarrhea
2. Hypotension
3. A hemoglobin of 12 mg/dL
4. Rebound tenderness
61. The nurse is reviewing the record of a client with Crohns disease. Which of the
following stool characteristics would the nurse expect to note documented on the clients
record?
1. Chronic constipation
2. Diarrhea
3. Constipation alternating with diarrhea
4. Stool constantly oozing from the rectum
62. The nurse is performing a colostomy irrigation on a client. During the irrigation, a
client begins to complain of abdominal cramps. Which of the following is the most
appropriate nursing action?
1. Notify the physician
2. Increase the height of the irrigation
3. Stop the irrigation temporarily.
4. Medicate with dilaudid and resume the irrigation
63. The nurse is teaching the client how to perform a colostomy irrigation. To enhance the
effectiveness of the irrigation and fecal returns, what measure should the nurse instruct the
client to do?
1. Increase fluid intake
2. Reduce the amount of irrigation solution
3. Perform the irrigation in the evening
4. Place heat on the abdomen
64. The nurse is reviewing the physicians orders written for a client admitted with acute
pancreatitis. Which physician order would the nurse question if noted on the clients chart?

1. NPO status
2. Insert a nasogastric tube
3. An anticholinergic medication
4. Morphine for pain
65. The nurse is doing an admission assessment on a client with a history of duodenal ulcer.
To determine whether the problem is currently active, the nurse would assess the client for
which of the following most frequent symptom(s) of duodenal ulcer?
1. Pain that is relieved by food intake
2. Pain that radiated down the right arm
3. N/V
4. Weight loss
66. The nurse instructs the ileostomy client to do which of the following as a part of
essential care of the stoma?
1. Cleanse the peristomal skin meticulously
2. Take in high-fiber foods such as nuts
3. Massage the area below the stoma
4. Limit fluid intake to prevent diarrhea.
67. The client who has undergone creation of a colostomy has a nursing diagnosis of
Disturbed body image. The nurse would evaluate that the client is making the most
significant progress toward identified goals if the client:
1. Watches the nurse empty the colostomy bag
2. Looks at the ostomy site
3. Reads the ostomy product literature
4. Practices cutting the ostomy appliance
68. The nurse is assessing for stoma prolapse in a client with a colostomy. The nurse would
observe which of the following if stoma prolapse occurred?
1. Sunken and hidden stoma
2. Dark- and bluish-colored stoma
3. Narrowed and flattened stoma
4. Protruding stoma
69. The client with a new colostomy is concerned about the odor from the stool in the
ostomy drainage bag. The nurse teaches the client to include which of the following foods in
the diet to reduce odor?

1. Yogurt
2. Broccoli
3. Cucumbers
4. Eggs
70. The nurse has given instructions to the client with an ileostomy about foods to eat to
thicken the stool. The nurse determines that the client needs further instructions if the
client stated to eat which of the following foods to make the stools less watery?
1. Pasta
2. Boiled rice
3. Bran
4. Low-fat cheese
71. The client has just had surgery to create an ileostomy. The nurse assesses the client in
the immediate post-op period for which of the following most frequent complications of this
type of surgery?
1. Intestinal obstruction
2. Fluid and electrolyte imbalance
3. Malabsorption of fat
4. Folate deficiency
72. The nurse is doing pre-op teaching with the client who is about to undergo creation of a
Kock pouch. The nurse interprets that the client has the best understanding of the nature
of the surgery if the client makes which of the following statements?
1. I will need to drain the pouch regularly with a catheter.
2. I will need to wear a drainage bag for the rest of my life.
3. The drainage from this type of ostomy will be formed.
4. I will be able to pass stool from my rectum eventually.
73. The client with a colostomy has an order for irrigation of the colostomy. The nurse used
which solution for irrigation?
1. Distilled water
2. Tap water
3. Sterile water
4. Lactated Ringers
74. A nurse is monitoring a client admitted to the hospital with a diagnosis of appendicitis.
The client is scheduled for surgery in 2 hours. The client begins to complain of increased

abdominal pain and begins to vomit. On assessment the nurse notes that the abdomen is
distended and the bowel sounds are diminished. Which of the following is the most
appropriate nursing intervention?
1. Administer dilaudid
2. Notify the physician
3. Call and ask the operating room team to perform the surgery as soon as possible
4. Reposition the client and apply a heating pad on a warm setting to the clients abdomen.
75. The client has been admitted with a diagnosis of acute pancreatitis. The nurse would
assess this client for pain that is:
1. Severe and unrelenting, located in the epigastric area and radiating to the back.
2. Severe and unrelenting, located in the left lower quadrant and radiating to the groin.
3. Burning and aching, located in the epigastric area and radiating to the umbilicus.
4. Burning and aching, located in the left lower quadrant and radiating to the hip.
76. The client with Crohns disease has a nursing diagnosis of acute pain. The nurse would
teach the client to avoid which of the following in managing this problem?
1. Lying supine with the legs straight
2. Massaging the abdomen
3. Using antispasmodic medication
4. Using relaxation techniques
77. A client with ulcerative colitis has an order to begin salicylate medication to reduce
inflammation. The nurse instructs the client to take the medication:
1. 30 minutes before meals
2. On an empty stomach
3. After meals
4. On arising
78. During the assessment of a clients mouth, the nurse notes the absence of saliva. The
client is also complaining of pain near the area of the ear. The client has been NPO for
several days because of the insertion of a NG tube. Based on these findings, the nurse
suspects that the client is developing which of the following mouth conditions?
1. Stomatitis
2. Oral candidiasis
3. Parotitis
4. Gingivitis

79. The nurse evaluates the clients stoma during the initial post-op period. Which of the
following observations should be reported immediately to the physician?
1. The stoma is slightly edematous
2. The stoma is dark red to purple
3. The stoma oozes a small amount of blood
4. The stoma does not expel stool
80. When planning care for a client with ulcerative colitis who is experiencing symptoms,
which client care activities can the nurse appropriately delegate to a unlicensed assistant?
Select all that apply.
1. Assessing the clients bowel sounds
2. Providing skin care following bowel movements
3. Evaluating the clients response to antidiarrheal medications
4. Maintaining intake and output records
5. Obtaining the clients weight.
81. Which goal of the clients care should take priority during the first days of
hospitalization for an exacerbation of ulcerative colitis?
1. Promoting self-care and independence
2. Managing diarrhea
3. Maintaining adequate nutrition
4. Promoting rest and comfort
82. A clients ulcerative colitis symptoms have been present for longer than 1 week. The
nurse recognizes that the client should be assessed carefully for signs of which of the
following complications?
1. Heart failure
2. DVT
3. Hypokalemia
4. Hypocalcemia
83. A client who has ulcerative colitis has persistent diarrhea. He is thin and has lost 12
pounds since the exacerbation of his ulcerative colitis. The nurse should anticipate that the
physician will order which of the following treatment approaches to help the client meet his
nutritional needs?
1. Initiate continuous enteral feedings
2. Encourage a high protein, high-calorie diet

3. Implement total parenteral nutrition


4. Provide six small meals a day.
84. Digoxin preparations and absorbents should not be given simultaneously. As a nurse,
you are aware that if these agents are given simultaneously, which of the following will
occur?
1. Increased absorption of digoxin
2. Decreased absorption of digoxin
3. Increased absorption of the absorbent
4. Decreased absorption of the absorbent
85. When used with hyperacidic disorders of the stomach, antacids are given to elevate the
gastric pH to:
1. 2.0
2. 4.0
3. 6.0
4. >8.0
86. One of your patients is receiving digitalis orally and is also to receive an antacid at the
same time. Your most appropriate action, based on the pharmacokinetics of antacids, is to:
1. Delay the digitalis for 1 to 2 hours until the antacid is absorbed
2. Give the antacid at least 2 to 4 hours before administering the digitalis
3. Administer both medications as ordered and document in nurses notes
4. Contact the physician regarding the drug interaction and request a change in the time of dosing
of the drugs.
87. The nurse would teach patients that antacids are effective in treatment of hyperacidity
because they:
1. Neutralize gastric acid
2. Decrease stomach motility
3. Decrease gastric pH
4. Decrease duodenal pH
88. The nurse would monitor for which of the following adverse reactions to aluminumcontaining antacids such as aluminum hydroxide (Amphojel)?
1. Diarrhea
2. Constipation

3. GI upset
4. Fluid retention
89. The nurse would question an order for which type of antacid in patients with chronic
renal failure?
1. Aluminum-containing antacids
2. Calcium-containing antacids
3. Magnesium-containing antacids
4. All of the above.
90. The nurse would monitor a patient using sodium bicarbonate to treat gastric
hyperacidity for signs and symptoms of:
1. Metabolic alkalosis
2. Metabolic acidosis
3. Hyperkalemia
4. Hypercalcemia
91. Which of the following nursing diagnoses is appropriate for a patient receiving
famotidine (Pepcid)?
1. Increased risk for infection due to immunosuppression
2. Potential risk for bleeding related to thrombocytopenia.
3. Alteration in urinary elimination related to retention
4. Alteration in tissue perfusion related to hypertension
92. Histamine2-receptor antagonists:
1. Compete with histamine for binding sites on the parietal cells
2. Irreversibly bind to H+/K+ATPase
3. Cause a decrease in stomach pH
4. Decrease signs and symptoms of allergies related to histamine release
93. Proton pump inhibitors:
1. Gastric ulcer formation
2. GERD
3. Achlorhydria
4. Diverticulosis

94. A patient unable to tolerate oral medications may be prescribed which of the following
proton pump inhibitors to be administered intravenously?
1. lansoprazole (Prevacid)
2. omeprazole (Prilosec)
3. pantoprazole (Protonix)
4. esomeprazole (Nexium)
95. When administering sucralfate (Carafate) to a patient with a nasogastric tube, it is
important to:
1. Crush the tablet into a fine powder before mixing with water
2. Administer with a bolus tube feeding
3. Allow the tablet to dissolve in water before administering
4. Administer with an antacid for maximum benefit
96. Sucralfate (Carafate) achieves a therapeutic effect by:
1. Neutralizing gastric acid
2. Enhancing gastric absorption
3. Forming a protective barrier around gastric mucosa
4. Inhibiting gastric acid secretion
97. To avoid fecal impaction, psyllium (Metamucil) should be administered with at least
how many ounces of fluid?
1. 4
2. 6
3. 8
4. 10
98. Bismuth subsalicylate (Pepto-Bismol), as an absorbent, has which of the following
mechanisms of action?
1. Decreased GI motility
2. Decreased gastric secretions
3. Increased fluid absorption
4. Binding to diarrhea-causing bacteria for excretion
99. Side effects of loperamide (Imodium) include all of the following except?

1. Diarrhea
2. epigastric pain
3. Dry mouth
4. Anorexia
100. The mechanism of action of diphenoxylate (Lotomil) is:
1. An increase in intestinal excretion of water
2. An increase in intestinal motility
3. A decrease in peristalsis in the intestinal wall
4. A decrease in the reabsorption of water in the bowel
Answers and Rationale

1. Answer: 1. A fecalith
A fecalith is a fecal calculus, or stone, that occludes the lumen of the appendix and is the most
common cause of appendicitis. Bowel wall swelling, kinking of the appendix, and external
occlusion, not internal occlusion, of the bowel by adhesions can also be causes of appendicitis.
2. Answer: 4. Steady
The pain begins in the epigastrium or periumbilical region, then shifts to the right lower quadrant
and becomes steady. The pain may be moderate to severe.
3. Answer: 4. Assessing for symptoms of peritonitis
The focus of care is to assess for peritonitis, or inflammation of the peritoneal cavity. Peritonitis
is most commonly caused by appendix rupture and invasion of bacteria, which could be lethal.
The client with appendicitis will have pain that should be controlled with analgesia. The nurse
should discourage oral intake in preparation of surgery. Discharge teaching is important;
however, in the acute phase, management should focus on minimizing preoperative
complications and recognizing when such may be occurring.
4. Answer: 3. Inflammation of the gastric mucosa
Gastritis is an inflammation of the gastric mucosa that may be acute (often resulting from
exposure to local irritants) or chronic (associated with autoimmune infections or atrophic
disorders of the stomach). Erosion of the mucosa results in ulceration. Inflammation of a
diverticulum is called diverticulitis; reflux of stomach acid is known as gastroesophageal disease.
5. Answer: 4. Nonsteroidal anti-inflammatory drugs

NSAIDS are a common cause of gastritis because they inhibit prostaglandin synthesis. Milk,
once thought to help gastritis, has little effect on the stomach mucosa. Bicarbonate of soda, or
baking soda, may be used to neutralize stomach acid, but it should be used cautiously because it
may lead to metabolic acidosis. ASA with enteric coating shouldnt contribute significantly to
gastritis because the coating limits the aspirins effect on the gastric mucosa.
6. Answer: 2. A noninflamed outpouching of the intestine
Diverticulosis involves a noninflamed outpouching of the intestine. Diverticulitis involves an
inflamed outpouching. The partial impairment of forward flow of the intestine is an obstruction;
abnormal protrusion of an organ is a hernia.
7. Answer: 1. Low-fiber diet
Low-fiber diets have been implicated in the development of diverticula because these diets
decrease the bulk in the stool and predispose the person to the development of constipation. A
high-fiber diet is recommended to help prevent diverticulosis. A high-protein or lowcarbohydrate diet has no effect on the development of diverticulosis.
8. Answer: 4. Undigested food blocking the diverticulum, predisposing the area to bacterial
invasion.
Undigested food can block the diverticulum, decreasing blood supply to the area and
predisposing the area to invasion of bacteria. Chronic laxative use is a common problem in
elderly clients, but it doesnt cause diverticulitis. Chronic constipation can cause an obstruction
not diverticulitis. Herniation of the intestinal mucosa causes an intestinal perforation.
9. Answer: 1. No symptoms exist
Diverticulosis is an asymptomatic condition. The other choices are signs and symptoms of
diverticulitis.
10. Answer: 2. Barium enema
A barium enema will cause diverticula to fill with barium and be easily seen on x-ray. An
abdominal US can tell more about structures, such as the gallbladder, liver, and spleen, than the
intestine. A barium swallow and gastroscopy view upper GI structures.
11. Answer: 3. Administration of antibiotics
Antibiotics are used to reduce the inflammation. The client isnt typically isnt allowed anything
orally until the acute episode subsides. Parenteral fluids are given until the client feels better;

then its recommended that the client drink eight 8-ounce glasses of water per day and gradually
increase fiber in the diet to improve intestinal motility. During the acute phase, activities that
increase intra-abdominal pressure should be avoided to decrease pain and the chance of intestinal
obstruction.
12. Answer: 4. The small intestine and colon; affecting the entire thickness of the bowel
Crohns disease can involve any segment of the small intestine, the colon, or both, affecting the
entire thickness of the bowel. Answers 1 and 3 describe ulcerative colitis, answer 2 is too specific
and therefore, not likely.
13. Answer: 4. Terminal ileum
Studies have shown that the terminal ileum is the most common site for recurrence in clients
with Crohns disease. The other areas may be involved but arent as common.
14. Answer: 3. Hereditary
Although the definite cause of Crohns disease is unknown, its thought to be associated with
infectious, immune, or psychological factors. Because it has a higher incidence in siblings, it
may have a genetic cause.
15. Answer: 2. Altered immunity
Several theories exist regarding the cause of ulcerative colitis. One suggests altered immunity as
the cause based on the extraintestinal characteristics of the disease, such as peripheral arthritis
and cholangitis. Diet and constipation have no effect on the development of ulcerative colitis.
Emotional stress can exacerbate the attacks but isnt believed to be the primary cause.
16. Answer: 1. Crohns disease
The lesions of Crohns disease are transmural; that is, they involve all thickness of the bowel.
These lesions may perforate the bowel wall, forming fistulas with adjacent structures. Fistulas
dont develop in diverticulitis or diverticulosis. The ulcers that occur in the submucosal and
mucosal layers of the intestine in ulcerative colitis usually dont progress to fistula formation as
in Crohns disease.
17. Answer: 1. Anorectal
Fistulas occur in all these areas, but the anorectal area is most common because of the relative
thinness of the intestinal wall in this area.

18. Answer: 4. Toxic megacolon


Toxic megacolon is extreme dilation of a segment of the diseased colon caused by paralysis of
the colon, resulting in complete obstruction. This disorder is associated with both Crohns
disease and ulcerative colitis. The other disorders are more commonly associated with Crohns
disease.
19. Answer: 3. Malabsorption
Because of the transmural nature of Crohns disease lesions, malabsorption may occur with
Crohns disease. Ankylosing spondylitis and colon cancer are more commonly associated with
ulcerative colitis. Lactase deficiency is caused by a congenital defect in which an enzyme isnt
present.
20. Answer: 4. Steatorrhea
Steatorrhea from malaborption can occur with Crohns disease. N/V, and bloody diarrhea are
symptoms of ulcerative colitis. Narrow stools are associated with diverticular disease.
21. Answer: 2. Rectal bleeding
In ulcerative colitis, rectal bleeding is the predominant symptom. Soft stools are more commonly
associated with Crohns disease, in which malabsorption is more of a problem. Dumping
syndrome occurs after gastric surgeries. Fistulas are associated with Crohns disease.
22. Answer: 4. Colonoscopy with biopsy
A colonoscopy with biopsy can be performed to determine the state of the colons mucosal
layers, presence of ulcerations, and level of cytologic development. An abdominal x-ray or CT
scan wouldnt provide the cytologic information necessary to diagnose which disease it is. A
barium swallow doesnt involve the intestine.
23. Answer: 3. Using long-term steroid therapy
Management of Crohns disease may include long-term steroid therapy to reduce the
inflammation associated with the deeper layers of the bowel wall. Other management focuses on
bowel rest (not increasing oral intake) and reducing diarrhea with medications (not giving
laxatives). The pain associated with Crohns disease may require bed rest, not an increase in
physical activity.
24. Answer: 3. Decrease in body weight

A decrease in body weight may occur during therapy due to inadequate dietary intake, but isnt
related to antibiotic therapy. Effective antibiotic therapy will be noted by a decrease in
temperature, number of stools, and bleeding.
25. Answer: 4. Bowel perforation
Perforation, obstruction, hemorrhage, and toxic megacolon are common complications of
ulcerative colitis that may require surgery. Herniation and gastritis arent associated with irritable
bowel diseases, and outpouching of the bowel is diverticulosis.
26. Answer: 3. Steroids
The pain with irritable bowel disease is caused by inflammation, which steroids can reduce.
Stool softeners arent necessary. Acetaminophen has little effect on the pain, and opiate narcotics
wont treat its underlying cause.
27. Answer: 2. Ostomy care
Although all of these are concerns the nurse should address, being able to safely manage the
ostomy is crucial for the client before discharge.
28. Answer: 4. Ulcerative colitis
Chronic ulcerative colitis, granulomas, and familial polposis seem to increase a persons chance
of developing colon cancer. The other conditions listed have no known effect on colon cancer
risk.
29. Answer: 1. Low-fiber, high fat
A low-fiber, high-fat diet reduced motility and increases the chance of constipation. The
metabolic end products of this type of diet are carcinogenic. A low-fat, high-fiber diet is
recommended to prevent colon cancer.
30. Answer: 4. Fecal occult blood test
Surface blood vessels of polyps and cancers are fragile and often bleed with the passage of
stools. Abdominal x-ray and CT scan can help establish tumor size and metastasis. A
colonoscopy can help locate a tumor as well as polyps, which can be removed before they
become malignant.
31. Answer: 1. Reducing the size of the tumor

Radiation therapy is used to treat colon cancer before surgery to reduce the size of the tumor,
making it easier to be resected. Radiation therapy isnt curative, cant eliminate the malignant
cells (though it helps define tumor margins), can could slow postoperative healing.
32. Answer: 2. A change in bowel habits
The most common complaint of the client with colon cancer is a change in bowel habits. The
client may have anorexia, secondary abdominal distention, or weight loss. Fever isnt associated
with colon cancer.
33. Answer: 1. Peritonitis
Bowel spillage could occur during surgery, resulting in peritonitis. Complete or partial bowel
obstruction may occur before bowel resection. Diverticulosis doesnt result from surgery or colon
cancer.
34. Answer: 3. Feeling of fullness
The client with gastric cancer may report a feeling of fullness in the stomach, but not enough to
cause him to seek medical attention. Abdominal cramping isnt associated with gastric cancer.
Anorexia and weight loss (not increased hunger or weight gain) are common symptoms of
gastric cancer.
35. Answer: 3. Gastroscopy
A gastroscopy will allow direct visualization of the tumor. A colonoscopy or a barium enema
would help diagnose colon cancer. Serum chemistry levels dont contribute data useful to the
assessment of gastric cancer.
36. Answer: 2. Correction of nutritional deficits
Clients with gastric cancer commonly have nutritional deficits and may be cachectic. Discharge
planning before surgery is important, but correcting the nutrition deficit is a higher priority. At
present, radiation therapy hasnt been proven effective for gastric cancer, and teaching about it
preoperatively wouldnt be appropriate. Prevention of DVT also isnt a high priority to surgery,
though it assumes greater importance after surgery.
37. Answer: 2. Nutritional needs
After gastric resection, a client may require total parenteral nutrition or jejunostomy tube
feedings to maintain adequate nutritional status.

38. Answer: 2. Dumping syndrome


Dumping syndrome is a problem that occurs postprandially after gastric resection because
ingested food rapidly enters the jejunum without proper mixing and without the normal duodenal
digestive processing. Diarrhea, not constipation, may also be a symptom. Gastric or intestinal
spasms dont occur, but antispasmodics may be given to slow gastric emptying.
39. Answer: 3. Rectal bleeding
Rectal bleeding is a common symptom of rectal cancer. Rectal cancer may be missed because
other conditions such as hemorrhoids can cause rectal bleeding. Abdominal fullness may occur
with colon cancer, gastric fullness may occur with gastric cancer, and right upper quadrant pain
may occur with liver cancer.
40. Answer: 1. Adenomatous polyps
A client with adenomatous polyps has a higher risk for developing rectal cancer than others do.
Clients with diverticulitis are more likely to develop colon cancer. Hemorrhoids dont increase
the chance of any type of cancer. Clients with peptic ulcer disease have a higher incidence of
gastric cancer.
41. Answer: 3. Radiation
A client with rectal cancer can expect to have radiation therapy in addition to chemotherapy and
surgical resection of the tumor. A colonoscopy is performed to diagnose the disease. Radiation
therapy isnt usually indicated in colon cancer.
42. Answer: 3. Perforated ulcer
The most common cause of peritonitis is a perforated ulcer, which can pour contaminates into
the peritoneal cavity, causing inflammation and infection within the cavity. The other conditions
dont by themselves cause peritonitis. However, if cholelithiasis leads to rupture of the
gallbladder, gastritis leads to erosion of the stomach wall, or an incarcerated hernia leads to
rupture of the intestines, peritonitis may develop.
43. Answer: 2. Abdominal pain and rigidity
Abdominal pain causing rigidity of the abdominal muscles is characteristic of peritonitis.
Abdominal distention may occur as a late sign but not early on. Bowel sounds may be normal or
decreased but not increased. Right upper quadrant pain is characteristic of cholecystitis or
hepatitis.

44. Answer: 4. White blood cell count above 15,000


Because of infection, the clients WBC count will be elevated. A hemoglobin level below 10
mg/dl may occur from hemorrhage. A PT time longer than 100 seconds may suggest
disseminated intravascular coagulation, a serious complication of septic shock. A potassium level
above 5.5 mEq/L may indicate renal failure.
45. Answer: 4. Regular diet
The client with peritonitis usually isnt allowed anything orally until the source of peritonitis is
confirmed and treated. The client also requires broad-spectrum antibiotics to combat the
infection. I.V. fluids are given to maintain hydration and hemodynamic stability and to replace
electrolytes.
46. Answer: 1. Fluid and electrolyte balance
Peritonitis can advance to shock and circulatory failure, so fluid and electrolyte balance is the
priority focus of nursing management. Gastric irrigation may be needed periodically to ensure
patency of the nasogastric tube. Although pain management is important for comfort and
psychosocial care will address concerns such as anxiety, focusing on fluid and electrolyte
imbalance will maintain hemodynamic stability.
47. Answer: 2. High fiber, low-fat
The client with irritable bowel syndrome needs to be on a diet that contains at least 25 grams of
fiber per day. Fatty foods are to be avoided because they may precipitate symptoms.
48. Answer: 4. Metabolic alkalosis with hypokalemia
Gastric acid contains large amounts of potassium, chloride, and hydrogen ions. Excessive loss of
these substances, such as from vomiting, can lead to metabolic alkalosis and hypokalemia.
49. Answer: 3. Deficient fluid volume
Fluid shifts to the site of the bowel obstruction, causing a fluid deficit in the intravascular spaces.
If the obstruction isnt resolved immediately, the client may experience an imbalanced nutritional
status (less than body requirements); however, deficient fluid volume takes priority. The client
may also experience pain, but that nursing diagnosis is also of lower priority than deficient fluid
volume.
50. Answer: 4. Be sure to get regular exercise.

Exercise helps prevent constipation. Fluids and dietary fiber promote normal bowel function.
The client should drink eight to ten glasses of fluid each day. Although adding bran to cereal
helps prevent constipation by increasing dietary fiber, the client should start with a small amount
and gradually increase the amount as tolerated to a maximum of 2 grams a day.
51. Answer: 3. The client exhibits firm skin turgor
A client with diarrhea has a nursing diagnosis of Deficient fluid volume related to excessive fluid
loss in the stool. Expected outcomes include firm skin turgor, moist mucous membranes, and
urine output of at least 30 ml/hr. The client also has a nursing diagnosis of diarrhea, with
expected outcomes of passage of formed stools at regular intervals and a decrease in stool
frequency and liquidity. The client is at risk for impaired skin integrity related to irritation from
diarrhea; expected outcomes for this diagnosis include absence of erythema in perianal skin and
mucous membranes and absence of perianal tenderness or burning.
52. Answer: 1. Limit fat intake to 20% to 25% of your total daily calories.
To help prevent colon cancer, fats should account for no more than 20% to 25% of total daily
calories and the diet should include 25 to 30 grams of fiber per day. A digital rectal examination
isnt recommended as a stand-alone test for colorectal cancer. For colorectal cancer screening,
the American Cancer society advises clients over age 50 to have a flexible sigmoidoscopy every
5 years, yearly fecal occult blood tests, yearly fecal occult blood tests PLUS a flexible
sigmoidoscopy every 5 years, a double-contrast barium enema every 5 years, or a colonoscopy
every 10 years.
53. Answer: 3. Cereal grains (except rice and corn)
To manage gluten-induced enteropathy, the client must eliminate gluten, which means avoiding
all cereal grains except for rice and corn. In initial disease management, clients eat a high calorie,
high-protein diet with mineral and vitamin supplements to help normalize nutritional status.
54. Answer: 2. Explaining to the client why turning is important.
The appropriate action is to explain the importance of turning to avoid postoperative
complications. Asking a coworker to help turn the client would infringe on his rights. Allowing
him to turn when hes ready would increase his risk for postoperative complications. Telling him
he must turn because of the physicians orders would put him on the defensive and exclude him
from participating in care decision.
55. Answer: 1. Semi-Fowlers

To prevent aspiration of stomach contents, the nurse should place the client in semi-Fowlers
position. High Fowlers position isnt necessary and may not be tolerated as well as semiFowlers.
56. Answer: 2. Question the physician about the order
Enemas are contraindicated in an acute abdominal condition of unknown origin as well as after
recent colon or rectal surgery or myocardial infarction. The other answers are correct only when
enema administration is appropriate.
57. Answer: 1. Fast for 8 hours before the test
A barium swallow is an x-ray study that uses a substance called barium for contrast to highlight
abnormalities in the GI tract. The client should fast for 8 to 12 hours before the test, depending
on the physician instructions. Most oral medications also are withheld before the test. After the
procedure the nurse must monitor for constipation, which can occur as a result of the presence of
barium in the GI tract.
58. Answer: 3. Sweating and pallor
Early manifestations of dumping syndrome occur 5 to 30 minutes after eating. Symptoms include
vertigo, tachycardia, syncope, sweating, pallor, palpitations, and the desire to lie down.
59. Answer: 3. Avoiding coughing
Bedrest is not required following this surgical procedure. The client should take analgesics as
needed and as prescribed to control pain. A drain is not used in this surgical procedure, although
the client may be instructed in simple dressing changes. Coughing is avoided to prevent
disruption of the tissue integrity, which can occur because of the location of this surgical
procedure.
60. Answer: 4. Rebound tenderness
Rebound tenderness may indicate peritonitis. Blood diarrhea is expected to occur in ulcerative
colitis. Because of the blood loss, the client may be hypotensive and the hemoglobin level may
be lower than normal. Signs of peritonitis must be reported to the physician.
61. Answer: 2. Diarrhea
Crohns disease is characterized by nonbloody diarrhea of usually not more than four to five
stools daily. Over time, the diarrhea episodes increase in frequency, duration and severity. The
other option are not associated with diarrhea.

62. Answer: 3. Stop the irrigation temporarily.


If cramping occurs during a colostomy irrigation, the irrigation flow is stopped temporarily and
the client is allowed to rest. Cramping may occur from an infusion that is too rapid or is causing
too much pressure. Increasing the height of the irrigation will cause further discomfort. The
physician does not need to be notified. Medicating the client for pain is not the most appropriate
action.
63. Answer: 1. Increase fluid intake
To enhance effectiveness of the irrigation and fecal returns, the client is instructed to increase
fluid intake and prevent constipation.
64. Answer: 4. Morphine for pain
Meperidine (Demerol) rather than morphine is the medication of choice because morphine can
cause spasm in the sphincter of Oddi.
65. Answer: 1. Pain that is relieved by food intake
The most frequent symptom of duodenal ulcer is pain that is relieved by food intake. These
clients generally describe the pain as burning, heavy, sharp, or hungry pain that often localizes
in the midepigastric area. The client with duodenal ulcer usually does not experience weight loss
or N/V. These symptoms are usually more typical in the client with a gastric ulcer.
66. Answer: 1. Cleanse the peristomal skin meticulously
The peristomal skin must receive meticulous cleansing because the ileostomy drainage has more
enzymes and is more caustic to the skin than colostomy drainage. Foods such as nuts and those
with seeds will pass through the ileostomy. The client should be taught that these foods will
remain undigested. The area below the ileostomy may be massaged if needed if the ileostomy
becomes blocked by high fiber foods. Fluid intake should be maintained to at least six to eight
glasses of water per day to prevent dehydration.
67. Answer: 4. Practices cutting the ostomy appliance
The client is expected to have a body image disturbance after colostomy. The client progresses
through normal grieving stages to adjust to this change. The client demonstrates the greatest deal
of acceptance when the client participates in the actual colostomy care. Each of the incorrect
options represents an interest in colostomy care but is a passive activity. The correct option
shows the client is participating in self-care.

68. Answer: 4. Protruding stoma


A prolapsed stoma is one which the bowel protruded through the stoma. A stoma retraction is
characterized by sinking of the stoma. Ischemia of the stoma would be associated with dusky or
bluish color. A stoma with a narrowed opening at the level of the skin or fascia is said to be
stenosed.
69. Answer: 1. Yogurt
The client should be taught to include deodorizing foods in the diet, such a beet greens, parsley,
buttermilk, and yogurt. Spinach also reduces odor but is a gas forming food as well. Broccoli,
cucumbers, and eggs are gas forming foods.
70. Answer: 3. Bran
Foods that help thicken the stool of the client with an ileostomy include pasta, boiled rice, and
low-fat cheese. Bran is high in dietary fiber and thus will increase output of watery stool by
increasing propulsion through the bowel. Ileostomy output is liquid. Addition or elimination of
various foods can help thicken or loosen this liquid drainage.
71. Answer: 2. Fluid and electrolyte imbalance
A major complication that occurs most frequent following an ileostomy is fluid and electrolyte
imbalance. The client requires constant monitoring of intake and output to prevent this from
happening. Losses require replacement by intravenous infusion until the client can tolerate a diet
orally. Intestinal obstruction is a less frequent complication. Fat malabsorption and folate
deficiency are complications that could occur later in the postoperative period.
72. Answer: 1. I will need to drain the pouch regularly with a catheter.
A Kock pouch is a continent ileostomy. As the ileostomy begins to function, the client drains it
every 3 to 4 hours and then decreases the draining to about 3 times a day or as needed when full.
The client does not need to wear a drainage bag but should wear an absorbent dressing to absorb
mucus drainage from the stoma. Ileostomy drainage is liquid. The client would be able to pass
stool only from the rectum if an ileal-anal pouch or anastamosis were created. This type of
operation is a two-stage procedure.
73. Answer: 2. Tap water
Warm tap water or saline solution is used to irrigate a colostomy. If the tap water is not suitable
for drinking, then bottled water should be used.

74. Answer: 2. Notify the physician


Based on the signs and symptoms presented in the question, the nurse should suspect peritonitis
and should notify the physician. Administering pain medication is not an appropriate
intervention. Heat should never be applied to the abdomen of a client with suspected
appendicitis. Scheduling surgical time is not within the scope of nursing practice, although the
physician probably would perform the surgery earlier than the prescheduled time.
75. Answer: 1. Severe and unrelenting, located in the epigastric area and radiating to the
back.
The pain associated with acute pancreatitis is often severe and unrelenting, is located in the
epigastric region, and radiates to the back.
76. Answer: 1. Lying supine with the legs straight
The pain associated with Crohns disease is alleviated by the use of analgesics and
antispasmodics and also is reduced by having the client practice relaxation techniques, applying
local cold or heat to the abdomen, massaging the abdomen, and lying with the legs flexed. Lying
with the legs extended is not useful because it increases the muscle tension in the abdomen,
which could aggravate the inflamed intestinal tissues as the abdominal muscles are stretched.
77. Answer: 3. After meals
Salicylate compounds act by inhibiting prostaglandin synthesis and reducing inflammation. The
nurse teaches the client to take the medication with a full glass of water and to increase fluid
intake throughout the day. This medication needs to be taken after meals to reduce GI irritation.
78. Answer: 3. Parotitis
The lack of saliva, pain near the area of the ear, and the prolonged NPO status of the client
should lead the nurse to suspect the development of parotitis, or inflammation of the parotid
gland. Parotitis usually develops in cases of dehydration combined with poor oral hygiene or
when clients have been NPO for an extended period. Preventative measures include the use of
sugarless hard candy or gum to stimulate saliva production, adequate hydration, and frequent
mouth care. Stomatitis (inflammation of the mouth) produces excessive salivation and a sore
mouth.
79. Answer: 2. The stoma is dark red to purple

A dark red to purple stoma indicates inadequate blood supply. Mild edema and slight oozing of
blood are normal in the early post-op period. The colostomy would typically not begin
functioning until 2-4 days after surgery.
80. Answer: 2, 4, and 5.
The nurse can delegate the following basic care activities to the unlicensed assistant: providing
skin care following bowel movements, maintaining intake and output records, and obtaining the
clients weight. Assessing the clients bowel sounds and evaluating the clients response to
medication are registered nurse activities that cannot be delegated.
81. Answer: 2. Managing diarrhea
Diarrhea is the primary symptom in an exacerbation of ulcerative colitis, and decreasing the
frequency of stools is the first goal of treatment. The other goals are ongoing and will be best
achieved by halting the exacerbation. The client may receive antidiarrheal medications,
antispasmodic agents, bulk hydrophilic agents, or anti-inflammatory drugs.
82. Answer: 3. Hypokalemia
Excessive diarrhea causes significant depletion of the bodys stores of sodium and potassium as
well as fluid. The client should be closely monitored for hypokalemia and hyponatremia.
Ulcerative colitis does not place the client at risk for heart failure, DVT, or hypocalcemia.
83. Answer: 3. Implement total parenteral nutrition
Food will be withheld from the client with severe symptoms of ulcerative colitis to rest the
bowel. To maintain the clients nutritional status, the client will be started on TPN. Enteral
feedings or dividing the diet into 6 small meals does not allow the bowel to rest. A high-calorie,
high-protein diet will worsen the clients symptoms.
84. Answer: 2. Decreased absorption of digoxin
85. Answer: 1. 2.0
86. Answer: 4. Contact the physician regarding the drug interaction and request a change
in the time of dosing of the drugs.
87. Answer: 1. Neutralize gastric acid
Antacids work by neutralizing gastric acid, which would cause an increase in pH. They do not
affect gastric motility.

88. Answer: 2. Constipation


Aluminum- and calcium-containing antacids cause constipation, magnesium-containing antacids
cause diarrhea, and sodium-containing antacids cause sodium and fluid retention.
89. Answer: 3. Magnesium-containing antacids
Magnesium-containing antacids can cause hypermagnesemia in patients with chronic renal
failure. Aluminum-containing antacids may be used as a phosphate binder in patients with
chronic renal failure. Calcium-containing antacids are also appropriate because these patients
may be hypocalcemic.
90. Answer: 1. Metabolic alkalosis
Solutions containing sodium bicarbonate (a base) can cause metabolic alkalosis. Serum K and
serum calcium would decrease with alkalosis, not increase.
91. Answer: 2. Potential risk for bleeding related to thrombocytopenia.
A serious side effect of famotidine is thrombocytopenia, which is manifested by a decrease in
platelet count and an increased risk of bleeding.
92. Answer: 1. Compete with histamine for binding sites on the parietal cells
Histamine receptor blocking agents decrease gastric acid by competing with histamine for
binding sites on the parietal cells.
93. Answer: 3. Achlorhydria
Because the proton pump inhibitors stop the final step of acid secretion, they can block up to
90% of acid secretion, leading to achlorhydria (without acid).
94. Answer: 3. pantoprazole (Protonix)
Pantoprazole is the only proton pump inhibitor that is available for intravenous administration.
The other medications in this category may only be administered orally.
95. Answer: 3. Allow the tablet to dissolve in water before administering
It is important to give sucralfate on an empty stomach so that it may dissolve and form a
protective barrier over the gastric mucosa. The tablet form will not dissolve in water when

crushed; it must be left whole and allowed to dissolve. Crushing the medication so that it will not
dissolve could lead to clogging of the nasogastric tube and decreased effectiveness of the drug.
96. Answer: 3. Forming a protective barrier around gastric mucosa
Sucralfate has a local effect only on the gastric mucosa. It forms a paste-like substance in the
stomach, which adheres to the gastric lining, protecting against adverse effects related to gastric
acid. It also stimulates healing of any ulcerated areas of the gastric mucosa.
97. Answer: 3. 8
Bulk-forming laxatives must be given with at least 8 ounces of liquid plus additional liquid each
day to prevent intestinal obstruction.
98. Answer: 4. Binding to diarrhea-causing bacteria for excretion
Absorbent antidiarrheal medications bind to diarrhea-causing bacteria to form a nonabsorbable
complex, which is then excreted in the stool.
99. Answer: 1. Diarrhea
Side effects associated with loperamide include CNS fatigue and dizziness, epigastric pain,
abdominal cramps, nausea, dry mouth, vomiting, and anorexia. Diarrhea is an indication, not a
side effect.
100. Answer: 3. A decrease in peristalsis in the intestinal wall
Diphenoxylate acts on the smooth muscle of the intestinal tract to inhibit GI motility and
excessive propulsion of the GI tract (peristalsis).

3. Topics
Included topics in this practice quiz are:

Hiatal Hernia

Esophageal Disorders

Guidelines
Follow the guidelines below to make the most out of this exam:

Read each question carefully and choose the best answer.

You are given one minute per question. Spend your time wisely!

Answers and rationales are given below. Be sure to read them.

If you need more clarifications, please direct them to the comments section.

n Exam Mode: All questions are shown in random and the results, answers and rationales (if
any) will only be given after youve finished the quiz. You are given 1 minute per question, a
total of 50 minutes for this exam.

NCLEX Exam: Gastrointestinal Disorders 3 (50 Items)


49:36
Question 1
A client with suspected gastric cancer undergoes an endoscopy of the stomach.
Which of the following assessments made after the procedure would indicate the
development of a potential complication?
A The client complains of a sore throat
B The client displays signs of sedation
C The client experiences a sudden increase in temperature
D The client demonstrates a lack of appetite
Question 2
After a subtotal gastrectomy, the nurse should anticipate that nasogastric tube
drainage will be what color for about 12 to 24 hours after surgery?
A

Dark brown

Bile green

Bright red

Cloudy white

Question 3
The nurse would assess the client experiencing an acute episode of cholecysitis for
pain that is located in the right

A Upper quadrant and radiates to the left scapula and shoulder


B Upper quadrant and radiates to the right scapula and shoulder
C Lower quadrant and radiates to the umbilicus
D Lower quadrant and radiates to the back
Question 4
Which of the following tasks should be included in the immediate postoperative
management of a client who has undergone gastric resection?
A

Monitoring gastric pH to detect complications

Assessing for bowel sounds

Providing nutritional support

Monitoring for symptoms of hemorrhage

Question 5
The nurse is providing discharge instructions to a client following gastrectomy.
Which measure will the nurse instruct the client to follow to assist in preventing
dumping syndrome?
A

Eat high-carbohydrate foods

Limit the fluids taken with meals

Ambulate following a meal

Sit in a high-Fowlers position during meals

Question 6
The client with GERD complains of a chronic cough. The nurse understands that in a
client with GERD this symptom may be indicative of which of the following
conditions?
A

Development of laryngeal cancer

Irritation of the esophagus

Esophageal scar tissue formation

Aspiration of gastric contents

Question 7

If a gastric acid perforates, which of the following actions should not be included in
the immediate management of the client?
A

Blood replacement

Antacid administration

Nasogastric tube suction

Fluid and electrolyte replacement

Question 8
Which of the following factors should be the main focus of nursing management for
a client hospitalized for cholecystitis?
A

Administration of antibiotics

Assessment for complications

Preparation for lithotripsy

Preparation for surgery

Question 9
The nurse is caring for a client with chronic gastritis. The nurse monitors the client,
knowing that this client is at risk for which of the following vitamin deficiencies?
A

Vitamin A

Vitamin B12

Vitamin C

Vitamin E

Question 10
Which of the following tests can be performed to diagnose a hiatal hernia?
A

Colonoscopy

Lower GI series

Barium swallow

Abdominal x-rays

Question 11

A client has been taking aluminum hydroxide 30 mL six times per day at home to
treat his peptic ulcer. He tells the nurse that he has been unable to have a bowel
movement for 3 days. Based on this information, the nurse would determine that
which of the following is the most likely cause of the clients constipation?
A The client has not been including enough fiber in his diet
B The client needs to increase his daily exercise
C The client is experiencing a side effect of the aluminum hydroxide.
D The client has developed a gastrointestinal obstruction
Question 12
While caring for a client with peptic ulcer disease, the client reports that he has
been nauseated most of the day and is now feeling lightheaded and dizzy. Based
upon these findings, which nursing actions would be most appropriate for the nurse
to take? Select all that apply.
A

Administering an antacid hourly until nausea subsides.

Monitoring the clients vital signs

Notifying the physician of the clients symptoms

D Initiating oxygen therapy


E

Reassessing the client on an hour

Question 13
A client is admitted to the hospital after vomiting bright red blood and is diagnosed
with a bleeding duodenal ulcer. The client develops a sudden, sharp pain in the mid
epigastric area along with a rigid, board-like abdomen. These clinical manifestations
most likely indicate which of the following?
A

An intestinal obstruction has developed

Additional ulcers have developed

The esophagus has become inflamed

The ulcer has perforated

Question 14
The nurse is caring for a client who has had a gastroscopy. Which of the following
symptoms may indicate that the client is developing a complication related to the
procedure? Select all that apply.

The client complains of a sore throat

The client has a temperature of 100*F

The client appears drowsy following the procedure

The client complains of epigastric pain

The client experiences hematemesis

Question 15
A client is taking an antacid for treatment of a peptic ulcer. Which of the following
statements best indicates that the client understands how to correctly take the
antacid?
A I should take my antacid before I take my other medications.
B

I need to decrease my intake of fluids so that I dont dilute the effects of my


antacid.

My antacid will be most effective if I take it whenever I experience stomach


pains.

D It is best for me to take my antacid 1 to 3 hours after meals.


Question 16
The hospitalized client with GERD is complaining of chest discomfort that feels like
heartburn following a meal. After administering an ordered antacid, the nurse
encourages the client to lie in which of the following positions?
A Supine with the head of the bed flat
B On the stomach with the head flat
C On the left side with the head of the bed elevated 30 degrees
D On the right side with the head of the bed elevated 30 degrees
Question 17
Which of the following symptoms is common with a hiatal hernia?
A

Left arm pain

Lower back pain

Esophageal reflux

Abdominal cramping

Question 18
A client with peptic ulcer disease tells the nurse that he has black stools, which he
has not reported to his physician. Based on this information, which nursing
diagnosis would be appropriate for this client?
A Ineffective coping related to fear of diagnosis of chronic illness
B Deficient knowledge related to unfamiliarity with significant signs and symptoms
C Constipation related to decreased gastric motility
D Imbalanced nutrition: Less than body requirements due to gastric bleeding
Question 19
Which of the following best describes the method of action of medications, such as
ranitidine (Zantac), which are used in the treatment of peptic ulcer disease?
A

Neutralize acid

Reduce acid secretions

Stimulate gastrin release

Protect the mucosal barrier

Question 20
When a client has peptic ulcer disease, the nurse would expect a priority
intervention to be:
A

Assisting in inserting a Miller-Abbott tube

Assisting in inserting an arterial pressure line

Inserting a nasogastric tube

Inserting an I.V.

Question 21
Which of the following symptoms best describes Murphys sign?
A Periumbilical ecchymosis exists
B On deep palpitation and release, pain in elicited
C On deep inspiration, pain is elicited and breathing stops

D Abdominal muscles are tightened in anticipation of palpation


Question 22
A client being treated for chronic cholecystitis should be given which of the
following instructions?
A

Increase rest

Avoid antacids

Increase protein in diet

Use anticholinergics as prescribed

Question 23
A 40-year-old male client has been hospitalized with peptic ulcer disease. He is
being treated with a histamine receptor antagonist (cimetidine), antacids, and diet.
The nurse doing discharge planning will teach him that the action of cimetidine is
to:
A

Reduce gastric acid output

Protect the ulcer surface

Inhibit the production of hydrochloric acid (HCl)

Inhibit vagus nerve stimulation

Question 24
The nurse instructs the nursing assistant on how to provide oral hygiene for a client
who cannot perform this task for himself. Which of the following techniques should
the nurse tell the assistant to incorporate into the clients daily care?
A Assess the oral cavity each time mouth care is given and record observations
B Use a soft toothbrush to brush the clients teeth after each meal
C

Swab the clients tongue, gums, and lips with a soft foam applicator every 2
hours

D Rinse the clients mouth with mouthwash several times a day


Question 25
When obtaining a nursing history on a client with a suspected gastric ulcer, which
signs and symptoms would the nurse expect to see? Select all that apply.
A

Epigastric pain at night

Relief of epigastric pain after eating

Vomiting

Weight loss

Question 26
Which of the following tests is most commonly used to diagnose cholecystitis?
A

Abdominal CT scan

Abdominal ultrasound

Barium swallow

Endoscopy

Question 27
The most important pathophysiologic factor contributing to the formation of
esophageal varices is:
A

Decreased prothrombin formation

Decreased albumin formation by the liver

Portal hypertension

Increased central venous pressure

Question 28
The client with peptic ulcer disease is scheduled for a pyloroplasty. The client asks
the nurse about the procedure. The nurse plans to respond knowing that a
pyloroplasty involves:
A Cutting the vagus nerve
B Removing the distal portion of the stomach
C

Removal of the ulcer and a large portion of the cells that produce hydrochloric
acid

An incision and resuturing of the pylorus to relax the muscle and enlarge the
opening from the stomach to the duodenum

Question 29
Which of the following tests can be used to diagnose ulcers?

Abdominal x-ray

Barium swallow

Computed tomography (CT) scan

Esophagogastroduodenoscopy (EGD)

Question 30
A client is to take one daily dose of ranitidine (Zantac) at home to treat her peptic
ulcer. The nurse knows that the client understands proper drug administration of
ranitidine when she says that she will take the drug at which of the following times?
A

Before meals

With meals

At bedtime

When pain occurs

Question 31
The nurse is caring for a client following a Billroth II procedure. On review of the
post-operative orders, which of the following, if prescribed, would the nurse
question and verify?
A

Irrigating the nasogastric tube

Coughing a deep breathing exercises

Leg exercises

Early ambulation

Question 32
The nurse is assessing a client 24 hours following a cholecystectomy. The nurse
notes that the T-tube has drained 750ml of green-brown drainage. Which nursing
intervention is most appropriate?
A

Notify the physician

Document the findings

Irrigate the T-tube

Clamp the T-tube

Question 33

The client with a duodenal ulcer may exhibit which of the following findings on
assessment?
A

Hematemesis

Malnourishment

Melena

Pain with eating

Question 34
The client has orders for a nasogastric (NG) tube insertion. During the procedure,
instructions that will assist in the insertion would be:
A

Instruct the client to tilt his head back for insertion in the nostril, then flex his
neck for the final insertion

B After insertion into the nostril, instruct the client to extend his neck
C

Introduce the tube with the clients head tilted back, then instruct him to keep his
head upright for final insertion

D Instruct the client to hold his chin down, then back for insertion of the tube
Question 35
Mucosal barrier fortifiers are used in peptic ulcer disease management for which of
the following indications?
A To inhibit mucus production
B To neutralize acid production
C To stimulate mucus production
D To stimulate hydrogen ion diffusion back into the mucosa
Question 36
Which of the following would be an expected nutritional outcome for a client who
has undergone a subtotal gastrectomy for cancer?
A Regain weight loss within 1 month after surgery
B Resume normal dietary intake of three meals per day
C Control nausea and vomiting through regular use of antiemetics

D Achieve optimal nutritional status through oral or parenteral feedings


Question 37
The client being treated for esophageal varices has a Sengstaken-Blakemore tube
inserted to control the bleeding. The most important assessment is for the nurse to:
A

Check that the hemostat is on the bedside

Monitor IV fluids for the shift

Regularly assess respiratory status

Check that the balloon is deflated on a regular basis

Question 38
The nurse provides medication instructions to a client with peptic ulcer disease.
Which statement, if made by the client, indicates the best understanding of the
medication therapy?
A The cimetidine (Tagamet) will cause me to produce less stomach acid.
B Sucralfate (Carafate) will change the fluid in my stomach.
C Antacids will coat my stomach.
D Omeprazole (Prilosec) will coat the ulcer and help it heal.
Question 39
Which of the following conditions can cause a hiatal hernia?
A

Increased intrathoracic pressure

Weakness of the esophageal muscle

Increased esophageal muscle pressure

Weakness of the diaphragmic muscle

Question 40
A client with a peptic ulcer is scheduled for a vagotomy. The client asks the nurse
about the purpose of this procedure. The nurse tells the client that the procedure:
A

Decreases food absorption in the stomach

Heals the gastric mucosa

Halts stress reactions

Reduces the stimulus to acid secretions

Question 41
After a subtotal gastrectomy, care of the clients nasogastric tube and drainage
system should include which of the following nursing interventions?
A Irrigate the tube with 30 ml of sterile water every hour, if needed
B Reposition the tube if it is not draining well
C Monitor the client for N/V, and abdominal distention
D Turn the machine to high suction of the drainage is sluggish on low suction
Question 42
Which of the following dietary measures would be useful in preventing esophageal
reflux?
A

Eating small, frequent meals

Increasing fluid intake

Avoiding air swallowing with meals

Adding a bedtime snack to the dietary plan

Question 43
A client has been diagnosed with adenocarcinoma of the stomach and is scheduled
to undergo a subtotal gastrectomy (Billroth II procedure). During pre-operative
teaching, the nurse is reinforcing information about the procedure. Which of the
following explanations is most accurate?
A The procedure will result in enlargement of the pyloric sphincter
B The procedure will result in anastomosis of the gastric stump to the jejunum
C The procedure will result in removal of the duodenum
D The procedure will result in repositioning of the vagus nerve
Question 44
Which of the following measures should the nurse focus on for the client with
esophageal varices?
A

Recognizing hemorrhage

Controlling blood pressure

Encouraging nutritional intake

Teaching the client about varices

Question 45
The nurse is reviewing the medication record of a client with acute gastritis. Which
medication, if noted on the clients record, would the nurse question?
A

Digoxin (Lanoxin)

Indomethacin (Indocin)

Furosemide (Lasix)

Propranolol hydrochloride (Inderal)

Question 46
A client with a peptic ulcer reports epigastric pain that frequently awakens her at
night, a feeling of fullness in the abdomen, and a feeling of anxiety about her
health. Based on this information, which nursing diagnosis would be most
appropriate?
A Imbalanced Nutrition: Less than Body Requirements related to anorexia
B Disturbed Sleep Pattern related to epigastric pain
C Ineffective Coping related to exacerbation of duodenal ulcer
D Activity Intolerance related to abdominal pain
Question 47
When counseling a client in ways to prevent cholecystitis, which of the following
guidelines is most important?
A

Eat a low-protein diet

Eat a low-fat, low-cholesterol diet

Limit exercise to 10 minutes/day

Keep weight proportionate to height

Question 48

A female client complains of gnawing epigastric pain for a few hours after meals. At
times, when the pain is severe, vomiting occurs. Specific tests are indicated to rule
out:
A

Cancer of the stomach

Peptic ulcer disease

Chronic gastritis

Pylorospasm

Question 49
Risk factors for the development of hiatal hernias are those that lead to increased
abdominal pressure. Which of the following complications can cause increased
abdominal pressure?
A

Obesity

Volvulus

Constipation

Intestinal obstruction

Question 50
The pain of a duodenal ulcer can be distinguished from that of a gastric ulcer by
which of the following characteristics?
A

Early satiety

Pain on eating

Dull upper epigastric pain

Pain on empty stomach

Once you are finished, click the button below. Any items you have not completed
will be marked incorrect.

Practice Mode: This is an interactive version of the Text Mode. All questions are given in a
single page and correct answers, rationales or explanations (if any) are immediately shown after
you have selected an answer.

NCLEX Exam: Gastrointestinal Disorders 3 (50 Items)


Question 1
Which of the following conditions can cause a hiatal hernia?

Increased intrathoracic pressure

Weakness of the esophageal muscle

Increased esophageal muscle pressure

Weakness of the diaphragmic muscle

Question 2
Risk factors for the development of hiatal hernias are those that lead to increased
abdominal pressure. Which of the following complications can cause increased
abdominal pressure?
A

Obesity

Volvulus

Constipation

Intestinal obstruction

Question 3
Which of the following symptoms is common with a hiatal hernia?
A

Left arm pain

Lower back pain

Esophageal reflux

Abdominal cramping

Question 4
Which of the following tests can be performed to diagnose a hiatal hernia?
A

Colonoscopy

Lower GI series

Barium swallow

Abdominal x-rays

Question 5
Which of the following measures should the nurse focus on for the client with
esophageal varices?
A

Recognizing hemorrhage

Controlling blood pressure

Encouraging nutritional intake

Teaching the client about varices

Question 6
Which of the following tests can be used to diagnose ulcers?
A

Abdominal x-ray

Barium swallow

Computed tomography (CT) scan

Esophagogastroduodenoscopy (EGD)

Question 7
Which of the following best describes the method of action of medications, such as
ranitidine (Zantac), which are used in the treatment of peptic ulcer disease?
A

Neutralize acid

Reduce acid secretions

Stimulate gastrin release

Protect the mucosal barrier

Question 8
The hospitalized client with GERD is complaining of chest discomfort that feels like
heartburn following a meal. After administering an ordered antacid, the nurse
encourages the client to lie in which of the following positions?
A Supine with the head of the bed flat
B On the stomach with the head flat
C On the left side with the head of the bed elevated 30 degrees
D On the right side with the head of the bed elevated 30 degrees
Question 9
The nurse is caring for a client following a Billroth II procedure. On review of the
post-operative orders, which of the following, if prescribed, would the nurse
question and verify?

Irrigating the nasogastric tube

Coughing a deep breathing exercises

Leg exercises

Early ambulation

Question 10
The nurse is providing discharge instructions to a client following gastrectomy.
Which measure will the nurse instruct the client to follow to assist in preventing
dumping syndrome?
A

Eat high-carbohydrate foods

Limit the fluids taken with meals

Ambulate following a meal

Sit in a high-Fowlers position during meals

Question 11
The nurse instructs the nursing assistant on how to provide oral hygiene for a client
who cannot perform this task for himself. Which of the following techniques should
the nurse tell the assistant to incorporate into the clients daily care?
A Assess the oral cavity each time mouth care is given and record observations
B Use a soft toothbrush to brush the clients teeth after each meal
C

Swab the clients tongue, gums, and lips with a soft foam applicator every 2
hours

D Rinse the clients mouth with mouthwash several times a day


Question 12
A client with suspected gastric cancer undergoes an endoscopy of the stomach.
Which of the following assessments made after the procedure would indicate the
development of a potential complication?
A The client complains of a sore throat
B The client displays signs of sedation
C The client experiences a sudden increase in temperature
D The client demonstrates a lack of appetite

Question 13
A client has been diagnosed with adenocarcinoma of the stomach and is scheduled
to undergo a subtotal gastrectomy (Billroth II procedure). During pre-operative
teaching, the nurse is reinforcing information about the procedure. Which of the
following explanations is most accurate?
A The procedure will result in enlargement of the pyloric sphincter
B The procedure will result in anastomosis of the gastric stump to the jejunum
C The procedure will result in removal of the duodenum
D The procedure will result in repositioning of the vagus nerve
Question 14
After a subtotal gastrectomy, the nurse should anticipate that nasogastric tube
drainage will be what color for about 12 to 24 hours after surgery?
A

Dark brown

Bile green

Bright red

Cloudy white

Question 15
After a subtotal gastrectomy, care of the clients nasogastric tube and drainage
system should include which of the following nursing interventions?
A Irrigate the tube with 30 ml of sterile water every hour, if needed
B Reposition the tube if it is not draining well
C Monitor the client for N/V, and abdominal distention
D Turn the machine to high suction of the drainage is sluggish on low suction
Question 16
Which of the following would be an expected nutritional outcome for a client who
has undergone a subtotal gastrectomy for cancer?
A Regain weight loss within 1 month after surgery
B Resume normal dietary intake of three meals per day
C Control nausea and vomiting through regular use of antiemetics

D Achieve optimal nutritional status through oral or parenteral feedings


Question 17
The client with GERD complains of a chronic cough. The nurse understands that in a
client with GERD this symptom may be indicative of which of the following
conditions?
A

Development of laryngeal cancer

Irritation of the esophagus

Esophageal scar tissue formation

Aspiration of gastric contents

Question 18
Which of the following dietary measures would be useful in preventing esophageal
reflux?
A

Eating small, frequent meals

Increasing fluid intake

Avoiding air swallowing with meals

Adding a bedtime snack to the dietary plan

Question 19
A client is admitted to the hospital after vomiting bright red blood and is diagnosed
with a bleeding duodenal ulcer. The client develops a sudden, sharp pain in the mid
epigastric area along with a rigid, board-like abdomen. These clinical manifestations
most likely indicate which of the following?
A

An intestinal obstruction has developed

Additional ulcers have developed

The esophagus has become inflamed

The ulcer has perforated

Question 20
When obtaining a nursing history on a client with a suspected gastric ulcer, which
signs and symptoms would the nurse expect to see? Select all that apply.

Epigastric pain at night

Relief of epigastric pain after eating

Vomiting

Weight loss

Question 21
The nurse is caring for a client who has had a gastroscopy. Which of the following
symptoms may indicate that the client is developing a complication related to the
procedure? Select all that apply.
A

The client complains of a sore throat

The client has a temperature of 100*F

The client appears drowsy following the procedure

The client complains of epigastric pain

The client experiences hematemesis

Question 22
A client with peptic ulcer disease tells the nurse that he has black stools, which he
has not reported to his physician. Based on this information, which nursing
diagnosis would be appropriate for this client?
A Ineffective coping related to fear of diagnosis of chronic illness
B Deficient knowledge related to unfamiliarity with significant signs and symptoms
C Constipation related to decreased gastric motility
D Imbalanced nutrition: Less than body requirements due to gastric bleeding
Question 23
A client with a peptic ulcer reports epigastric pain that frequently awakens her at
night, a feeling of fullness in the abdomen, and a feeling of anxiety about her
health. Based on this information, which nursing diagnosis would be most
appropriate?
A Imbalanced Nutrition: Less than Body Requirements related to anorexia
B Disturbed Sleep Pattern related to epigastric pain
C Ineffective Coping related to exacerbation of duodenal ulcer

D Activity Intolerance related to abdominal pain


Question 24
While caring for a client with peptic ulcer disease, the client reports that he has
been nauseated most of the day and is now feeling lightheaded and dizzy. Based
upon these findings, which nursing actions would be most appropriate for the nurse
to take? Select all that apply.
A

Administering an antacid hourly until nausea subsides.

Monitoring the clients vital signs

Notifying the physician of the clients symptoms

D Initiating oxygen therapy


E

Reassessing the client on an hour

Question 25
A client is to take one daily dose of ranitidine (Zantac) at home to treat her peptic
ulcer. The nurse knows that the client understands proper drug administration of
ranitidine when she says that she will take the drug at which of the following times?
A

Before meals

With meals

At bedtime

When pain occurs

Question 26
A client has been taking aluminum hydroxide 30 mL six times per day at home to
treat his peptic ulcer. He tells the nurse that he has been unable to have a bowel
movement for 3 days. Based on this information, the nurse would determine that
which of the following is the most likely cause of the clients constipation?
A The client has not been including enough fiber in his diet
B The client needs to increase his daily exercise
C The client is experiencing a side effect of the aluminum hydroxide.
D The client has developed a gastrointestinal obstruction
Question 27

A client is taking an antacid for treatment of a peptic ulcer. Which of the following
statements best indicates that the client understands how to correctly take the
antacid?
A I should take my antacid before I take my other medications.
B

I need to decrease my intake of fluids so that I dont dilute the effects of my


antacid.

My antacid will be most effective if I take it whenever I experience stomach


pains.

D It is best for me to take my antacid 1 to 3 hours after meals.


Question 28
The nurse is caring for a client with chronic gastritis. The nurse monitors the client,
knowing that this client is at risk for which of the following vitamin deficiencies?
A

Vitamin A

Vitamin B12

Vitamin C

Vitamin E

Question 29
The nurse is reviewing the medication record of a client with acute gastritis. Which
medication, if noted on the clients record, would the nurse question?
A

Digoxin (Lanoxin)

Indomethacin (Indocin)

Furosemide (Lasix)

Propranolol hydrochloride (Inderal)

Question 30
The nurse is assessing a client 24 hours following a cholecystectomy. The nurse
notes that the T-tube has drained 750ml of green-brown drainage. Which nursing
intervention is most appropriate?
A

Notify the physician

Document the findings

Irrigate the T-tube

Clamp the T-tube

Question 31
The nurse provides medication instructions to a client with peptic ulcer disease.
Which statement, if made by the client, indicates the best understanding of the
medication therapy?
A The cimetidine (Tagamet) will cause me to produce less stomach acid.
B Sucralfate (Carafate) will change the fluid in my stomach.
C Antacids will coat my stomach.
D Omeprazole (Prilosec) will coat the ulcer and help it heal.
Question 32
The client with peptic ulcer disease is scheduled for a pyloroplasty. The client asks
the nurse about the procedure. The nurse plans to respond knowing that a
pyloroplasty involves:
A Cutting the vagus nerve
B Removing the distal portion of the stomach
C

Removal of the ulcer and a large portion of the cells that produce hydrochloric
acid

An incision and resuturing of the pylorus to relax the muscle and enlarge the
opening from the stomach to the duodenum

Question 33
A client with a peptic ulcer is scheduled for a vagotomy. The client asks the nurse
about the purpose of this procedure. The nurse tells the client that the procedure:
A

Decreases food absorption in the stomach

Heals the gastric mucosa

Halts stress reactions

Reduces the stimulus to acid secretions

Question 34
The nurse would assess the client experiencing an acute episode of cholecysitis for
pain that is located in the right

A Upper quadrant and radiates to the left scapula and shoulder


B Upper quadrant and radiates to the right scapula and shoulder
C Lower quadrant and radiates to the umbilicus
D Lower quadrant and radiates to the back
Question 35
Which of the following tasks should be included in the immediate postoperative
management of a client who has undergone gastric resection?
A

Monitoring gastric pH to detect complications

Assessing for bowel sounds

Providing nutritional support

Monitoring for symptoms of hemorrhage

Question 36
If a gastric acid perforates, which of the following actions should not be included in
the immediate management of the client?
A

Blood replacement

Antacid administration

Nasogastric tube suction

Fluid and electrolyte replacement

Question 37
Mucosal barrier fortifiers are used in peptic ulcer disease management for which of
the following indications?
A To inhibit mucus production
B To neutralize acid production
C To stimulate mucus production
D To stimulate hydrogen ion diffusion back into the mucosa
Question 38
When counseling a client in ways to prevent cholecystitis, which of the following
guidelines is most important?

Eat a low-protein diet

Eat a low-fat, low-cholesterol diet

Limit exercise to 10 minutes/day

Keep weight proportionate to height

Question 39
Which of the following symptoms best describes Murphys sign?
A Periumbilical ecchymosis exists
B On deep palpitation and release, pain in elicited
C On deep inspiration, pain is elicited and breathing stops
D Abdominal muscles are tightened in anticipation of palpation
Question 40
Which of the following tests is most commonly used to diagnose cholecystitis?
A

Abdominal CT scan

Abdominal ultrasound

Barium swallow

Endoscopy

Question 41
Which of the following factors should be the main focus of nursing management for
a client hospitalized for cholecystitis?
A

Administration of antibiotics

Assessment for complications

Preparation for lithotripsy

Preparation for surgery

Question 42
A client being treated for chronic cholecystitis should be given which of the
following instructions?
A

Increase rest

Avoid antacids

Increase protein in diet

Use anticholinergics as prescribed

Question 43
The client with a duodenal ulcer may exhibit which of the following findings on
assessment?
A

Hematemesis

Malnourishment

Melena

Pain with eating

Question 44
The pain of a duodenal ulcer can be distinguished from that of a gastric ulcer by
which of the following characteristics?
A

Early satiety

Pain on eating

Dull upper epigastric pain

Pain on empty stomach

Question 45
The client has orders for a nasogastric (NG) tube insertion. During the procedure,
instructions that will assist in the insertion would be:
A

Instruct the client to tilt his head back for insertion in the nostril, then flex his
neck for the final insertion

B After insertion into the nostril, instruct the client to extend his neck
C

Introduce the tube with the clients head tilted back, then instruct him to keep his
head upright for final insertion

D Instruct the client to hold his chin down, then back for insertion of the tube
Question 46
The most important pathophysiologic factor contributing to the formation of
esophageal varices is:

Decreased prothrombin formation

Decreased albumin formation by the liver

Portal hypertension

Increased central venous pressure

Question 47
The client being treated for esophageal varices has a Sengstaken-Blakemore tube
inserted to control the bleeding. The most important assessment is for the nurse to:
A

Check that the hemostat is on the bedside

Monitor IV fluids for the shift

Regularly assess respiratory status

Check that the balloon is deflated on a regular basis

Question 48
A female client complains of gnawing epigastric pain for a few hours after meals. At
times, when the pain is severe, vomiting occurs. Specific tests are indicated to rule
out:
A

Cancer of the stomach

Peptic ulcer disease

Chronic gastritis

Pylorospasm

Question 49
When a client has peptic ulcer disease, the nurse would expect a priority
intervention to be:
A

Assisting in inserting a Miller-Abbott tube

Assisting in inserting an arterial pressure line

Inserting a nasogastric tube

Inserting an I.V.

Question 50
A 40-year-old male client has been hospitalized with peptic ulcer disease. He is
being treated with a histamine receptor antagonist (cimetidine), antacids, and diet.

The nurse doing discharge planning will teach him that the action of cimetidine is
to:
A

Reduce gastric acid output

Protect the ulcer surface

Inhibit the production of hydrochloric acid (HCl)

Inhibit vagus nerve stimulation

Once you are finished, click the button below. Any items you have not completed
will be marked incorrect.

In Text Mode: All questions and answers are given for reading and answering at your own pace.
You can also copy this exam and make a print out.
1. Which of the following conditions can cause a hiatal hernia?
1. Increased intrathoracic pressure
2. Weakness of the esophageal muscle
3. Increased esophageal muscle pressure
4. Weakness of the diaphragmatic muscle
2. Risk factors for the development of hiatal hernias are those that lead to increased
abdominal pressure. Which of the following complications can cause increased abdominal
pressure?
1. Obesity
2. Volvulus
3. Constipation
4. Intestinal obstruction
3. Which of the following symptoms is common with a hiatal hernia?
1. Left arm pain
2. Lower back pain
3. Esophageal reflux
4. Abdominal cramping
4. Which of the following tests can be performed to diagnose a hiatal hernia?
1. Colonoscopy
2. Lower GI series

3. Barium swallow
4. Abdominal x-rays
5. Which of the following measures should the nurse focus on for the client with esophageal
varices?
1. Recognizing hemorrhage
2. Controlling blood pressure
3. Encouraging nutritional intake
4. Teaching the client about varices
6. Which of the following tests can be used to diagnose ulcers?
1. Abdominal x-ray
2. Barium swallow
3. Computed tomography (CT) scan
4. Esophagogastroduodenoscopy (EGD)
7. Which of the following best describes the method of action of medications, such as
ranitidine (Zantac), which are used in the treatment of peptic ulcer disease?
1. Neutralize acid
2. Reduce acid secretions
3. Stimulate gastrin release
4. Protect the mucosal barrier
8. The hospitalized client with GERD is complaining of chest discomfort that feels like
heartburn following a meal. After administering an ordered antacid, the nurse encourages
the client to lie in which of the following positions?
1. Supine with the head of the bed flat
2. On the stomach with the head flat
3. On the left side with the head of the bed elevated 30 degrees
4. On the right side with the head of the bed elevated 30 degrees.
9. The nurse is caring for a client following a Billroth II procedure. On review of the postoperative orders, which of the following, if prescribed, would the nurse question and
verify?
1. Irrigating the nasogastric tube
2. Coughing a deep breathing exercises

3. Leg exercises
4. Early ambulation
10. The nurse is providing discharge instructions to a client following gastrectomy. Which
measure will the nurse instruct the client to follow to assist in preventing dumping
syndrome?
1. Eat high-carbohydrate foods
2. Limit the fluids taken with meals
3. Ambulate following a meal
4. Sit in a high-Fowlers position during meals
11. The nurse instructs the nursing assistant on how to provide oral hygiene for a client
who cannot perform this task for himself. Which of the following techniques should the
nurse tell the assistant to incorporate into the clients daily care?
1. Assess the oral cavity each time mouth care is given and record observations
2. Use a soft toothbrush to brush the clients teeth after each meal
3. Swab the clients tongue, gums, and lips with a soft foam applicator every 2 hours.
4. Rinse the clients mouth with mouthwash several times a day.
12. A client with suspected gastric cancer undergoes an endoscopy of the stomach. Which of
the following assessments made after the procedure would indicate the development of a
potential complication?
1. The client complains of a sore throat
2. The client displays signs of sedation
3. The client experiences a sudden increase in temperature
4. The client demonstrates a lack of appetite
13. A client has been diagnosed with adenocarcinoma of the stomach and is scheduled to
undergo a subtotal gastrectomy (Billroth II procedure). During pre-operative teaching, the
nurse is reinforcing information about the procedure. Which of the following explanations
is most accurate?
1. The procedure will result in enlargement of the pyloric sphincter
2. The procedure will result in anastomosis of the gastric stump to the jejunum
3. The procedure will result in removal of the duodenum
4. The procedure will result in repositioning of the vagus nerve
14. After a subtotal gastrectomy, the nurse should anticipate that nasogastric tube drainage
will be what color for about 12 to 24 hours after surgery?

1. Dark brown
2. Bile green
3. Bright red
4. Cloudy white
15. After a subtotal gastrectomy, care of the clients nasogastric tube and drainage system
should include which of the following nursing interventions?
1. Irrigate the tube with 30 ml of sterile water every hour, if needed.
2. Reposition the tube if it is not draining well
3. Monitor the client for N/V, and abdominal distention
4. Turn the machine to high suction of the drainage is sluggish on low suction.
16. Which of the following would be an expected nutritional outcome for a client who has
undergone a subtotal gastrectomy for cancer?
1. Regain weight loss within 1 month after surgery
2. Resume normal dietary intake of three meals per day
3. Control nausea and vomiting through regular use of antiemetics
4. Achieve optimal nutritional status through oral or parenteral feedings
17. The client with GERD complains of a chronic cough. The nurse understands that in a
client with GERD this symptom may be indicative of which of the following conditions?
1. Development of laryngeal cancer
2. Irritation of the esophagus
3. Esophageal scar tissue formation
4. Aspiration of gastric contents
18. Which of the following dietary measures would be useful in preventing esophageal
reflux?
1. Eating small, frequent meals
2. Increasing fluid intake
3. Avoiding air swallowing with meals
4. Adding a bedtime snack to the dietary plan
19. A client is admitted to the hospital after vomiting bright red blood and is diagnosed with
a bleeding duodenal ulcer. The client develops a sudden, sharp pain in the mid epigastric
area along with a rigid, board-like abdomen. These clinical manifestations most likely
indicate which of the following?

1. An intestinal obstruction has developed


2. Additional ulcers have developed
3. The esophagus has become inflamed
4. The ulcer has perforated
20. When obtaining a nursing history on a client with a suspected gastric ulcer, which signs
and symptoms would the nurse expect to see? Select all that apply.
1. Epigastric pain at night
2. Relief of epigastric pain after eating
3. Vomiting
4. Weight loss
21. The nurse is caring for a client who has had a gastroscopy. Which of the following
symptoms may indicate that the client is developing a complication related to the
procedure? Select all that apply.
1. The client complains of a sore throat
2. The client has a temperature of 100*F
3. The client appears drowsy following the procedure
4. The client complains of epigastric pain
5. The client experiences hematemesis
22. A client with peptic ulcer disease tells the nurse that he has black stools, which he has
not reported to his physician. Based on this information, which nursing diagnosis would be
appropriate for this client?
1. Ineffective coping related to fear of diagnosis of chronic illness
2. Deficient knowledge related to unfamiliarity with significant signs and symptoms
3. Constipation related to decreased gastric motility
4. Imbalanced nutrition: Less than body requirements due to gastric bleeding
23. A client with a peptic ulcer reports epigastric pain that frequently awakens her at night,
a feeling of fullness in the abdomen, and a feeling of anxiety about her health. Based on this
information, which nursing diagnosis would be most appropriate?
1. Imbalanced Nutrition: Less than Body Requirements related to anorexia.
2. Disturbed Sleep Pattern related to epigastric pain
3. Ineffective Coping related to exacerbation of duodenal ulcer
4. Activity Intolerance related to abdominal pain

24. While caring for a client with peptic ulcer disease, the client reports that he has been
nauseated most of the day and is now feeling lightheaded and dizzy. Based upon these
findings, which nursing actions would be most appropriate for the nurse to take? Select all
that apply.
1. Administering an antacid hourly until nausea subsides.
2. Monitoring the clients vital signs
3. Notifying the physician of the clients symptoms
4. Initiating oxygen therapy
5. Reassessing the client on an hour
25. A client is to take one daily dose of ranitidine (Zantac) at home to treat her peptic ulcer.
The nurse knows that the client understands proper drug administration of ranitidine
when she says that she will take the drug at which of the following times?
1. Before meals
2. With meals
3. At bedtime
4. When pain occurs
26. A client has been taking aluminum hydroxide 30 mL six times per day at home to treat
his peptic ulcer. He tells the nurse that he has been unable to have a bowel movement for 3
days. Based on this information, the nurse would determine that which of the following is
the most likely cause of the clients constipation?
1. The client has not been including enough fiber in his diet
2. The client needs to increase his daily exercise
3. The client is experiencing a side effect of the aluminum hydroxide.
4. The client has developed a gastrointestinal obstruction.
27. A client is taking an antacid for treatment of a peptic ulcer. Which of the following
statements best indicates that the client understands how to correctly take the antacid?
1. I should take my antacid before I take my other medications.
2. I need to decrease my intake of fluids so that I dont dilute the effects of my antacid.
3. My antacid will be most effective if I take it whenever I experience stomach pains.
4. It is best for me to take my antacid 1 to 3 hours after meals.
28. The nurse is caring for a client with chronic gastritis. The nurse monitors the client,
knowing that this client is at risk for which of the following vitamin deficiencies?

1. Vitamin A
2. Vitamin B12
3. Vitamin C
4. Vitamin E
29. The nurse is reviewing the medication record of a client with acute gastritis. Which
medication, if noted on the clients record, would the nurse question?
1. Digoxin (Lanoxin)
2. Indomethacin (Indocin)
3. Furosemide (Lasix)
4. Propranolol hydrochloride (Inderal)
30. The nurse is assessing a client 24 hours following a cholecystectomy. The nurse notes
that the T-tube has drained 750ml of green-brown drainage. Which nursing intervention is
most appropriate?
1. Notify the physician
2. Document the findings
3. Irrigate the T-tube
4. Clamp the T-tube
31. The nurse provides medication instructions to a client with peptic ulcer disease. Which
statement, if made by the client, indicates the best understanding of the medication
therapy?
1. The cimetidine (Tagamet) will cause me to produce less stomach acid.
2. Sucralfate (Carafate) will change the fluid in my stomach.
3. Antacids will coat my stomach.
4. Omeprazole (Prilosec) will coat the ulcer and help it heal.
32. The client with peptic ulcer disease is scheduled for a pyloroplasty. The client asks the
nurse about the procedure. The nurse plans to respond knowing that a pyloroplasty
involves:
1. Cutting the vagus nerve
2. Removing the distal portion of the stomach
3. Removal of the ulcer and a large portion of the cells that produce hydrochloric acid
4. An incision and resuturing of the pylorus to relax the muscle and enlarge the opening from the
stomach to the duodenum.

33. A client with a peptic ulcer is scheduled for a vagotomy. The client asks the nurse about
the purpose of this procedure. The nurse tells the client that the procedure:
1. Decreases food absorption in the stomach
2. Heals the gastric mucosa
3. Halts stress reactions
4. Reduces the stimulus to acid secretions
34. The nurse would assess the client experiencing an acute episode of cholecystitis for
pain that is located in the right
1. Upper quadrant and radiates to the left scapula and shoulder
2. Upper quadrant and radiates to the right scapula and shoulder
3. Lower quadrant and radiates to the umbilicus
4. Lower quadrant and radiates to the back
35. Which of the following tasks should be included in the immediate postoperative
management of a client who has undergone gastric resection?
1. Monitoring gastric pH to detect complications
2. Assessing for bowel sounds
3. Providing nutritional support
4. Monitoring for symptoms of hemorrhage
36. If a gastric acid perforates, which of the following actions should not be included in the
immediate management of the client?
1. Blood replacement
2. Antacid administration
3. Nasogastric tube suction
4. Fluid and electrolyte replacement
37. Mucosal barrier fortifiers are used in peptic ulcer disease management for which of the
following indications?
1. To inhibit mucus production
2. To neutralize acid production
3. To stimulate mucus production
4. To stimulate hydrogen ion diffusion back into the mucosa
38. When counseling a client in ways to prevent cholecystitis, which of the following
guidelines is most important?

1. Eat a low-protein diet


2. Eat a low-fat, low-cholesterol diet
3. Limit exercise to 10 minutes/day
4. Keep weight proportionate to height
39. Which of the following symptoms best describes Murphys sign?
1. Periumbilical ecchymosis exists
2. On deep palpation and release, pain in elicited
3. On deep inspiration, pain is elicited and breathing stops
4. Abdominal muscles are tightened in anticipation of palpation
40. Which of the following tests is most commonly used to diagnose cholecystitis?
1. Abdominal CT scan
2. Abdominal ultrasound
3. Barium swallow
4. Endoscopy
41. Which of the following factors should be the main focus of nursing management for a
client hospitalized for cholecystitis?
1. Administration of antibiotics
2. Assessment for complications
3. Preparation for lithotripsy
4. Preparation for surgery
42. A client being treated for chronic cholecystitis should be given which of the following
instructions?
1. Increase rest
2. Avoid antacids
3. Increase protein in diet
4. Use anticholinergics as prescribed
43. The client with a duodenal ulcer may exhibit which of the following findings on
assessment?
1. Hematemesis
2. Malnourishment
3. Melena
4. Pain with eating

44. The pain of a duodenal ulcer can be distinguished from that of a gastric ulcer by which
of the following characteristics?
1. Early satiety
2. Pain on eating
3. Dull upper epigastric pain
4. Pain on empty stomach
45. The client has orders for a nasogastric (NG) tube insertion. During the procedure,
instructions that will assist in the insertion would be:
1. Instruct the client to tilt his head back for insertion in the nostril, then flex his neck for the
final insertion
2. After insertion into the nostril, instruct the client to extend his neck
3. Introduce the tube with the clients head tilted back, then instruct him to keep his head upright
for final insertion
4. Instruct the client to hold his chin down, then back for insertion of the tube
46. The most important pathophysiologic factor contributing to the formation of
esophageal varices is:
1. Decreased prothrombin formation
2. Decreased albumin formation by the liver
3. Portal hypertension
4. Increased central venous pressure
47. The client being treated for esophageal varices has a Sengstaken-Blakemore tube
inserted to control the bleeding. The most important assessment is for the nurse to:
1. Check that the hemostat is on the bedside
2. Monitor IV fluids for the shift
3. Regularly assess respiratory status
4. Check that the balloon is deflated on a regular basis
48. A female client complains of gnawing epigastric pain for a few hours after meals. At
times, when the pain is severe, vomiting occurs. Specific tests are indicated to rule out:
1. Cancer of the stomach
2. Peptic ulcer disease
3. Chronic gastritis
4. Pylorospasm

49. When a client has peptic ulcer disease, the nurse would expect a priority intervention to
be:
1. Assisting in inserting a Miller-Abbott tube
2. Assisting in inserting an arterial pressure line
3. Inserting a nasogastric tube
4. Inserting an I.V.
50. A 40-year-old male client has been hospitalized with peptic ulcer disease. He is being
treated with a histamine receptor antagonist (cimetidine), antacids, and diet. The nurse
doing discharge planning will teach him that the action of cimetidine is to:
1. Reduce gastric acid output
2. Protect the ulcer surface
3. Inhibit the production of hydrochloric acid (HCl)
4. Inhibit vagus nerve stimulation
Answers and Rationale

1. Answer: 4. Weakness of the diaphragmatic muscle


A hiatal hernia is caused by weakness of the diaphragmatic muscle and increased intraabdominalnot intrathoracicpressure. This weakness allows the stomach to slide into the
esophagus. The esophageal supports weaken, but esophageal muscle weakness or increased
esophageal muscle pressure isnt a factor in hiatal hernia.
2. Answer: 1. Obesity
Obesity may cause increased abdominal pressure that pushes the lower portion of the stomach
into the thorax.
3. Answer: 3. Esophageal reflux
Esophageal reflux is a common symptom of hiatal hernia. This seems to be associated with
chronic exposure of the lower esophageal sphincter to the lower pressure of the thorax, making it
less effective.
4. Answer: 3. Barium swallow
A barium swallow with fluoroscopy shows the position of the stomach in relation to the
diaphragm. A colonoscopy and a lower GI series show disorders of the intestine.

5. Answer: 1. Recognizing hemorrhage


Recognizing the rupture of esophageal varices, or hemorrhage, is the focus of nursing care
because the client could succumb to this quickly. Controlling blood pressure is also important
because it helps reduce the risk of variceal rupture. It is also important to teach the client what
varices are and what foods he should avoid such as spicy foods.
6. Answer: 4. Esophagogastroduodenoscopy (EGD)
The EGD can visualize the entire upper GI tract as well as allow for tissue specimens and
electrocautery if needed. The barium swallow could locate a gastric ulcer. A CT scan and an
abdominal x-ray arent useful in the diagnosis of an ulcer.
7. Answer: 2. Reduce acid secretions
Ranitidine is a histamine-2 receptor antagonist that reduces acid secretion by inhibiting gastrin
secretion.
8. Answer: 3. On the left side with the head of the bed elevated 30 degrees
The discomfort of reflux is aggravated by positions that compress the abdomen and the stomach.
These include lying flat on the back or on the stomach after a meal of lying on the right side. The
left side-lying position with the head of the bed elevated is most likely to give relief to the client.
9. Answer: 1. Irrigating the nasogastric tube
In a Billroth II procedure the proximal remnant of the stomach is anastomosed to the proximal
jejunum. Patency of the NG tube is critical for preventing the retention of gastric secretions. The
nurse should never irrigate or reposition the gastric tube after gastric surgery, unless specifically
ordered by the physician. In this situation, the nurse would clarify the order.
10. Answer: 2. Limit the fluids taken with meals
The nurse should instruct the client to decrease the amount of fluid taken at meals and to avoid
high carbohydrate foods including fluids such as fruit nectars; to assume a low-Fowlers position
during meals; to lie down for 30 minutes after eating to delay gastric emptying; and to take
antispasmodics as prescribed.
11. Answer: 2. Use a soft toothbrush to brush the clients teeth after each meal
A soft toothbrush should be used to brush the clients teeth after each meal and more often as
needed. Mechanical cleaning is necessary to maintain oral health, simulate gingiva, and remove

plaque. Assessing the oral cavity and recording observations is the responsibility of the nurse, not
the nursing assistant. Swabbing with a safe foam applicator does not provide enough friction to
clean the mouth. Mouthwash can be a drying irritant and is not recommended for frequent use.
12. Answer: 3. The client experiences a sudden increase in temperature
The most likely complication of an endoscopic procedure is perforation. A sudden temperature
spike with 1 to 2 hours after the procedure is indicative of a perforation and should be reported
immediately to the physician. A sore throat is to be anticipated after an endoscopy. Clients are
given sedatives during the procedure, so it is expected that they will display signs of sedation
after the procedure is completed. A lack of appetite could be the result of many factors, including
the disease process.
13. Answer: 2. The procedure will result in anastomosis of the gastric stump to the jejunum
A Billroth II procedure bypasses the duodenum and connects the gastric stump directly to the
jejunum. The pyloric sphincter is removed, along with some of the stomach fundus.
14. Answer: 1. Dark brown
About 12 to 24 hours after a subtotal gastrectomy, gastric drainage is normally brown, which
indicates digested blood. Bile green or cloudy white drainage is not expected during the first 12
to 24 hours after a subtotal gastrectomy. Drainage during the first 6 to 12 hours contains some
bright red blood, but large amounts of blood or excessively bloody drainage should be reported
to the physician promptly.
15. Answer: 3. Monitor the client for N/V, and abdominal distention
Nausea, vomiting, or abdominal distention indicated that gas and secretions are accumulating
within the gastric pouch due to impaired peristalsis or edema at the operative site and may
indicate that the drainage system is not working properly. Saline solution is used to irrigate
nasogastric tubes. Hypotonic solutions such as water increase electrolyte loss. In addition, a
physicians order is needed to irrigate the NG tube, because this procedure could disrupt the
suture line. After gastric surgery, only the surgeon repositions the NG tube because of the danger
of rupturing or dislodging the suture line. The amount of suction varies with the type of tube
used and is ordered by the physician. High suction may create too much tension on the gastric
suture line.
16. Answer: 4. Achieve optimal nutritional status through oral or parenteral feedings
An appropriate expected outcome is for the client to achieve optimal nutritional status through
the use of oral feedings or total parenteral nutrition (TPN). TPN may be used to supplement oral

intake, or it may be used alone if the client cannot tolerate oral feedings. The client would not be
expected to regain lost weight within 1 month after surgery or to tolerate a normal dietary intake
of three meals per day. Nausea and vomiting would not be considered an expected outcome of
gastric surgery, and regular use of antiemetics would not be anticipated.
17. Answer: 4. Aspiration of gastric contents
Clients with GERD can develop pulmonary symptoms such as coughing, wheezing, and dyspnea
that are caused by the aspiration of gastric contents. GERD does not predispose the client to the
development of laryngeal cancer. Irritation of the esophagus and esophageal scar tissue
formation can develop as a result of GERD. However, GERD is more likely to cause painful and
difficult swallowing.
18. Answer: 1. Eating small, frequent meals
Esophageal reflux worsens when the stomach is overdistended with food. Therefore, an
important measure is to eat small, frequent meals. Fluid intake should be decreased during meals
to reduce abdominal distention. Avoiding air swallowing does not prevent esophageal reflux.
Food intake in the evening should be strictly limited to reduce the incidence of nighttime reflux,
so bedtime snacks are not recommended.
19. Answer: 4. The ulcer has perforated
The body reacts to perforation of an ulcer by immobilizing the area as much as possible. This
results in boardlike muscle rigidity, usually with extreme pain. Perforation is a medical
emergency requiring immediate surgical intervention because peritonitis develops quickly after
perforation. An intestinal obstruction would not cause midepigastric pain. Esophageal
inflammation or the development of additional ulcers would not cause a rigid, boardlike
abdomen.
20. Answers: 3 and 4.
Vomiting and weight loss are common with gastric ulcers. Clients with a gastric ulcer are most
likely to complain of a burning epigastric pain that occurs about one hour after eating. Eating
frequently aggravates the pain. Clients with duodenal ulcers are more likely to complain about
pain that occurs during the night and is frequently relieved by eating.
21. Answers: 2, 4, and 5.
Following a gastroscopy, the nurse should monitor the client for complications, which include
perforation and the potential for aspiration. An elevated temperature, complaints of epigastric
pain, or the vomiting of blood (hematemesis) are all indications of a possible perforation and

should be reported promptly. A sore throat is a common occurrence following a gastroscopy.


Clients are usually sedated to decrease anxiety and the nurse would anticipate that the client will
be drowsy following the procedure.
22. Answer: 2. Deficient knowledge related to unfamiliarity with significant signs and
symptoms
Black, tarry stools are an important warning sign of bleeding in peptic ulcer disease. Digested
blood in the stomach causes it to be black. The odor of the stool is very stinky. Clients with
peptic ulcer disease should be instructed to report the incidence of black stools promptly to their
physician.
23. Answer: 2. Disturbed Sleep Pattern related to epigastric pain
Based on the data provided, the most appropriate nursing diagnosis would be Disturbed Sleep
pattern. A client with a duodenal ulcer commonly awakens at night with pain. The clients
feelings of anxiety do not necessarily indicate that she is coping ineffectively.
24. Answers: 2 and 3.
The symptoms of nausea and dizziness in a client with peptic ulcer disease may be indicative of
hemorrhage and should not be ignored. The appropriate nursing actions at this time are for the
nurse to monitor the clients vital signs and notify the physician of the clients symptoms. To
administer an antacid hourly or to wait one hour to reassess the client would be inappropriate;
prompt intervention is essential in a client who is potentially experiencing a gastrointestinal
hemorrhage. The nurse would notify the physician of assessment findings and then initiate
oxygen therapy if ordered by the physician.
25. Answer: 3. At bedtime
Ranitidine blocks secretion of hydrochloric acid. Clients who take only one daily dose of
ranitidine are usually advised to take it at bedtime to inhibit nocturnal secretion of acid. Clients
who take the drug twice a day are advised to take it in the morning and at bedtime.
26. Answer: 3. The client is experiencing a side effect of the aluminum hydroxide.
It is most likely that the client is experiencing a side effect of the antacid. Antacids with
aluminum salt products, such as aluminum hydroxide, form insoluble salts in the body. These
precipitate and accumulate in the intestines, causing constipation. Increasing dietary fiber intake
or daily exercise may be a beneficial lifestyle change for the client but is not likely to relieve the
constipation caused by the aluminum hydroxide. Constipation, in isolation from other symptoms,
is not a sign of bowel obstruction.

27. Answer: 4. It is best for me to take my antacid 1 to 3 hours after meals.


Antacids are most effective if taken 1 to 3 hours after meals and at bedtime. When an antacid is
taken on an empty stomach, the duration of the drugs action is greatly decreased. Taking
antacids 1 to 3 hours after a meal lengthens the duration of action, thus increasing the therapeutic
action of the drug. Antacids should be administered about 2 hours after other medications to
decrease the chance of drug interactions. It is not necessary to decrease fluid intake when taking
antacids.
28. Answer: 2. Vitamin B12
Chronic gastritis causes deterioration and atrophy of the lining of the stomach, leading to the loss
of the functioning parietal cells. The source of the intrinsic factor is lost, which results in the
inability to absorb vitamin B12. This leads to the development of pernicious anemia.
29. Answer: 2. Indomethacin (Indocin)
Indomethacin (Indocin) is a NSAID and can cause ulceration of the esophagus, stomach,
duodenum, or small intestine. Indomethacin is contraindicated in a client with GI disorders.
30. Answer: 2. Document the findings
Following cholecystectomy, drainage from the T-tube is initially bloody and then turns to greenbrown. The drainage is measured as output. The amount of expected drainage will range from
500 to 1000 ml per day. The nurse would document the output.
31. Answer: 1 The cimetidine (Tagamet) will cause me to produce less stomach acid.
Cimetidine (Tagamet), a histamine H2 receptor antagonist, will decrease the secretion of gastric
acid. Sucralfate (Carafate) promotes healing by coating the ulcer. Antacids neutralize acid in the
stomach. Omeprazole (Prilosec) inhibits gastric acid secretion.
32. Answer: 4. An incision and resuturing of the pylorus to relax the muscle and enlarge the
opening from the stomach to the duodenum.
Option 4 describes the procedure for a pyloroplasty. A vagotomy involves cutting the vagus
nerve. A subtotal gastrectomy involves removing the distal portion of the stomach. A Billroth II
procedure involves removal of the ulcer and a large portion of the tissue that produces
hydrochloric acid.
33. Answer: 4. Reduces the stimulus to acid secretions

A vagotomy, or cutting the vagus nerve, is done to eliminate parasympathetic stimulation of


gastric secretion.
34. Answer: 2. Upper quadrant and radiates to the right scapula and shoulder
During an acute gallbladder attack, the client may complain of severe right upper quadrant
pain that radiates to the right scapula and shoulder. This is governed by the pattern on
dermatomes in the body.
35. Answer: 4. Monitoring for symptoms of hemorrhage
The client should be monitored closely for signs and symptoms of hemorrhage, such as bright
red blood in the nasogastric tube suction, tachycardia, or a drop in blood pressure. Gastric pH
may be monitored to evaluate the need for histamine-2 receptor antagonists. Bowel sounds may
not return for up to 72 hours postoperatively. Nutritional needs should be addressed soon after
surgery.
36. Answer: 2. Antacid administration
Antacids arent helpful in perforation. The client should be treated with antibiotics as well as
fluid, electrolyte, and blood replacement. NG tube suction should also be performed to prevent
further spillage of stomach contents into the peritoneal cavity.
37. Answer: 3. To stimulate mucus production
The mucosal barrier fortifiers stimulate mucus production and prevent hydrogen ion diffusion
back into the mucosa, resulting in accelerated ulcer healing. Antacids neutralize acid production.
38. Answer: 4. Keep weight proportionate to height
Obesity is a known cause of gallstones, and maintaining a recommended weight will help protect
against gallstones. Excessive dietary intake of cholesterol is associated with the development of
gallstones in many people. Dietary protein isnt implicated in cholecystitis. Liquid protein and
low-calorie diets (with rapid weight loss of more than 5 lb [2.3kg] per week) are implicated as
the cause of some cases of cholecystitis. Regular exercise (30 minutes/three times a week) may
help reduce weight and improve fat metabolism. Reducing stress may reduce bile production,
which may also indirectly decrease the chances of developing cholecystitis.
39. Answer: 3. On deep inspiration, pain is elicited and breathing stops
Murphys sign is elicited when the client reacts to pain and stops breathing. Its a common
finding in clients with cholecystitis. Periumbilical ecchymosis, Cullens sign, is present in

peritonitis. Pain on deep palpation and release is rebound tenderness. Tightening up abdominal
muscles in anticipation of palpation is guarding.
40. Answer: 2. Abdominal ultrasound
An abdominal ultrasound can show if the gallbladder is enlarged, if gallstones are present, if the
gallbladder wall is thickened, or if distention of the gallbladder lumen is present. An abdominal
CT scan can be used to diagnose cholecystitis, but it usually isnt necessary. A barium swallow
looks at the stomach and the duodenum. Endoscopy looks at the esophagus, stomach, and
duodenum.
41. Answer: 2. Assessment for complications
The client with acute cholecystitis should first be monitored for perforation, fever, abscess,
fistula, and sepsis. After assessment, antibiotics will be administered to reduce the infection.
Lithotripsy is used only for a small percentage of clients. Surgery is usually done after the acute
infection has subsided.
42. Answer: 4. Use anticholinergics as prescribed
Conservative therapy for chronic cholecystitis includes weight reduction by increasing physical
activity, a low-fat diet, antacid use to treat dyspepsia, and anticholinergic use to relax smooth
muscles and reduce ductal tone and spasm, thereby reducing pain.
43. Answer: 3. Melena
The client with a duodenal ulcer may have bleeding at the ulcer site, which shows up as melena
(black tarry stool). The other findings are consistent with a gastric ulcer.
44. Answer: 4. Pain on empty stomach
Pain on empty stomach is relieved by taking foods or antacids. The other symptoms are those of
a gastric ulcer.
45. Answer: 1. Instruct the client to tilt his head back for insertion in the nostril, then flex
his neck for the final insertion
NG insertion technique is to have the client first tilt his head back for insertion into the nostril,
then to flex his neck forward and swallow. Extension of the neck (2) will impede NG tube
insertion.
46. Answer: 3. Portal hypertension

As the liver cells become fatty and degenerate, they are no longer able to accommodate the large
amount of blood necessary for homeostasis. The pressure in the liver increases and causes
increased pressure in the venous system. As the portal pressure increases, fluid exudes into the
abdominal cavity. This is called ascites.
47. Answer: 3. Regularly assess respiratory status
The respiratory system can become occluded if the balloon slips and moves up the esophagus,
putting pressure on the trachea. This would result in respiratory distress and should be assessed
frequently. Scissors should be kept at the bedside to cut the tube if distress occurs. This is a
safety intervention.
48. Answer: 2. Peptic ulcer disease
Peptic ulcer disease is characteristically gnawing epigastric pain that may radiate to the back.
Vomiting usually reflects pyloric spasm from muscular spasm or obstruction. Cancer (1) would
not evidence pain or vomiting unless the pylorus was obstructed.
49. Answer: 3. Inserting a nasogastric tube
An NG tube insertion is the most appropriate intervention because it will determine the presence
of active GI bleeding. A Miller-Abbott tube (1) is a weighted, mercury-filled ballooned tube used
to resolve bowel obstructions. There is no evidence of shock or fluid overload in the client;
therefore, an arterial line (2) is not appropriate at this time and an IV (4) is optional.
50. Answer: 1. Reduce gastric acid output
These drugs inhibit action of histamine on the H2 receptors of parietal cells, thus reducing gastric
acid output.

4. Topics
Included topics in this practice quiz are:

Pancreatitis

Colostomy

Nasogastric Intubation

Vagotomy

Guidelines
Follow the guidelines below to make the most out of this exam:

Read each question carefully and choose the best answer.

You are given one minute per question. Spend your time wisely!

Answers and rationales are given below. Be sure to read them.

If you need more clarifications, please direct them to the comments section.

In Exam Mode: All questions are shown in random and the results, answers and rationales (if
any) will only be given after youve finished the quiz. You are given 1 minute per question, a
total of 30 minutes for this exam.

NCLEX Exam: Gastrointestinal Disorders 4 (30 Items)


29:45
Question 1
The nurse is instructing the male client who has an inguinal hernia repair how to
reduce postoperative swelling following the procedure. What should the nurse tell
the client?
A

Limit oral fluid

Elevate the scrotum

Apply heat to the abdomen

Remain in a low-fiber diet

Question 2
Dr. Smith has determined that the client with hepatitis has contracted the infection
from contaminated food. The nurse understands that this client is most likely
experiencing what type of hepatitis?
A

Hepatitis A

Hepatitis B

Hepatitis C

Hepatitis D

Question 3
The nurse is providing discharge instructions to a male client following gastrectomy
and instructs the client to take which measure to assist in preventing dumping
syndrome?
A

Ambulate following a meal

Eat high carbohydrate foods

Limit the fluid taken with meal

Sit in a high-Fowlers position during meals

Question 4
The nurse is performing a colostomy irrigation on a male client. During the
irrigation, the client begins to complain of abdominal cramps. What is the
appropriate nursing action?
A

Notify the physician

Stop the irrigation temporarily

Increase the height of the irrigation

Medicate for pain and resume the irrigation

Question 5
The nurse is monitoring a female client with a diagnosis of peptic ulcer. Which
assessment findings would most likely indicate perforation of the ulcer?
A

Bradycardia

Numbness in the legs

Nausea and vomiting

A rigid, board-like abdomen

Question 6
A nurse is preparing to care for a female client with esophageal varices who has just
has a Sengstaken-Blakemore tube inserted. The nurse gathers supplies, knowing
that which of the following items must be kept at the bedside at all times?

An obturator

Kelly clamp

An irrigation set

A pair of scissors

Question 7
A nurse is preparing to remove a nasogastric tube from a female client. The nurse
should instruct the client to do which of the following just before the nurse removes
the tube?
A

Exhale

Inhale and exhale quickly

Take and hold a deep breath

Perform a Valsalva maneuver

Question 8
The nurse is teaching a female client how to perform a colostomy irrigation. To
enhance the effectiveness of the irrigation and fecal returns, what measure should
the nurse instruct the client to do?
A

Increase fluid intake

Place heat on the abdomen

Perform the irrigation in the evening

Reduce the amount of irrigation solution

Question 9
Nurse Oliver checks for residual before administering a bolus tube feeding to a
client with a nasogastric tube and obtains a residual amount of 150 mL. What is
appropriate action for the nurse to take?
A Hold the feeding
B Reinstill the amount and continue with administering the feeding
C Elevate the clients head at least 45 degrees and administer the feeding
D Discard the residual amount and proceed with administering the feeding
Question 10

The nurse is caring for a hospitalized female client with a diagnosis of ulcerative
colitis. Which finding, if noted on assessment of the client, would the nurse report to
the physician?
A

Hypotension

Bloody diarrhea

Rebound tenderness

A hemoglobin level of 12 mg/dL

Question 11
Nurse Ryan is assessing for correct placement of a nasogastric tube. The nurse
aspirates the stomach contents and check the contents for pH. The nurse verifies
correct tube placement if which pH value is noted?
A

3.5

7.0

7.35

7.5

Question 12
The nurse is caring for a male client postoperatively following creation of a
colostomy. Which nursing diagnosis should the nurse include in the plan of care?
A

Sexual dysfunction

Body image, disturbed

Fear related to poor prognosis

Nutrition: more than body requirements, imbalanced

Question 13
The nurse is caring for a female client following a Billroth II procedure. Which
postoperative order should the nurse question and verify?
A

Leg exercises

Early ambulation

Irrigating the nasogastric tube

Coughing and deep-breathing exercises

Question 14
The nurse is preparing a discharge teaching plan for the male client who had
umbilical hernia repair. What should the nurse include in the plan?
A

Irrigating the drain

Avoiding coughing

Maintaining bed rest

Restricting pain medication

Question 15
A male client who is recovering from surgery has been advanced from a clear liquid
diet to a full liquid diet. The client is looking forward to the diet change because he
has been bored with the clear liquid diet. The nurse would offer which full liquid
item to the client?
A

Tea

Gelatin

Custard

Popsicle

Question 16
The nurse is reviewing the medication record of a female client with acute gastritis.
Which medication, if noted on the clients record, would the nurse question?
A

Digoxin (Lanoxin)

Furosemide (Lasix)

Indomethacin (Indocin)

Propranolol hydrochloride (Inderal)

Question 17
The nurse is reviewing the physicians orders written for a male client admitted to
the hospital with acute pancreatitis. Which physician order should the nurse
question if noted on the clients chart?

NPO status

Nasogastric tube inserted

Morphine sulfate for pain

An anticholinergic medication

Question 18
The nurse is caring for a male client with a diagnosis of chronic gastritis. The nurse
monitors the client knowing that this client is at risk for which vitamin deficiency?
A

Vitamin A

Vitamin B12

Vitamin C

Vitamin E

Question 19
Nurse Berlinda is assigned to a 41-year-old client who has a diagnosis of chronic
pancreatitis. The nurse reviews the laboratory result, anticipating a laboratory
report that indicates a serum amylase level of:
A

45 units/L

100 units/L

300 units/L

500 units/L

Question 20
A client is suspected of having hepatitis. Which diagnostic test result will assist in
confirming this diagnosis?
A

Elevated hemoglobin level

Elevated serum bilirubin level

Elevated blood urea nitrogen level

Decreased erythrocyte sedimentation rate

Question 21

A male client with a peptic ulcer is scheduled for a vagotomy and the client asks the
nurse about the purpose of this procedure. Which response by the nurse best
describes the purpose of a vagotomy?
A

Halts stress reactions

Heals the gastric mucosa

Reduces the stimulus to acid secretions

Decreases food absorption in the stomach

Question 22
Nurse Juvy is caring for a client with cirrhosis of the liver. To minimize the effects of
the disorder, the nurse teaches the client about foods that are high in thiamine. The
nurse determines that the client has the best understanding of the dietary
measures to follow if the client states an intention to increase the intake of:
A

Pork

Milk

Chicken

Broccoli

Question 23
A nurse is inserting a nasogastric tube in an adult male client. During the procedure,
the client begins to cough and has difficulty breathing. Which of the following is the
appropriate nursing action?
A Quickly insert the tube
B Notify the physician immediately
C Remove the tube and reinsert when the respiratory distress subsides
D Pull back on the tube and wait until the respiratory distress subsides
Question 24
The nurse is reviewing the record of a female client with Crohns disease. Which
stool characteristics should the nurse expect to note documented in the clients
record?
A

Diarrhea

Chronic constipation

Constipation alternating with diarrhea

Stools constantly oozing from the rectum

Question 25
Polyethylene glycol-electrolyte solution (GoLYTELY) is prescribed for the female
client scheduled for a colonoscopy. The client begins to experience diarrhea
following administration of the solution. What action by the nurse is appropriate?
A

Start an IV infusion

Administer an enema

Cancel the diagnostic test

Explain that diarrhea is expected

Question 26
Nurse Joy is preparing to administer medication through a nasogastric tube that is
connected to suction. To administer the medication, the nurse would:
A Position the client supine to assist in medication absorption
B Aspirate the nasogastric tube after medication administration to maintain patency
C

Clamp the nasogastric tube for 30 minutes following administration of the


medication

Change the suction setting to low intermittent suction for 30 minutes after
medication administration

Question 27
The nurse is monitoring a female client for the early signs and symptoms of
dumping syndrome. Which of the following indicate this occurrence?
A

Sweating and pallor

Bradycardia and indigestion

Double vision and chest pain

Abdominal cramping and pain

Question 28
The nurse is performing an abdominal assessment and inspects the skin of the
abdomen. The nurse performs which assessment technique next?

Palpates the abdomen for size

Palpates the liver at the right rib margin

Listens to bowel sounds in all four quadrants

Percusses the right lower abdominal quadrant

Question 29
A female client being seen in a physicians office has just been scheduled for a
barium swallow the next day. The nurse writes down which instruction for the client
to follow before the test?
A

Fast for 8 hours before the test

Eat a regular supper and breakfast

Continue to take all oral medications as scheduled

D Monitor own bowel movement pattern for constipation


Question 30
The nurse is assessing a male client 24 hours following a cholecystectomy. The
nurse noted that the T tube has drained 750 mL of green-brown drainage since the
surgery. Which nursing intervention is appropriate?
A

Clamp the T tube

Irrigate the T tube

Notify the physician

Document the findings

Once you are finished, click the button below. Any items you have not completed
will be marked incorrect.

Practice Mode: This is an interactive version of the Text Mode. All questions are given in a
single page and correct answers, rationales or explanations (if any) are immediately shown after
you have selected an answer.

NCLEX Exam: Gastrointestinal Disorders 4 (30 Items)


Question 1
Nurse Berlinda is assigned to a 41-year-old client who has a diagnosis of chronic
pancreatitis. The nurse reviews the laboratory result, anticipating a laboratory
report that indicates a serum amylase level of:

45 units/L

100 units/L

300 units/L

500 units/L

Question 2
A male client who is recovering from surgery has been advanced from a clear liquid
diet to a full liquid diet. The client is looking forward to the diet change because he
has been bored with the clear liquid diet. The nurse would offer which full liquid
item to the client?
A

Tea

Gelatin

Custard

Popsicle

Question 3
Nurse Juvy is caring for a client with cirrhosis of the liver. To minimize the effects of
the disorder, the nurse teaches the client about foods that are high in thiamine. The
nurse determines that the client has the best understanding of the dietary
measures to follow if the client states an intention to increase the intake of:
A

Pork

Milk

Chicken

Broccoli

Question 4
Nurse Oliver checks for residual before administering a bolus tube feeding to a
client with a nasogastric tube and obtains a residual amount of 150 mL. What is
appropriate action for the nurse to take?
A Hold the feeding
B Reinstill the amount and continue with administering the feeding
C Elevate the clients head at least 45 degrees and administer the feeding
D Discard the residual amount and proceed with administering the feeding

Question 5
A nurse is inserting a nasogastric tube in an adult male client. During the procedure,
the client begins to cough and has difficulty breathing. Which of the following is the
appropriate nursing action?
A Quickly insert the tube
B Notify the physician immediately
C Remove the tube and reinsert when the respiratory distress subsides
D Pull back on the tube and wait until the respiratory distress subsides
Question 6
Nurse Ryan is assessing for correct placement of a nasogastric tube. The nurse
aspirates the stomach contents and check the contents for pH. The nurse verifies
correct tube placement if which pH value is noted?
A

3.5

7.0

7.35

7.5

Question 7
A nurse is preparing to remove a nasogastric tube from a female client. The nurse
should instruct the client to do which of the following just before the nurse removes
the tube?
A

Exhale

Inhale and exhale quickly

Take and hold a deep breath

Perform a Valsalva maneuver

Question 8
Nurse Joy is preparing to administer medication through a nasogastric tube that is
connected to suction. To administer the medication, the nurse would:
A Position the client supine to assist in medication absorption

B Aspirate the nasogastric tube after medication administration to maintain patency


C

Clamp the nasogastric tube for 30 minutes following administration of the


medication

Change the suction setting to low intermittent suction for 30 minutes after
medication administration

Question 9
A nurse is preparing to care for a female client with esophageal varices who has just
has a Sengstaken-Blakemore tube inserted. The nurse gathers supplies, knowing
that which of the following items must be kept at the bedside at all times?
A

An obturator

Kelly clamp

An irrigation set

A pair of scissors

Question 10
Dr. Smith has determined that the client with hepatitis has contracted the infection
from contaminated food. The nurse understands that this client is most likely
experiencing what type of hepatitis?
A

Hepatitis A

Hepatitis B

Hepatitis C

Hepatitis D

Question 11
A client is suspected of having hepatitis. Which diagnostic test result will assist in
confirming this diagnosis?
A

Elevated hemoglobin level

Elevated serum bilirubin level

Elevated blood urea nitrogen level

Decreased erythrocyte sedimentation rate

Question 12

The nurse is reviewing the physicians orders written for a male client admitted to
the hospital with acute pancreatitis. Which physician order should the nurse
question if noted on the clients chart?
A

NPO status

Nasogastric tube inserted

Morphine sulfate for pain

An anticholinergic medication

Question 13
A female client being seen in a physicians office has just been scheduled for a
barium swallow the next day. The nurse writes down which instruction for the client
to follow before the test?
A

Fast for 8 hours before the test

Eat a regular supper and breakfast

Continue to take all oral medications as scheduled

D Monitor own bowel movement pattern for constipation


Question 14
The nurse is performing an abdominal assessment and inspects the skin of the
abdomen. The nurse performs which assessment technique next?
A

Palpates the abdomen for size

Palpates the liver at the right rib margin

Listens to bowel sounds in all four quadrants

Percusses the right lower abdominal quadrant

Question 15
Polyethylene glycol-electrolyte solution (GoLYTELY) is prescribed for the female
client scheduled for a colonoscopy. The client begins to experience diarrhea
following administration of the solution. What action by the nurse is appropriate?
A

Start an IV infusion

Administer an enema

Cancel the diagnostic test

Explain that diarrhea is expected

Question 16
The nurse is caring for a male client with a diagnosis of chronic gastritis. The nurse
monitors the client knowing that this client is at risk for which vitamin deficiency?
A

Vitamin A

Vitamin B12

Vitamin C

Vitamin E

Question 17
The nurse is reviewing the medication record of a female client with acute gastritis.
Which medication, if noted on the clients record, would the nurse question?
A

Digoxin (Lanoxin)

Furosemide (Lasix)

Indomethacin (Indocin)

Propranolol hydrochloride (Inderal)

Question 18
The nurse is assessing a male client 24 hours following a cholecystectomy. The
nurse noted that the T tube has drained 750 mL of green-brown drainage since the
surgery. Which nursing intervention is appropriate?
A

Clamp the T tube

Irrigate the T tube

Notify the physician

Document the findings

Question 19
The nurse is monitoring a female client with a diagnosis of peptic ulcer. Which
assessment findings would most likely indicate perforation of the ulcer?
A

Bradycardia

Numbness in the legs

Nausea and vomiting

A rigid, board-like abdomen

Question 20
A male client with a peptic ulcer is scheduled for a vagotomy and the client asks the
nurse about the purpose of this procedure. Which response by the nurse best
describes the purpose of a vagotomy?
A

Halts stress reactions

Heals the gastric mucosa

Reduces the stimulus to acid secretions

Decreases food absorption in the stomach

Question 21
The nurse is caring for a female client following a Billroth II procedure. Which
postoperative order should the nurse question and verify?
A

Leg exercises

Early ambulation

Irrigating the nasogastric tube

Coughing and deep-breathing exercises

Question 22
The nurse is providing discharge instructions to a male client following gastrectomy
and instructs the client to take which measure to assist in preventing dumping
syndrome?
A

Ambulate following a meal

Eat high carbohydrate foods

Limit the fluid taken with meal

Sit in a high-Fowlers position during meals

Question 23
The nurse is monitoring a female client for the early signs and symptoms of
dumping syndrome. Which of the following indicate this occurrence?
A

Sweating and pallor

Bradycardia and indigestion

Double vision and chest pain

Abdominal cramping and pain

Question 24
The nurse is preparing a discharge teaching plan for the male client who had
umbilical hernia repair. What should the nurse include in the plan?
A

Irrigating the drain

Avoiding coughing

Maintaining bed rest

Restricting pain medication

Question 25
The nurse is instructing the male client who has an inguinal hernia repair how to
reduce postoperative swelling following the procedure. What should the nurse tell
the client?
A

Limit oral fluid

Elevate the scrotum

Apply heat to the abdomen

Remain in a low-fiber diet

Question 26
The nurse is caring for a hospitalized female client with a diagnosis of ulcerative
colitis. Which finding, if noted on assessment of the client, would the nurse report to
the physician?
A

Hypotension

Bloody diarrhea

Rebound tenderness

A hemoglobin level of 12 mg/dL

Question 27

The nurse is caring for a male client postoperatively following creation of a


colostomy. Which nursing diagnosis should the nurse include in the plan of care?
A

Sexual dysfunction

Body image, disturbed

Fear related to poor prognosis

Nutrition: more than body requirements, imbalanced

Question 28
The nurse is reviewing the record of a female client with Crohns disease. Which
stool characteristics should the nurse expect to note documented in the clients
record?
A

Diarrhea

Chronic constipation

Constipation alternating with diarrhea

Stools constantly oozing from the rectum

Question 29
The nurse is performing a colostomy irrigation on a male client. During the
irrigation, the client begins to complain of abdominal cramps. What is the
appropriate nursing action?
A

Notify the physician

Stop the irrigation temporarily

Increase the height of the irrigation

Medicate for pain and resume the irrigation

Question 30
The nurse is teaching a female client how to perform a colostomy irrigation. To
enhance the effectiveness of the irrigation and fecal returns, what measure should
the nurse instruct the client to do?
A

Increase fluid intake

Place heat on the abdomen

Perform the irrigation in the evening

Reduce the amount of irrigation solution

Once you are finished, click the button below. Any items you have not completed
will be marked incorrect.

In Text Mode: All questions and answers are given for reading and answering at your own pace.
You can also copy this exam and make a print out.
1. Nurse Berlinda is assigned to a 41-year-old client who has a diagnosis of chronic
pancreatitis. The nurse reviews the laboratory result, anticipating a laboratory report that
indicates a serum amylase level of:
A. 45 units/L
B. 100 units/L
C. 300 units/L
D. 500 units/L
2. A male client who is recovering from surgery has been advanced from a clear liquid diet
to a full liquid diet. The client is looking forward to the diet change because he has been
bored with the clear liquid diet. The nurse would offer which full liquid item to the
client?
A. Tea
B. Gelatin
C. Custard
D. Popsicle
3. Nurse Juvy is caring for a client with cirrhosis of the liver. To minimize the effects of the
disorder, the nurse teaches the client about foods that are high in thiamine. The nurse
determines that the client has the best understanding of the dietary measures to follow if
the client states an intention to increase the intake of:
A. Pork
B. Milk
C. Chicken
D. Broccoli
4. Nurse Oliver checks for residual before administering a bolus tube feeding to a client
with a nasogastric tube and obtains a residual amount of 150 mL. What is appropriate
action for the nurse to take?

A. Hold the feeding


B. Reinstill the amount and continue with administering the feeding
C. Elevate the clients head at least 45 degrees and administer the feeding
D. Discard the residual amount and proceed with administering the feeding
5. A nurse is inserting a nasogastric tube in an adult male client. During the procedure, the
client begins to cough and has difficulty breathing. Which of the following is the
appropriate nursing action?
A. Quickly insert the tube
B. Notify the physician immediately
C. Remove the tube and reinsert when the respiratory distress subsides
D. Pull back on the tube and wait until the respiratory distress subsides
6. Nurse Ryan is assessing for correct placement of a nasogastric tube. The nurse aspirates
the stomach contents and check the contents for pH. The nurse verifies correct tube
placement if which pH value is noted?
A. 3.5
B. 7.0
C. 7.35
D. 7.5
7. A nurse is preparing to remove a nasogastric tube from a female client. The nurse should
instruct the client to do which of the following just before the nurse removes the tube?
A. Exhale
B. Inhale and exhale quickly
C. Take and hold a deep breath
D. Perform a Valsalva maneuver
8. Nurse Joy is preparing to administer medication through a nasogastric tube that is
connected to suction. To administer the medication, the nurse would:
A. Position the client supine to assist in medication absorption
B. Aspirate the nasogastric tube after medication administration to maintain patency
C. Clamp the nasogastric tube for 30 minutes following administration of the medication
D. Change the suction setting to low intermittent suction for 30 minutes after medication
administration

9. A nurse is preparing to care for a female client with esophageal varices who has just has
a Sengstaken-Blakemore tube inserted. The nurse gathers supplies, knowing that which of
the following items must be kept at the bedside at all times?
A. An obturator
B. Kelly clamp
C. An irrigation set
D. A pair of scissors
10. Dr. Smith has determined that the client with hepatitis has contracted the
infection from contaminated food. The nurse understands that this client is most likely
experiencing what type of hepatitis?
A. Hepatitis A
B. Hepatitis B
C. Hepatitis C
D. Hepatitis D
11. A client is suspected of having hepatitis. Which diagnostic test result will assist in
confirming this diagnosis?
A. Elevated hemoglobin level
B. Elevated serum bilirubin level
C. Elevated blood urea nitrogen level
D. Decreased erythrocyte sedimentation rate
12. The nurse is reviewing the physicians orders written for a male client admitted to the
hospital with acute pancreatitis. Which physician order should the nurse question if noted
on the clients chart?
A. NPO status
B. Nasogastric tube inserted
C. Morphine sulfate for pain
D. An anticholinergic medication
13. A female client being seen in a physicians office has just been scheduled for a barium
swallow the next day. The nurse writes down which instruction for the client to follow
before the test?
A. Fast for 8 hours before the test
B. Eat a regular supper and breakfast

C. Continue to take all oral medications as scheduled


D. Monitor own bowel movement pattern for constipation
14. The nurse is performing an abdominal assessment and inspects the skin of the
abdomen. The nurse performs which assessment technique next?
A. Palpates the abdomen for size
B. Palpates the liver at the right rib margin
C. Listens to bowel sounds in all for quadrants
D. Percusses the right lower abdominal quadrant
15. Polyethylene glycol-electrolyte solution (GoLYTELY) is prescribed for the female client
scheduled for a colonoscopy. The client begins to experience diarrhea following
administration of the solution. What action by the nurse is appropriate?
A. Start an IV infusion
B. Administer an enema
C. Cancel the diagnostic test
D. Explain that diarrhea is expected
16. The nurse is caring for a male client with a diagnosis of chronic gastritis. The nurse
monitors the client knowing that this client is at risk for which vitamin deficiency?
A. Vitamin A
B. Vitamin B12
C. Vitamin C
D. Vitamin E
17. The nurse is reviewing the medication record of a female client with acute gastritis.
Which medication, if noted on the clients record, would the nurse question?
A. Digoxin (Lanoxin)
B. Furosemide (Lasix)
C. Indomethacin (Indocin)
D. Propranolol hydrochloride (Inderal)
18. The nurse is assessing a male client 24 hours following a cholecystectomy. The nurse
noted that the T tube has drained 750 mL of green-brown drainage since the surgery.
Which nursing intervention is appropriate?
A. Clamp the T tube
B. Irrigate the T tube

C. Notify the physician


D. Document the findings
19. The nurse is monitoring a female client with a diagnosis of peptic ulcer. Which
assessment findings would most likely indicate perforation of the ulcer?
A. Bradycardia
B. Numbness in the legs
C. Nausea and vomiting
D. A rigid, board-like abdomen
20. A male client with a peptic ulcer is scheduled for a vagotomy and the client asks the
nurse about the purpose of this procedure. Which response by the nurse best describes the
purpose of a vagotomy?
A. Halts stress reactions
B. Heals the gastric mucosa
C. Reduces the stimulus to acid secretions
D. Decreases food absorption in the stomach
21. The nurse is caring for a female client following a Billroth II procedure. Which
postoperative order should the nurse question and verify?
A. Leg exercises
B. Early ambulation
C. Irrigating the nasogastric tube
D. Coughing and deep-breathing exercises
22. The nurse is providing discharge instructions to a male client following gastrectomy
and instructs the client to take which measure to assist in preventing dumping syndrome?
A. Ambulate following a meal
B. Eat high carbohydrate foods
C. Limit the fluid taken with meal
D. Sit in a high-Fowlers position during meals
23. The nurse is monitoring a female client for the early signs and symptoms of dumping
syndrome. Which of the following indicate this occurrence?
A. Sweating and pallor
B. Bradycardia and indigestion

C. Double vision and chest pain


D. Abdominal cramping and pain
24. The nurse is preparing a discharge teaching plan for the male client who had umbilical
hernia repair. What should the nurse include in the plan?
A. Irrigating the drain
B. Avoiding coughing
C. Maintaining bed rest
D. Restricting pain medication
25. The nurse is instructing the male client who has an inguinal hernia repair how to
reduce postoperative swelling following the procedure. What should the nurse tell the
client?
A. Limit oral fluid
B. Elevate the scrotum
C. Apply heat to the abdomen
D. Remain in a low-fiber diet
26. The nurse is caring for a hospitalized female client with a diagnosis of ulcerative colitis.
Which finding, if noted on assessment of the client, would the nurse report to the
physician?
A. Hypotension
B. Bloody diarrhea
C. Rebound tenderness
D. A hemoglobin level of 12 mg/dL
27. The nurse is caring for a male client postoperatively following creation of a colostomy.
Which nursing diagnosis should the nurse include in the plan of care?
A. Sexual dysfunction
B. Body image, disturbed
C. Fear related to poor prognosis
D. Nutrition: more than body requirements, imbalanced
28. The nurse is reviewing the record of a female client with Crohns disease. Which stool
characteristics should the nurse expect to note documented in the clients record?
A. Diarrhea
B. Chronic constipation

C. Constipation alternating with diarrhea


D. Stools constantly oozing from the rectum
29. The nurse is performing a colostomy irrigation on a male client. During the irrigation,
the client begins to complain of abdominal cramps. What is the appropriate nursing
action?
A. Notify the physician
B. Stop the irrigation temporarily
C. Increase the height of the irrigation
D. Medicate for pain and resume the irrigation
30. The nurse is teaching a female client how to perform a colostomy irrigation. To enhance
the effectiveness of the irrigation and fecal returns, what measure should the nurse instruct
the client to do?
A. Increase fluid intake
B. Place heat on the abdomen
C. Perform the irrigation in the evening
D. Reduce the amount of irrigation solution
Answers and Rationale

1. Answer: C. 300 units/L


The normal serum amylase level is 25 to 151 units/L. With chronic cases of pancreatitis, the rise
in serum amylase levels usually does not exceed three times the normal value. In acute
pancreatitis, the value may exceed five times the normal value. Options A and B are within
normal limits. Option D is an extremely elevated level seen in acute pancreatitis.
2. Answer: C. Custard
Full liquid food items include items such as plain ice cream, sherbet, breakfast drinks, milk,
pudding and custard, soups that are strained, and strained vegetable juices. A clear liquid diet
consists of foods that are relatively transparent. The food items in options A, B, and D are clear
liquids.
3. Answer: A. Pork
The client with cirrhosis needs to consume foods high in thiamine. Thiamine is present in a
variety of foods of plant and animal origin. Pork products are especially rich in this vitamin.

Other good food sources include nuts, whole grain cereals, and legumes. Milk contains vitamins
A, D, and B2. Poultry contains niacin. Broccoli contains vitamins C, E, and K and folic acid
4. Answer: A. Hold the feeding
Unless specifically indicated, residual amounts more than 100 mL require holding the feeding.
Therefore options B, C, and D are incorrect. Additionally, the feeding is not discarded unless its
contents are abnormal in color or characteristics.
5. Answer: D. Pull back on the tube and wait until the respiratory distress subsides
During the insertion of a nasogastric tube, if the client experiences difficulty breathing or any
respiratory distress, withdraw the tube slightly, stop the tube advancement, and wait until the
distress subsides. Options B and C are unnecessary. Quickly inserting the tube is not an
appropriate action because, in this situation, it may be likely that the tube has entered the
bronchus.
6. Answer: A. 3.5
If the nasogastric tube is in the stomach, the pH of the contents will be acidic. Gastric aspirates
have acidic pH values and should be 3.5 or lower. Option B indicates a slightly acidic pH. Option
C indicates a neutral pH. Option D indicates an alkaline pH.
7. Answer: C. Take and hold a deep breath
When the nurse removes a nasogastric tube, the client is instructed to take and hold a deep
breath. This will close the epiglottis. This allows for easy withdrawal through the esophagus into
the nose. The nurse removes the tube with one smooth, continuous pull.
8. Answer: C. Clamp the nasogastric tube for 30 minutes following administration of the
medication
If a client has a nasogastric tube connected to suction, the nurse should wait up to 30 minutes
before reconnecting the tube to the suction apparatus to allow adequate time for medication
absorption. Aspirating the nasogastric tube will remove the medication just administered. Low
intermittent suction also will remove the medication just administered. The client should not be
placed in the supine position because of the risk for aspiration.
9. Answer: D. A pair of scissors
When the client has a Sengstaken-Blakemore tube, a pair of scissors must be kept at the clients
bedside at all times. The client needs to be observed for sudden respiratory distress, which occurs

if the gastric balloon ruptures and the entire tube moves upward. If this occurs, the nurse
immediately cuts all balloon lumens and removes the tube. An obturator and a Kelly clamp are
kept at the bedside of a client with a tracheostomy. An irrigation set may be kept at the bedside,
but it is not the priority item.
10. Answer: A. Hepatitis A
Hepatitis A is transmitted by the fecal-oral route via contaminated food or infected food handlers.
Hepatitis B, C, and D are transmitted most commonly via infected blood or body fluids.
11. Answer: B. Elevated serum bilirubin level
Laboratory indicators of hepatitis include elevated liver enzyme levels, elevated serum bilirubin
levels, elevated erythrocyte sedimentation rates, and leukopenia. An elevated blood urea nitrogen
level may indicate renal dysfunction. A hemoglobin level is unrelated to this diagnosis.
12. Answer: C. Morphine sulfate for pain
Meperidine (Demerol) rather than morphine sulfate is the medication of choice to treat pain
because morphine sulfate can cause spasms in the sphincter of Oddi. Options A, B, and D are
appropriate interventions for the client with acute pancreatitis.
13. Answer: A. Fast for 8 hours before the test
A barium swallow is an x-ray study that uses a substance called barium for contrast to highlight
abnormalities in the gastrointestinal tract. The client should fast for 8 to 12 hours before the test,
depending on physician instructions. Most oral medications also are withheld before the test.
After the procedure, the nurse must monitor for constipation, which can occur as a result of the
presence of barium in the gastrointestinal tract.
14. Answer: C. Listens to bowel sounds in all for quadrants
The appropriate sequence for abdominal examination is inspection, auscultation, percussion, and
palpation. Auscultation is performed after inspection to ensure that the motility of the bowel and
bowel sounds are not altered by percussion or palpation. Therefore, after inspecting the skin on
the abdomen, the nurse should listen for bowel sounds.
15. Answer: D. Explain that diarrhea is expected
The solution GoLYTELY is a bowel evacuant used to prepare a client for a colonoscopy by
cleansing the bowel. The solution is expected to cause a mild diarrhea and will clear the bowel in
4 to 5 hours. Options A, B, and C are inappropriate actions.

16. Answer: B. Vitamin B12


Chronic gastritis causes deterioration and atrophy of the lining of the stomach, leading to the loss
of the function of the parietal cells. The source of the intrinsic factor is lost, which results in the
inability to absorb vitamin B12. This leads to the development of pernicious anemia. The client
is not at risk for vitamin A, C, or E deficiency.
17. Answer: C. Indomethacin (Indocin)
Indomethacin (Indocin) is a nonsteroidal anti-inflammatory drug and can cause ulceration of the
esophagus, stomach, or small intestine. Indomethacin is contraindicated in a client with
gastrointestinal disorders. Furosemide (Lasix) is a loop diuretic. Digoxin is a cardiac medication.
Propranolol (Inderal) is a -adrenergic blocker. Furosemide, digoxin, and propranolol are not
contraindicated in clients with gastric disorders.
18. Answer: D. Document the findings
Following cholecystectomy, drainage from the T tube is initially bloody and then turns to a
greenish-brown color. The drainage is measured as output. The amount of expected drainage will
range from 500 to 1000 mL/day. The nurse would document the output.
19. Answer: D. A rigid, board-like abdomen
Perforation of an ulcer is a surgical emergency and is characterized by sudden, sharp, intolerable
severe pain beginning in the mid epigastric area and spreading over the abdomen, which
becomes rigid and board-like. Nausea and vomiting may occur. Tachycardia may occur as
hypovolemic shock develops. Numbness in the legs is not an associated finding.
20. Answer: C. Reduces the stimulus to acid secretions
A vagotomy, or cutting of the vagus nerve, is done to eliminate parasympathetic stimulation of
gastric secretion. Options A, B, and D are incorrect descriptions of a vagotomy.
21. Answer: C. Irrigating the nasogastric tube
In a Billroth II procedure, the proximal remnant of the stomach is anastomosed to the proximal
jejunum. Patency of the nasogastric tube is critical for preventing the retention of gastric
secretions. The nurse should never irrigate or reposition the gastric tube after gastric surgery,
unless specifically ordered by the physician. In this situation, the nurse should clarify the order.
Options A, B, and D are appropriate postoperative interventions.
22. Answer: C. Limit the fluid taken with meal

Dumping syndrome is a term that refers to a constellation of vasomotor symptoms that occurs
after eating, especially following a Billroth II procedure. Early manifestations usually occur
within 30 minutes of eating and include vertigo, tachycardia, syncope, sweating, pallor,
palpitations, and the desire to lie down. The nurse should instruct the client to decrease the
amount of fluid taken at meals and to avoid high-carbohydrate foods, including fluids such as
fruit nectars; to assume a low-Fowlers position during meals; to lie down for 30 minutes after
eating to delay gastric emptying; and to take antispasmodics as prescribed.
23. Answer: A. Sweating and pallor
Early manifestations of dumping syndrome occur 5 to 30 minutes after eating. Symptoms include
vertigo, tachycardia, syncope, sweating, pallor, palpitations, and the desire to lie down.
24. Answer: B. Avoiding coughing
Coughing is avoided following umbilical hernia repair to prevent disruption of tissue integrity,
which can occur because of the location of this surgical procedure. Bed rest is not required
following this surgical procedure. The client should take analgesics as needed and as prescribed
to control pain. A drain is not used in this surgical procedure, although the client may be
instructed in simple dressing changes.
25. Answer: B. Elevate the scrotum
Following inguinal hernia repair, the client should be instructed to elevate the scrotum and apply
ice packs while in bed to decrease pain and swelling. The nurse also should instruct the client to
apply a scrotal support when out of bed. Heat will increase swelling. Limiting oral fluids and a
low-fiber diet can cause constipation.
26. Answer: C. Rebound tenderness
Rebound tenderness may indicate peritonitis. Bloody diarrhea is expected to occur in ulcerative
colitis. Because of the blood loss, the client may be hypotensive and the hemoglobin level may
be lower than normal. Signs of peritonitis must be reported to the physician.
27. Answer: B. Body image, disturbed
Body image, disturbed relates to loss of bowel control, the presence of a stoma, the release of
fecal material onto the abdomen, the passage of flatus, odor, and the need for an appliance
(external pouch). No data in the question support options A and C. Nutrition: less than body
requirements, imbalanced is the more likely nursing diagnosis.
28. Answer: A. Diarrhea

Crohns disease is characterized by nonbloody diarrhea of usually not more than four to five
stools daily. Over time, the diarrhea episodes increase in frequency, duration, and severity.
Options B, C, and D are not characteristics of Crohns disease.
29. Answer: B. Stop the irrigation temporarily
If cramping occurs during a colostomy irrigation, the irrigation flow is stopped temporarily and
the client is allowed to rest. Cramping may occur from an infusion that is too rapid or is causing
too much pressure. The physician does not need to be notified. Increasing the height of the
irrigation will cause further discomfort. Medicating the client for pain is not the appropriate
action in this situation.
30. Answer: A. Increase fluid intake
To enhance effectiveness of the irrigation and fecal returns, the client is instructed to increase
fluid intake and to take other measures to prevent constipation. Options B, C and D will not
enhance the effectiveness of this procedure.

5. Topics
Included topics in this practice quiz are:

Pancreatitis

Liver Cirrhosis

Hepatitis

Guidelines
Follow the guidelines below to make the most out of this exam:

Read each question carefully and choose the best answer.

You are given one minute per question. Spend your time wisely!

Answers and rationales are given below. Be sure to read them.

If you need more clarifications, please direct them to the comments section.

In Exam Mode: All questions are shown in random and the results, answers and
rationales (if any) will only be given after youve finished the quiz. You are given 1
minute per question, a total of 30 minutes for this exam.

NCLEX Exam: Gastrointestinal Disorders 5 (30 Items)

29:15

Question 1

A male client with pancreatitis complains of pain. The nurse expects the
physician to prescribe meperidine (Demerol) instead of morphine to relieve
pain because:

A meperidine provides a better, more prolonged analgesic effect.


B morphine may cause spasms of Oddis sphincter.
C meperidine is less addictive than morphine.
D morphine may cause hepatic dysfunction.
Question 2

To prevent gastroesophageal reflux in a male client with hiatal hernia, the


nurse should provide which discharge instruction?

Lie down after meals to promote digestion.

Avoid coffee and alcoholic beverages.

Take antacids with meals.

Limit fluid intake with meals.

Question 3

A female client with dysphagia is being prepared for discharge. Which


outcome indicates that the client is ready for discharge?

The client doesnt exhibit rectal tenesmus

The client is free from esophagitis and achalasia

The client reports diminished duodenal inflammation

The client has normal gastric structures

Question 4

The nurse is monitoring a female client receiving paregoric to treat diarrhea


for drug interactions. Which drugs can produce additive constipation when
given with an opium preparation?

Antiarrhythmic drugs

Anticholinergic drugs

Anticoagulant drugs

Antihypertensive drugs

Question 5

A male client is recovering from an ileostomy that was performed to treat


inflammatory bowel disease. During discharge teaching, the nurse should
stress the importance of:

increasing fluid intake to prevent dehydration

wearing an appliance pouch only at bedtime

consuming a low-protein, high-fiber diet

taking only enteric-coated medications

Question 6

Which of the following factors can cause hepatitis A?

Contact with infected blood

Blood transfusions with infected blood

Eating contaminated shellfish

Sexual contact with an infected person

Question 7

Which diagnostic test would be used first to evaluate a client with upper GI
bleeding?

Endoscopy

Upper GI series

Hemoglobin (Hb) levels and hematocrit (HCT)

Arteriography

Question 8

A male client has undergone a colon resection. While turning him, wound
dehiscence with evisceration occurs. The nurses first response is to:

call the physician

place saline-soaked sterile dressings on the wound

take a blood pressure and pulse

pull the dehiscence closed

Question 9

A female client who has just been diagnosed with hepatitis A asks, How
could I have gotten this disease? What is the nurses best response?

A You may have eaten contaminated restaurant food.


B You could have gotten it by using I.V. drugs.
C You must have received an infected blood transfusion.
D You probably got it by engaging in unprotected sex.
Question 10

A male client with extreme weakness, pallor, weak peripheral pulses, and
disorientation is admitted to the emergency department. His wife reports that
he has been spitting up blood. A Mallory-Weiss tear is suspected, and the
nurse begins taking a client history from the clients wife. The question by the
nurse that demonstrates her understanding of Mallory-Weiss tearing is:

A Tell me about your husbands alcohol usage.


B Is your husband being treated for tuberculosis?
C Has your husband recently fallen or injured his chest?
D Describe spices and condiments your husband uses on food.
Question 11

Mandy, an adolescent girl is admitted to an acute care facility with severe


malnutrition. After a thorough examination, the physician diagnoses anorexia
nervosa. When developing the plan of care for this client, the nurse is most
likely to include which nursing diagnosis?
Hopelessness

Powerlessness

Chronic low self esteem

Deficient knowledge

Question 12

A female client with viral hepatitis A is being treated in an acute care facility.
Because the client requires enteric precautions, the nurse should:

A place the client in a private room


B wear a mask when handling the clients bedpan
C wash the hands after touching the client
D wear a gown when providing personal care for the client
Question 13

While palpating a female clients right upper quadrant (RUQ), the nurse would
expect to find which of the following structures?

Sigmoid colon

Appendix

Spleen

Liver

Question 14

The nurse is caring for a male client with cirrhosis. Which assessment findings
indicate that the client has deficient vitamin K absorption caused by this
hepatic disease?

Dyspnea and fatigue

Ascites and orthopnea

Purpura and petechiae

Gynecomastia and testicular atrophy

Question 15

While a female client is being prepared for discharge, the nasogastric (NG)
feeding tube becomes cloggeD. To remedy this problem and teach the clients
family how to deal with it at home, what should the nurse do?

Irrigate the tube with cola

Advance the tube into the intestine

Apply intermittent suction to the tube

Withdraw the obstruction with a 30-ml syringe

Question 16

Nurse Hannah is teaching a group of middle-aged men about peptic ulcers.


When discussing risk factors for peptic ulcers, the nurse should mention:

a sedentary lifestyle and smoking.

a history of hemorrhoids and smoking

alcohol abuse and a history of acute renal failure

alcohol abuse and smoking

Question 17

The nurse is caring for a female client with active upper GI bleeding. What is
the appropriate diet for this client during the first 24 hours after admission?

Regular diet

Skim milk

Nothing by mouth

Clear liquids

Question 18

When preparing a male client, age 51, for surgery to treat appendicitis, the
nurse formulates a nursing diagnosis of Risk for infection related to
inflammation, perforation, and surgery. What is the rationale for choosing this
nursing diagnosis?

Obstruction of the appendix may increase venous drainage and cause the
appendix to rupture

Obstruction of the appendix reduces arterial flow, leading to ischemia,


inflammation, and rupture of the appendix

The appendix may develop gangrene and rupture, especially in a middle-aged


client

D Infection of the appendix diminishes necrotic arterial blood flow and increases

venous drainage
Question 19

When evaluating a male client for complications of acute pancreatitis, the


nurse would observe for:

increased intracranial pressure

decreased urine output

bradycardia

hypertension

Question 20

A male client is recovering from a small-bowel resection. To relieve pain, the


physician prescribes meperidine (Demerol), 75 mg I.M. every 4 hours. How
soon after administration should meperidine onset of action occur?

5 to 10 minutes

15 to 30 minutes

30 to 60 minutes

2 to 4 hours

Question 21

A male client undergoes total gastrectomy. Several hours after surgery, the
nurse notes that the clients nasogastric (NG) tube has stopped draining. How
should the nurse respond?

Notify the physician

Reposition the tube

Irrigate the tube

Increase the suction level

Question 22

The nurse caring for a client with small-bowel obstruction would plan to
implement which nursing intervention first?

Administering pain medication

Obtaining a blood sample for laboratory studies

Preparing to insert a nasogastric (NG) tube

Administering I.V. fluids

Question 23

A male client has just been diagnosed with hepatitis A. On assessment, the
nurse expects to note:

severe abdominal pain radiating to the shoulder

anorexia, nausea, and vomiting

eructation and constipation

abdominal ascites

Question 24

A male client with a recent history of rectal bleeding is being prepared for a
colonoscopy. How should the nurse position the client for this test initially?

Lying on the right side with legs straight

Lying on the left side with knees bent

Prone with the torso elevated

Bent over with hands touching the floor

Question 25

What laboratory finding is the primary diagnostic indicator for pancreatitis?

Elevated blood urea nitrogen (BUN)

Elevated serum lipase

Elevated aspartate aminotransferase (AST)

Increased lactate dehydrogenase (LD)

Question 26

Which of the following nursing interventions should the nurse perform for a
female client receiving enteral feedings through a gastrostomy tube?

A Change the tube feeding solutions and tubing at least every 24 hours.
B Maintain the head of the bed at a 15-degree elevation continuously.

C Check the gastrostomy tube for position every 2 days.


D Maintain the client on bed rest during the feedings.
Question 27

Which condition is most likely to have a nursing diagnosis of fluid volume


deficit?

Appendicitis

Pancreatitis

Cholecystitis

Gastric ulcer

Question 28

A female client with hepatitis C develops liver failure and GI hemorrhage. The
blood products that would most likely bring about hemostasis in the client
are:

whole blood and albumin

platelets and packed red blood cells

fresh frozen plasma and whole blood

cryoprecipitate and fresh frozen plasma

Question 29

A male client with cholelithiasis has a gallstone lodged in the common bile
duct. When assessing this client, the nurse expects to note:

yellow sclera

light amber urine

circumoral pallor

black, tarry stools

Question 30

During preparation for bowel surgery, a male client receives an antibiotic to


reduce intestinal bacteria. Antibiotic therapy may interfere with synthesis of
which vitamin and may lead to hypoprothrombinemia?

vitamin A

vitamin D

vitamin E

vitamin K

Once you are finished, click the button below. Any items you have not
completed will be marked incorrect.

1. During preparation for bowel surgery, a male client receives an antibiotic to reduce
intestinal bacteria. Antibiotic therapy may interfere with synthesis of which vitamin and
may lead to hypoprothrombinemia?
A. vitamin A
B. vitamin D
C. vitamin E
D. vitamin K
2. When evaluating a male client for complications of acute pancreatitis, the nurse would
observe for:
A. increased intracranial pressure.
B. decreased urine output.
C. bradycardia.
D. hypertension.
3. A male client with a recent history of rectal bleeding is being prepared for a colonoscopy.
How should the nurse position the client for this test initially?
A. Lying on the right side with legs straight
B. Lying on the left side with knees bent
C. Prone with the torso elevated
D. Bent over with hands touching the floor
4. A male client with extreme weakness, pallor, weak peripheral pulses, and disorientation
is admitted to the emergency department. His wife reports that he has been spitting up
blood. A Mallory-Weiss tear is suspected, and the nurse begins taking a client history from
the clients wife. The question by the nurse that demonstrates her understanding of
Mallory-Weiss tearing is:
A. Tell me about your husbands alcohol usage.
B. Is your husband being treated for tuberculosis?
C. Has your husband recently fallen or injured his chest?
D. Describe spices and condiments your husband uses on food.

5. Which of the following nursing interventions should the nurse perform for a female
client receiving enteral feedings through a gastrostomy tube?
A. Change the tube feeding solutions and tubing at least every 24 hours.
B. Maintain the head of the bed at a 15-degree elevation continuously.
C. Check the gastrostomy tube for position every 2 days.
D. Maintain the client on bed rest during the feedings.
6. A male client is recovering from a small-bowel resection. To relieve pain, the physician
prescribes meperidine (Demerol), 75 mg I.M. every 4 hours. How soon after administration
should meperidine onset of action occur?
A. 5 to 10 minutes
B. 15 to 30 minutes
C. 30 to 60 minutes
D. 2 to 4 hours
7. The nurse is caring for a male client with cirrhosis. Which assessment findings indicate
that the client has deficient vitamin K absorption caused by this hepatic disease?
A. Dyspnea and fatigue
B. Ascites and orthopnea
C. Purpura and petechiae
D. Gynecomastia and testicular atrophy
8. Which condition is most likely to have a nursing diagnosis of fluid volume deficit?
A. Appendicitis
B. Pancreatitis
C. Cholecystitis
D. Gastric ulcer
9. While a female client is being prepared for discharge, the nasogastric (NG) feeding tube
becomes cloggeD. To remedy this problem and teach the clients family how to deal with it
at home, what should the nurse do?
A. Irrigate the tube with cola.
B. Advance the tube into the intestine.
C. Apply intermittent suction to the tube.
D. Withdraw the obstruction with a 30-ml syringe.
10. A male client with pancreatitis complains of pain. The nurse expects the physician to
prescribe meperidine (Demerol) instead of morphine to relieve pain because:
A. meperidine provides a better, more prolonged analgesic effect.
B. morphine may cause spasms of Oddis sphincter.

C. meperidine is less addictive than morphine.


D. morphine may cause hepatic dysfunction.
11. Mandy, an adolescent girl is admitted to an acute care facility with severe malnutrition.
After a thorough examination, the physician diagnoses anorexia nervosa. When developing
the plan of care for this client, the nurse is most likely to include which nursing diagnosis?
A. Hopelessness
B. Powerlessness
C. Chronic low self esteem
D. Deficient knowledge
12. Which diagnostic test would be used first to evaluate a client with upper GI bleeding?
A. Endoscopy
B. Upper GI series
C. Hemoglobin (Hb) levels and hematocrit (HCT)
D. Arteriography
13. A female client who has just been diagnosed with hepatitis A asks, How could I have
gotten this disease? What is the nurses best response?
A. You may have eaten contaminated restaurant food.
B. You could have gotten it by using I.V. drugs.
C. You must have received an infected blood transfusion.
D. You probably got it by engaging in unprotected sex.
14. When preparing a male client, age 51, for surgery to treat appendicitis, the nurse
formulates a nursing diagnosis of Risk for infection related to inflammation, perforation,
and surgery. What is the rationale for choosing this nursing diagnosis?
A. Obstruction of the appendix may increase venous drainage and cause the appendix to rupture.
B. Obstruction of the appendix reduces arterial flow, leading to ischemia, inflammation, and
rupture of the appendix.
C. The appendix may develop gangrene and rupture, especially in a middle-aged client.
D. Infection of the appendix diminishes necrotic arterial blood flow and increases venous
drainage.
15. A female client with hepatitis C develops liver failure and GI hemorrhage. The blood
products that would most likely bring about hemostasis in the client are:
A. whole blood and albumin.
B. platelets and packed red blood cells.
C. fresh frozen plasma and whole blood.
D. cryoprecipitate and fresh frozen plasma.

16. To prevent gastroesophageal reflux in a male client with hiatal hernia, the nurse should
provide which discharge instruction?
A. Lie down after meals to promote digestion.
B. Avoid coffee and alcoholic beverages.
C. Take antacids with meals.
D. Limit fluid intake with meals.
17. The nurse caring for a client with small-bowel obstruction would plan to implement
which nursing intervention first?
A. Administering pain medication
B. Obtaining a blood sample for laboratory studies
C. Preparing to insert a nasogastric (NG) tube
D. Administering I.V. fluids
18. A female client with dysphagia is being prepared for discharge. Which outcome
indicates that the client is ready for discharge?
A. The client doesnt exhibit rectal tenesmus.
B. The client is free from esophagitis and achalasia.
C. The client reports diminished duodenal inflammation.
D. The client has normal gastric structures.
19. A male client undergoes total gastrectomy. Several hours after surgery, the nurse notes
that the clients nasogastric (NG) tube has stopped draining. How should the nurse
respond?
A. Notify the physician
B. Reposition the tube
C. Irrigate the tube
D. Increase the suction level
20. What laboratory finding is the primary diagnostic indicator for pancreatitis?
A. Elevated blood urea nitrogen (BUN)
B. Elevated serum lipase
C. Elevated aspartate aminotransferase (AST)
D. Increased lactate dehydrogenase (LD)
21. A male client with cholelithiasis has a gallstone lodged in the common bile duct. When
assessing this client, the nurse expects to note:
A. yellow sclera.
B. light amber urine.

C. circumoral pallor.
D. black, tarry stools.
22. Nurse Hannah is teaching a group of middle-aged men about peptic ulcers. When
discussing risk factors for peptic ulcers, the nurse should mention:
A. a sedentary lifestyle and smoking.
B. a history of hemorrhoids and smoking.
C. alcohol abuse and a history of acute renal failure.
D. alcohol abuse and smoking.
23. While palpating a female clients right upper quadrant (RUQ), the nurse would expect
to find which of the following structures?
A. Sigmoid colon
B. Appendix
C. Spleen
D. Liver
24. A male client has undergone a colon resection. While turning him, wound dehiscence
with evisceration occurs. The nurses first response is to:
A. call the physician.
B. place saline-soaked sterile dressings on the wound.
C. take a blood pressure and pulse.
D. pull the dehiscence closed.
25. The nurse is monitoring a female client receiving paregoric to treat diarrhea for drug
interactions. Which drugs can produce additive constipation when given with an opium
preparation?
A. Antiarrhythmic drugs
B. Anticholinergic drugs
C. Anticoagulant drugs
D. Antihypertensive drugs
26. A male client is recovering from an ileostomy that was performed to treat inflammatory
bowel disease. During discharge teaching, the nurse should stress the importance of:
A. increasing fluid intake to prevent dehydration.
B. wearing an appliance pouch only at bedtime.
C. consuming a low-protein, high-fiber diet.
D. taking only enteric-coated medications.
27. The nurse is caring for a female client with active upper GI bleeding. What is the
appropriate diet for this client during the first 24 hours after admission?

A. Regular diet
B. Skim milk
C. Nothing by mouth
D. Clear liquids
28. A male client has just been diagnosed with hepatitis A. On assessment, the nurse expects
to note:
A. severe abdominal pain radiating to the shoulder.
B. anorexia, nausea, and vomiting.
C. eructation and constipation.
D. abdominal ascites.
29. A female client with viral hepatitis A is being treated in an acute care facility. Because
the client requires enteric precautions, the nurse should:
A. place the client in a private room.
B. wear a mask when handling the clients bedpan.
C. wash the hands after touching the client.
D. wear a gown when providing personal care for the client.
30. Which of the following factors can cause hepatitis A?
A. Contact with infected blood
B. Blood transfusions with infected blood
C. Eating contaminated shellfish
D. Sexual contact with an infected person

Answers and Rationale


1. Answer: D. vitamin K
Intestinal bacteria synthesize such nutritional substances as vitamin K, thiamine, riboflavin,
vitamin B12, folic acid, biotin, and nicotinic acid. Therefore, antibiotic therapy may interfere
with synthesis of these substances, including vitamin K. Intestinal bacteria dont synthesize
vitamins A, D, or E.
2. Answer: B. decreased urine output.
Acute pancreatitis can cause decreased urine output, which results from the renal failure that
sometimes accompanies this condition. Intracranial pressure neither increases nor decreases in a
client with pancreatitis. Tachycardia, not bradycardia, usually is associated with pulmonary or
hypovolemic complications of pancreatitis. Hypotension can be caused by a hypovolemic
complication, but hypertension usually isnt related to acute pancreatitis.
3. Answer: B. Lying on the left side with knees bent

For a colonoscopy, the nurse initially should position the client on the left side with knees bent.
Placing the client on the right side with legs straight, prone with the torso elevated, or bent over
with hands touching the floor wouldnt allow proper visualization of the large intestine.
4. Answer: A. Tell me about your husbands alcohol usage.
A Mallory-Weiss tear is associated with massive bleeding after a tear occurs in the mucous
membrane at the junction of the esophagus and stomach. There is a strong relationship between
ethanol usage, resultant vomiting, and a Mallory-Weiss tear. The bleeding is coming from the
stomach, not from the lungs as would be true in some cases of tuberculosis. A Mallory-Weiss tear
doesnt occur from chest injuries or falls and isnt associated with eating spicy foods.
5. Answer: A. Change the tube feeding solutions and tubing at least every 24 hours.
Tube feeding solutions and tubing should be changed every 24 hours, or more frequently if the
feeding requires it. Doing so prevents contamination and bacterial growth. The head of the bed
should be elevated 30 to 45 degrees continuously to prevent aspiration. Checking for
gastrostomy tube placement is performed before initiating the feedings and every 4 hours during
continuous feedings. Clients may ambulate during feedings.
6. Answer: B. 15 to 30 minutes
Meperidines onset of action is 15 to 30 minutes. It peaks between 30 and 60 minutes and has a
duration of action of 2 to 4 hours.
7. Answer: C. Purpura and petechiae
A hepatic disorder, such as cirrhosis, may disrupt the livers normal use of vitamin K to produce
prothrombin (a clotting factor). Consequently, the nurse should monitor the client for signs of
bleeding, including purpura and petechiae. Dyspnea and fatigue suggest anemia. Ascites and
orthopnea are unrelated to vitamin K absorption. Gynecomastia and testicular atrophy result
from decreased estrogen metabolism by the diseased liver.
8. Answer: B. Pancreatitis
Hypovolemic shock from fluid shifts is a major factor in acute pancreatitis. The other conditions
are less likely to exhibit fluid volume deficit.
9. Answer: A. Irrigate the tube with cola.
The nurse should irrigate the tube with cola because its effervescence and acidity are suited to
the purpose, its inexpensive, and its readily available in most homes. Advancing the NG tube is
inappropriate because the tube is designed to stay in the stomach and isnt long enough to reach
the intestines. Applying intermittent suction or using a syringe for aspiration is unlikely to
dislodge the material clogging the tube but may create excess pressure. Intermittent suction may
even collapse the tube.

10. Answer: B. morphine may cause spasms of Oddis sphincter.


For a client with pancreatitis, the physician will probably avoid prescribing morphine because
this drug may trigger spasms of the sphincter of Oddi (a sphincter at the end of the pancreatic
duct), causing irritation of the pancreas. Meperidine has a somewhat shorter duration of action
than morphine. The two drugs are equally addictive. Morphine isnt associated with hepatic
dysfunction.
11. Answer: C. Chronic low self esteem
Young women with Chronic low self esteem are at highest risk for anorexia nervosa because
they perceive being thin as a way to improve their self-confidence. Hopelessness and
Powerlessness are inappropriate nursing diagnoses because clients with anorexia nervosa seldom
feel hopeless or powerless; instead, they use food to control their desire to be thin and hope that
restricting food intake will achieve this goal. Anorexia nervosa doesnt result from a knowledge
deficit, such as one regarding good nutrition.
12. Answer: A. Endoscopy
Endoscopy permits direct evaluation of the upper GI tract and can detect 90% of bleeding
lesions. An upper GI series, or barium study, usually isnt the diagnostic method of choice,
especially in a client with acute active bleeding whos vomiting and unstable. An upper GI series
is also less accurate than endoscopy. Although an upper GI series might confirm the presence of
a lesion, it wouldnt necessarily reveal whether the lesion is bleeding. Hb levels and HCT, which
indicate loss of blood volume, arent always reliable indicators of GI bleeding because a decrease
in these values may not be seen for several hours. Arteriography is an invasive study associated
with life-threatening complications and wouldnt be used for an initial evaluation.
13. Answer: A. You may have eaten contaminated restaurant food.
Hepatitis A virus typically is transmitted by the oral-fecal route commonly by consuming food
contaminated by infected food handlers. The virus isnt transmitted by the I.V. route, blood
transfusions, or unprotected sex. Hepatitis B can be transmitted by I.V. drug use or blood
transfusion. Hepatitis C can be transmitted by unprotected sex.
14. Answer: B. Obstruction of the appendix reduces arterial flow, leading to ischemia,
inflammation, and rupture of the appendix.
A client with appendicitis is at risk for infection related to inflammation, perforation, and surgery
because obstruction of the appendix causes mucus fluid to build up, increasing pressure in the
appendix and compressing venous outflow drainage. The pressure continues to rise with venous
obstruction; arterial blood flow then decreases, leading to ischemia from lack of perfusion.
Inflammation and bacterial growth follow, and swelling continues to raise pressure within the
appendix, resulting in gangrene and rupture. Geriatric, not middle-aged, clients are especially
susceptible to appendix rupture.

15. Answer: D. cryoprecipitate and fresh frozen plasma.


The liver is vital in the synthesis of clotting factors, so when its diseased or dysfunctional, as in
hepatitis C, bleeding occurs. Treatment consists of administering blood products that aid clotting.
These include fresh frozen plasma containing fibrinogen and cryoprecipitate, which have most of
the clotting factors. Although administering whole blood, albumin, and packed cells will
contribute to hemostasis, those products arent specifically used to treat hemostasis. Platelets are
helpful, but the best answer is cryoprecipitate and fresh frozen plasma.
16. Answer: B. Avoid coffee and alcoholic beverages.
To prevent reflux of stomach acid into the esophagus, the nurse should advise the client to avoid
foods and beverages that increase stomach acid, such as coffee and alcohol. The nurse also
should teach the client to avoid lying down after meals, which can aggravate reflux, and to take
antacids after eating. The client need not limit fluid intake with meals as long as the fluids arent
gastric irritants.
17. Answer: D. Administering I.V. fluids
I.V. infusions containing normal saline solution and potassium should be given first to maintain
fluid and electrolyte balance. For the clients comfort and to assist in bowel decompression, the
nurse should prepare to insert an NG tube next. A blood sample is then obtained for laboratory
studies to aid in the diagnosis of bowel obstruction and guide treatment. Blood studies usually
include a complete blood count, serum electrolyte levels, and blood urea nitrogen level. Pain
medication often is withheld until obstruction is diagnosed because analgesics can decrease
intestinal motility.
18. Answer: B. The client is free from esophagitis and achalasia.
Dysphagia may be the reason why a client with esophagitis or achalasia seeks treatment.
Dysphagia isnt associated with rectal tenesmus, duodenal inflammation, or abnormal gastric
structures.
19. Answer: A. Notify the physician
An NG tube that fails to drain during the postoperative period should be reported to the physician
immediately. It may be clogged, which could increase pressure on the suture site because fluid
isnt draining adequately. Repositioning or irrigating an NG tube in a client who has undergone
gastric surgery can disrupt the anastomosis. Increasing the level of suction may cause trauma to
GI mucosa or the suture line.
20. Answer: B. Elevated serum lipase
Elevation of serum lipase is the most reliable indicator of pancreatitis because this enzyme is
produced solely by the pancreas. A clients BUN is typically elevated in relation to renal

dysfunction; the AST, in relation to liver dysfunction; and LD, in relation to damaged cardiac
muscle.
21. Answer: A. yellow sclera.
Yellow sclerae may be the first sign of jaundice, which occurs when the common bile duct is
obstructed. Urine normally is light amber. Circumoral pallor and black, tarry stools dont occur
in common bile duct obstruction; they are signs of hypoxia and GI bleeding, respectively.
22. Answer: D. alcohol abuse and smoking.
Risk factors for peptic (gastric and duodenal) ulcers include alcohol abuse, smoking, and stress.
A sedentary lifestyle and a history of hemorrhoids arent risk factors for peptic ulcers. Chronic
renal failure, not acute renal failure, is associated with duodenal ulcers.
23. Answer: D. Liver
The RUQ contains the liver, gallbladder, duodenum, head of the pancreas, hepatic flexure of the
colon, portions of the ascending and transverse colon, and a portion of the right kidney. The
sigmoid colon is located in the left lower quadrant; the appendix, in the right lower quadrant; and
the spleen, in the left upper quadrant.
24. Answer: B. place saline-soaked sterile dressings on the wound.
The nurse should first place saline-soaked sterile dressings on the open wound to prevent tissue
drying and possible infection. Then the nurse should call the physician and take the clients vital
signs. The dehiscence needs to be surgically closed, so the nurse should never try to close it.
25. Answer: B. Anticholinergic drugs
Paregoric has an additive effect of constipation when used with anticholinergic drugs.
Antiarrhythmics, anticoagulants, and antihypertensives arent known to interact with paregoric.
26. Answer: A. increasing fluid intake to prevent dehydration.
Because stool forms in the large intestine, an ileostomy typically drains liquid waste. To avoid
fluid loss through ileostomy drainage, the nurse should instruct the client to increase fluid intake.
The nurse should teach the client to wear a collection appliance at all times because ileostomy
drainage is incontinent, to avoid high-fiber foods because they may irritate the intestines, and to
avoid enteric-coated medications because the body cant absorb them after an ileostomy
27. Answer: C. Nothing by mouth
Shock and bleeding must be controlled before oral intake, so the client should receive nothing by
mouth. A regular diet is incorrect. When the bleeding is controlled, the diet is gradually
increased, starting with ice chips and then clear liquids. Skim milk shouldnt be given because it

increases gastric acid production, which could prolong bleeding. A liquid diet is the first diet
offered after bleeding and shock are controlled.
28. Answer: B. anorexia, nausea, and vomiting.
Hallmark signs and symptoms of hepatitis A include anorexia, nausea, vomiting, fatigue, and
weakness. Abdominal pain may occur but doesnt radiate to the shoulder. Eructation and
constipation are common in gallbladder disease, not hepatitis A. Abdominal ascites is a sign of
advanced hepatic disease, not an early sign of hepatitis A.
29. Answer: C. wash the hands after touching the client.
To maintain enteric precautions, the nurse must wash the hands after touching the client or
potentially contaminated articles and before caring for another client. A private room is
warranted only if the client has poor hygiene for instance, if the client is unlikely to wash the
hands after touching infective material or is likely to share contaminated articles with other
clients. For enteric precautions, the nurse need not wear a mask and must wear a gown only if
soiling from fecal matter is likely.
30. Answer: C. Eating contaminated shellfish
Hepatitis A can be caused by consuming contaminated water, milk, or food especially
shellfish from contaminated water. Hepatitis B is caused by blood and sexual contact with an
infected person. Hepatitis C is usually caused by contact with infected blood, including receiving
blood transfusions.
Practice Mode: This is an interactive version of the Text Mode. All questions are given in a
single page and correct answers, rationales or explanations (if any) are immediately shown after
you have selected an answer.

NCLEX Exam: Gastrointestinal Disorders 5 (30 Items)


Question 1
During preparation for bowel surgery, a male client receives an antibiotic to reduce
intestinal bacteria. Antibiotic therapy may interfere with synthesis of which vitamin
and may lead to hypoprothrombinemia?
A

vitamin A

vitamin D

vitamin E

vitamin K

Question 2

When evaluating a male client for complications of acute pancreatitis, the nurse
would observe for:
A

increased intracranial pressure

decreased urine output

bradycardia

hypertension

Question 3
A male client with a recent history of rectal bleeding is being prepared for a
colonoscopy. How should the nurse position the client for this test initially?
A

Lying on the right side with legs straight

Lying on the left side with knees bent

Prone with the torso elevated

Bent over with hands touching the floor

Question 4
A male client with extreme weakness, pallor, weak peripheral pulses, and
disorientation is admitted to the emergency department. His wife reports that he
has been spitting up blood. A Mallory-Weiss tear is suspected, and the nurse
begins taking a client history from the clients wife. The question by the nurse that
demonstrates her understanding of Mallory-Weiss tearing is:
A Tell me about your husbands alcohol usage.
B Is your husband being treated for tuberculosis?
C Has your husband recently fallen or injured his chest?
D Describe spices and condiments your husband uses on food.
Question 5
Which of the following nursing interventions should the nurse perform for a female
client receiving enteral feedings through a gastrostomy tube?
A Change the tube feeding solutions and tubing at least every 24 hours.
B Maintain the head of the bed at a 15-degree elevation continuously.
C Check the gastrostomy tube for position every 2 days.

D Maintain the client on bed rest during the feedings.


Question 6
A male client is recovering from a small-bowel resection. To relieve pain, the
physician prescribes meperidine (Demerol), 75 mg I.M. every 4 hours. How soon
after administration should meperidine onset of action occur?
A

5 to 10 minutes

15 to 30 minutes

30 to 60 minutes

2 to 4 hours

Question 7
The nurse is caring for a male client with cirrhosis. Which assessment findings
indicate that the client has deficient vitamin K absorption caused by this hepatic
disease?
A

Dyspnea and fatigue

Ascites and orthopnea

Purpura and petechiae

Gynecomastia and testicular atrophy

Question 8
Which condition is most likely to have a nursing diagnosis of fluid volume deficit?
A

Appendicitis

Pancreatitis

Cholecystitis

Gastric ulcer

Question 9
While a female client is being prepared for discharge, the nasogastric (NG) feeding
tube becomes cloggeD. To remedy this problem and teach the clients family how to
deal with it at home, what should the nurse do?
A

Irrigate the tube with cola

Advance the tube into the intestine

Apply intermittent suction to the tube

Withdraw the obstruction with a 30-ml syringe

Question 10
A male client with pancreatitis complains of pain. The nurse expects the physician
to prescribe meperidine (Demerol) instead of morphine to relieve pain because:
A meperidine provides a better, more prolonged analgesic effect.
B morphine may cause spasms of Oddis sphincter.
C meperidine is less addictive than morphine.
D morphine may cause hepatic dysfunction.
Question 11
Mandy, an adolescent girl is admitted to an acute care facility with severe
malnutrition. After a thorough examination, the physician diagnoses anorexia
nervosa. When developing the plan of care for this client, the nurse is most likely to
include which nursing diagnosis?
A

Hopelessness

Powerlessness

Chronic low self esteem

Deficient knowledge

Question 12
Which diagnostic test would be used first to evaluate a client with upper GI
bleeding?
A

Endoscopy

Upper GI series

Hemoglobin (Hb) levels and hematocrit (HCT)

Arteriography

Question 13
A female client who has just been diagnosed with hepatitis A asks, How could I
have gotten this disease? What is the nurses best response?

A You may have eaten contaminated restaurant food.


B You could have gotten it by using I.V. drugs.
C You must have received an infected blood transfusion.
D You probably got it by engaging in unprotected sex.
Question 14
When preparing a male client, age 51, for surgery to treat appendicitis, the nurse
formulates a nursing diagnosis of Risk for infection related to inflammation,
perforation, and surgery. What is the rationale for choosing this nursing diagnosis?
A

Obstruction of the appendix may increase venous drainage and cause the
appendix to rupture

Obstruction of the appendix reduces arterial flow, leading to ischemia,


inflammation, and rupture of the appendix

The appendix may develop gangrene and rupture, especially in a middle-aged


client

Infection of the appendix diminishes necrotic arterial blood flow and increases
venous drainage

Question 15
A female client with hepatitis C develops liver failure and GI hemorrhage. The blood
products that would most likely bring about hemostasis in the client are:
A

whole blood and albumin

platelets and packed red blood cells

fresh frozen plasma and whole blood

cryoprecipitate and fresh frozen plasma

Question 16
To prevent gastroesophageal reflux in a male client with hiatal hernia, the nurse
should provide which discharge instruction?
A

Lie down after meals to promote digestion.

Avoid coffee and alcoholic beverages.

Take antacids with meals.

Limit fluid intake with meals.

Question 17
The nurse caring for a client with small-bowel obstruction would plan to implement
which nursing intervention first?
A

Administering pain medication

Obtaining a blood sample for laboratory studies

Preparing to insert a nasogastric (NG) tube

Administering I.V. fluids

Question 18
A female client with dysphagia is being prepared for discharge. Which outcome
indicates that the client is ready for discharge?
A

The client doesnt exhibit rectal tenesmus

The client is free from esophagitis and achalasia

The client reports diminished duodenal inflammation

The client has normal gastric structures

Question 19
A male client undergoes total gastrectomy. Several hours after surgery, the nurse
notes that the clients nasogastric (NG) tube has stopped draining. How should the
nurse respond?
A

Notify the physician

Reposition the tube

Irrigate the tube

Increase the suction level

Question 20
What laboratory finding is the primary diagnostic indicator for pancreatitis?
A

Elevated blood urea nitrogen (BUN)

Elevated serum lipase

Elevated aspartate aminotransferase (AST)

Increased lactate dehydrogenase (LD)

Question 21
A male client with cholelithiasis has a gallstone lodged in the common bile duct.
When assessing this client, the nurse expects to note:
A

yellow sclera

light amber urine

circumoral pallor

black, tarry stools

Question 22
Nurse Hannah is teaching a group of middle-aged men about peptic ulcers. When
discussing risk factors for peptic ulcers, the nurse should mention:
A

a sedentary lifestyle and smoking.

a history of hemorrhoids and smoking

alcohol abuse and a history of acute renal failure

alcohol abuse and smoking

Question 23
While palpating a female clients right upper quadrant (RUQ), the nurse would
expect to find which of the following structures?
A

Sigmoid colon

Appendix

Spleen

Liver

Question 24
A male client has undergone a colon resection. While turning him, wound
dehiscence with evisceration occurs. The nurses first response is to:
A

call the physician

place saline-soaked sterile dressings on the wound

take a blood pressure and pulse

pull the dehiscence closed

Question 25
The nurse is monitoring a female client receiving paregoric to treat diarrhea for drug
interactions. Which drugs can produce additive constipation when given with an
opium preparation?
A

Antiarrhythmic drugs

Anticholinergic drugs

Anticoagulant drugs

Antihypertensive drugs

Question 26
A male client is recovering from an ileostomy that was performed to treat
inflammatory bowel disease. During discharge teaching, the nurse should stress the
importance of:
A

increasing fluid intake to prevent dehydration

wearing an appliance pouch only at bedtime

consuming a low-protein, high-fiber diet

taking only enteric-coated medications

Question 27
The nurse is caring for a female client with active upper GI bleeding. What is the
appropriate diet for this client during the first 24 hours after admission?
A

Regular diet

Skim milk

Nothing by mouth

Clear liquids

Question 28
A male client has just been diagnosed with hepatitis A. On assessment, the nurse
expects to note:
A

severe abdominal pain radiating to the shoulder

anorexia, nausea, and vomiting

eructation and constipation

abdominal ascites

Question 29
A female client with viral hepatitis A is being treated in an acute care facility.
Because the client requires enteric precautions, the nurse should:
A place the client in a private room
B wear a mask when handling the clients bedpan
C wash the hands after touching the client
D wear a gown when providing personal care for the client
Question 30
Which of the following factors can cause hepatitis A?
A

Contact with infected blood

Blood transfusions with infected blood

Eating contaminated shellfish

Sexual contact with an infected person

Once you are finished, click the button below. Any items you have not completed
will be marked incorrect.

Potrebbero piacerti anche